Sunteți pe pagina 1din 87

Download All PDF form here :- https://t.

me/pdf4exams

Environmental Science

Target Prelims 2018

Most Important 150 Prelims Model Questions


of Environmental Science from
NEO TPS 2018
 9446331522 www.neoias.com
www.neoias.com
9446331522 info@neoias.com
info@neoias.com

https://t.me/testseries4exams
Download All PDF form here :- https://t.me/pdf4exams
NEO IAS Most Important 150 Environmental Science Model Questions from TPS 2018

 9446331522 9446331522 www.neoias.com Page 1


https://t.me/testseries4exams
Download All PDF form here :- https://t.me/pdf4exams
NEO IAS Most Important 150 Environmental Science Model Questions from TPS 2018

1. Consider the following States of India:


1. Goa
2. Kerala
3. Tamil Nadu
Which of the above states has/have declared Coconut tree as state tree?
(a) 1 and 2 only (b) 3 only (c) 1, 2 and 3 (d) None

Answer (a)
Explanation
 Kerala and Goa are the states which have coconut tree as state tree. The State
government has decided to include coconut palm as tree in the Goa, Daman and Diu
(Preservation of) Trees Act, 1984 to regulate felling of coconut trees under this Act,”
 The population of Tamil Nadu's state tree, palm tree is dwindling. Though every
part of the palm tree (Borassus flabellifer) is useful, its future is uncertain, as
authorities concerned pay very little attention to it.

2. ‘Kambala’, is a type of annual Buffalo race held traditionally in


(a) Andhra Pradesh
(b) Tamil Nadu
(c) Karnataka
(d) Kerala

Answer (c)
Explanation
 Kambala, the traditional slush-track buffalo race of coastal Karnataka, resumed in
Moodbidri in Dakshin Kannada district after a year.
 The Supreme Court on November 6 refused to stay a plea filed by People for the
Ethical Treatment of Animals to ban a legislation in Karnataka that permits the
annual buffalo race.
 The sport is similar to the other controversial traditional game called Jallikattu. The
Kambala season begins in November and goes on till March.
 Kambala is a traditional sport in which water buffaloes are made to race down a
track filled with mud and water.
 Kambala used to be held in coastal regions of Karnataka.
 It was especially popular in Udupi and Dakshina Kannada districts.
 In Kambala, two water buffaloes, yoked together, were made to run at top speed
down a muddy, water-filled route.
 They were guided and whipped along by a man who ran with them.

3. Which of the following statements best describes the term ‘Earth Overshoot Day (EOD)’,
recently seen in the news?
(a) It is an annual environmental campaign that brings attention to the effects of
climate change by asking people to switch of lights at homes and businesses for an
hour on a certain day
(b) The Day designated by the UN General Assembly in 1972 on the first day of United
Nations Conference on the Human Environment
(c) The day which marks the anniversary of the birth of the modern environmental
movement in 1970.
(d) The day when the consumption exceeds the earth’s capacity to regenerate natural
resources.

 9446331522 9446331522 www.neoias.com Page 2


https://t.me/testseries4exams
Download All PDF form here :- https://t.me/pdf4exams
NEO IAS Most Important 150 Environmental Science Model Questions from TPS 2018

Answer (d)
Explanation
Earth Overshoot Day (EOD)
 Humanity is consuming natural resources faster and faster.
 Earth Overshoot Day (EOD), first calculated by economist Andrew Simms.
 EOD represents the day, each year, when the world goes into deficit spending
— ecologically speaking.
 The day the world completely consumes all the natural resources produced
that year is EOD, or the day when our consumption exceeds the earth’s
capacity to regenerate natural resources.
 According to the Global Footprint Network, which calculates EOD every year,
about 60 per cent of the world’s ecological footprint is made up by carbon
emissions, which the world is attempting to curb under the Paris climate change
agreement.
 If we cut carbon emissions in half, the date of Earth Overshoot Day would be
pushed back by 89 days, or about three months.
Earth Hour:
 It is an annual environmental campaign that brings attention to the effects of
climate change by asking people to switch of lights at homes and businesses for
an hour on a certain day
Earth Day:
 Each year, Earth Day—April 22—marks the anniversary of the birth of the
modern environmental movement in 1970.
World Environment day - June 5
 The Day designated by the UN General Assembly in 1972 on the first day of
United Nations Conference on the Human Environment

4. Recently, researchers at the Indian Institute of Technology have used an activated seed
powder to bring the fluoride content in drinking water to less than the WHO limit.
Which one of the following could be that seed?
(a) Jamun
(b) Pappaya
(c) Jackfruit
(d) Cluster been

Answer (a)
Explanation
Activated jamun to remove fluoride from water
 Researchers at the Indian Institute of Technology (IIT) Hyderabad have used
activated jamun seed powder to bring the fluoride content in drinking water to less
than the WHO limit of 1.5 mg per litre.
 The jamun seed powder mix thoroughly with potassium hydroxide and heat it to 900
degree C for an hour to produce activated jamun powder.
 The activated jamun powder can be reused up to five times by heating it to 50
degree C
 About 96% of the fluoride can be desorbed. So there is a loss of only 4% efficiency
after each desorption
 While removing excess fluoride from drinking water, the usual problems such as
high operational costs and getting rid of toxic sludge will be a thing of the past.

 9446331522 9446331522 www.neoias.com Page 3


https://t.me/testseries4exams
Download All PDF form here :- https://t.me/pdf4exams
NEO IAS Most Important 150 Environmental Science Model Questions from TPS 2018

5. With reference to Fly ash, sometimes seen in news, consider the following statements:
1. Fly ash is emitted from coal powered thermal plants.
2. It can be used in making bricks.
Which of the statements given above is/are correct?
(a) 1 only (b) 2 only (c) Both 1 and 2 (d) Neither 1 nor 2

Answer (c)
Explanation
 Fly ash or coal dust is the ash produced in small dark flecks due to the burning of
powdered coal during electricity generation and is harmful to health and
environment.
 This fly ash will be used to make bricks, blocks, tiles, wall panels, cement and
other construction materials.

6. Which one of the following terms best describes the condition where the two sexes of
the same species exhibit different characteristics beyond the differences in their sexual
organs.
(a) Polyploidy
(b) Monogamy
(c) Sexual dimorphism
(d) Polygamy

Answer (c)
Explanation
 Sexual dimorphism, the differences in appearance between males and females of
the same species, such as in colour, shape, size, and structure, that are caused by
the inheritance of one or the other sexual pattern in the genetic material.
 Polyploidy, the condition in which a normally diploid cell or organism acquires one
or more additional sets of chromosomes. In other words, the polyploid cell or
organism has three or more times the haploid chromosome number.
 Monogamy is a form of relationship in which an individual has only one partner
during their lifetime, whereas polygamy has multiple partners.

7. Recently, a plant species was excluded from the definition of tree under the Indian
Forest Act, claiming it would improve the earnings of tribals and dwellers living around
forests. Which one of the following is that plant species?
(a) Neem
(b) Bamboo
(c) Seemai Karuvelam
(d) Jamum tree

Answer (b)
Explanation
 The Parliament passed a Bill to exclude bamboo from the definition of tree under
the Indian Forest Act, claiming it would improve the earnings of tribals and
dwellers living around forests.
 After 90 years, the bamboo has legally ceased to be a tree with the government,
amending the Indian Forest Act and axing the bamboo - taxonomically a grass -
from a list of plants that also included palms, skumps, brush-wood and canes.

 9446331522 9446331522 www.neoias.com Page 4


https://t.me/testseries4exams
Download All PDF form here :- https://t.me/pdf4exams
NEO IAS Most Important 150 Environmental Science Model Questions from TPS 2018

 The government hoped to promote cultivation of bamboo in non-forest areas to


achieve the “twin objectives” of increasing the income of farmers and also increasing
the green cover of the country.
 Bamboo grown in the forest areas would continue to be governed by the
provisions of the Indian Forest Act.

8. Which one of the following is the best description of the term ‘Invasive species’?
(a) Species that move from one habitat to another during different times of the year, as
they cannot live in the same environment all year round due to seasonal limitations
in factors such as food, sunlight, and temperature.
(b) A species that has been introduced by humans intentionally or otherwise through
human agency or accidentally from one region to another.
(c) An alien species which becomes established in natural or semi-natural ecosystems
and is an agent of change, and threatens native biological diversity
(d) Species which are the first to colonize previously disrupted or damaged ecosystems,
beginning a chain of ecological succession that ultimately leads to a more
biologically diverse ecosystem.

Answer (c)
Explanation
 Alien species: An alien species is one that has been introduced by humans
intentionally or otherwise through human agency or accidentally from one region to
another.
 Invasive species: An alien become so successful as to spread in and displace native
biota or threatens valued environmental, agricultural or personal resources by the
damage it causes are considered invasive.
o Invasive species defined by IUCN - “an alien species which becomes established
in natural or semi-natural ecosystems or habitat, is an agent of change, and
threatens native biological diversity”
 Migratory species are species that move from one habitat to another during
different times of the year, as they cannot live in the same environment all year
round due to seasonal limitations in factors such as food, sunlight, and
temperature.
 Pioneer species: Species which are the first to colonize previously disrupted or
damaged ecosystems, beginning a chain of ecological succession that ultimately
leads to a more biodiversity ecosystem.

9. Consider the following pairs


1. Mulberry plant : Silk
2. Cashmere goat : Pashmina
3. Cluster bean : Gaur gum
4. Hemp : Building materials
Which of the above pairs has/have association with each other?
(a) 1, 2 and 3 only (b) 2 and 4 only (c) 1 and 3 only (d) 1, 2, 3 and 4

Answer (d)
Explanation
 Mulberry leaves are the only leaves that silkworms will eat. Hence Silk
production, or sericulture is related with Mulberry plant.

 9446331522 9446331522 www.neoias.com Page 5


https://t.me/testseries4exams
Download All PDF form here :- https://t.me/pdf4exams
NEO IAS Most Important 150 Environmental Science Model Questions from TPS 2018

 Pashmina is a fine type of Kashmiri wool. Pashmina has become known


internationally as a term applied to the wool, and products made from the wool, that
is derived from the undercoat of the ‘Cashmere goat’.
 The gum made from cluster bean (Guar) seeds is used in the extraction of shale
gas. It helps in improving the viscosity and flow of water in the fracking process. The
gum is extracted from guar ki phalli, grown mainly by farmers in arid lands in
Rajasthan and Haryana.
 Hemb refers to the non-psychoactive (less than 1% THC) varieties of Cannabis sativa
L. Both hemp and marijuana come from the same cannabis species, but are
genetically distinct and are further distinguished by use, chemical makeup, and
cultivation methods. Hemp can be grown as a renewable source for raw materials
that can be incorporated into thousands of products.
 Hemp is one of the oldest domesticated crops known to man.
 Its seeds and flowers are used in health foods, organic body care, and other
nutraceuticals.
 Industrial hemp was the desired fiber used to manufacture rope, canvas, paper,
and clothing until alternative textiles and synthetics for these purposes were
discovered.
 The fiber and seeds are incredible valuable and is why hemp is often called a “cash
crop”.
 Hemp building materials are another growing segment of the hemp industry. It is
also used in hemp oils, biofuel, plastic composites, and more.

10. An alliance ‘GrACE' is sometime seen in the news in the context of


(a) Maintenance of Tiger Reserves
(b) Captive breeding of Wild Fauna
(c) Indigenous Satellite Navigation System
(d) Protection of the Eastern Ghats

Answer (d)
Explanation
Greens’ Alliance for Conservation of Eastern Ghats (GrACE)
 Greens Alliance for Conservation Eastern Ghats (GrACE) is an alliance to
provide an ideal platform for the wide range people’s movement for sustainability of
the hill ranges of Eastern Ghats.
 It is launched by Council for Green Revolution, an environmental action group
based at Hyderabad working for the protection of environment and sustainable
development.

11. Sanitary fills and waste heaps are hazardous due to


1. Leachates
2. Emanating gases
Select the correct answer using the code given below:
(a) 1 only (b) 2 only (c) Both 1 and 2 (d) Neither 1 nor 2
Answer (c)
Explanation
 Leachate is the liquid that drains or 'leaches' from a landfill. It varies widely in
composition regarding the age of the landfill and the type of waste that it contains. It
usually contains both dissolved and suspended material.

 9446331522 9446331522 www.neoias.com Page 6


https://t.me/testseries4exams
Download All PDF form here :- https://t.me/pdf4exams
NEO IAS Most Important 150 Environmental Science Model Questions from TPS 2018

 Leachates contaminates the ground water.


 There are lots of toxic gases emitted from landfills, of which methane gas is the
most serious. Methane gas is naturally produced during the process of organic
matter decay.

12. ‘NoMore50’, sometimes seen in the news, is related to


(a) campaign calling for stronger and higher penalties for Tax evading
(b) campaign calling for stronger and higher penalties for animal cruelty
(c) campaign calling for stronger and higher penalties for environmental pollution
(d) campaign calling for stronger and higher penalties for smoking in public place
Answer (b)
Explanation
About NoMore50
 The campaign includes a series of videos from celebrities, urging the ministry of
environment and forests to increase punishment for animal cruelty in the
Prevention of Cruelty to Animals Act, 1960.
 Currently, the maximum penalty even for the most heinous form of animal abuse is
a petty Rs 50.
 NoMore50 is a campaign from the animal rights groups - the Humane Society
International/India and the People for Animals.

13. With reference to ozone, consider the following statements:


1. Ozone helps to sustain life on earth
2. It is formed in the atmosphere through photochemical reaction
3. Ozone depletion will result in minor cooling effect
Which of the statements given above is/are correct?
(a) 1 and 2 only (b) 3 only (c) 1 and 3 only (d) 1, 2 and 3

Answer (d)
Explanation
 Ozone is a gas found in the atmosphere consisting of three oxygen atoms: O3. Ozone
is formed in the atmosphere when energetic ultraviolet (UV) radiation dissociates
molecules of oxygen, O2 , into separate oxygen atoms. Free oxygen atoms can
recombine to form oxygen molecules but if a free oxygen atom collides with an
oxygen molecule, it joins up, forming ozone.
 The ozone hole is an area in the stratosphere above Antarctica where chlorine and
bromine gases from human-produced chlorofluorocarbons (CFCs) and halons have
destroyed ozone molecules.
 Global warming is the rise in average global surface temperature caused primarily
by the build-up of human-produced greenhouses gases, mostly carbon dioxide and
methane, which trap heat in the lower levels of the atmosphere.
 Ozone helps to sustain life on earth: Ozone (O3) absorbs ultraviolet radiation from
the sun, thereby protecting living organisms below from this dangerous radiation.
 It is formed in the atmosphere through photochemical reaction
 Ozone is continuously being produced and destroyed
 The ozone hole and global warming are not the same thing, and neither is the main
cause of the other.
 There are some connections between the two phenomena (ozone hole and Global
warming). For example, the CFCs that destroy ozone are also potent greenhouse
gases.

 9446331522 9446331522 www.neoias.com Page 7


https://t.me/testseries4exams
Download All PDF form here :- https://t.me/pdf4exams
NEO IAS Most Important 150 Environmental Science Model Questions from TPS 2018

 The ozone hole itself has a minor cooling effect (about 2 percent of the warming
effect of greenhouses gases) because ozone in the stratosphere absorbs heat radiated
to space by gases in a lower layer of Earth’s atmosphere (the upper troposphere).
The loss of ozone means slightly more heat can escape into space from that
region.
 Global warming is also predicted to have a modest impact on the Antarctic
ozone hole. The chlorine gases in the lower stratosphere interact with tiny cloud
particles that form at extremely cold temperatures — below -80 degrees Celsius (-
112 degrees Fahrenheit). While greenhouse gases absorb heat at a relatively low
altitudes and warm the surface, they actually cool the stratosphere. Near the South
Pole, this cooling of the stratosphere results in an increase in polar stratospheric
clouds, increasing the efficiency of chlorine release into reactive forms that can
rapidly deplete ozone.

14. With reference to ‘pacha chedi’ (Neurocalyx calycinus), sometimes seen in the news,
consider the following statements:
1. It is a medicinal plant endemic to the southern parts of Western Ghats and Sri
Lanka.
2. It has a potential for developing anti-cancer drug.
Which of the statements given above is/are correct?
(a) 1 only (b) 2 only (c) Both 1 and 2 (d) Neither 1 nor 2

Answer (c)
Explanation
 Neurocalyx calycinus is a medicinal plant endemic to the southern parts of
Western Ghats and Sri Lanka.
 It could offer scientists the key to new herbal formulations and modern drugs for the
treatment of cancer and wounds and burns.
 Scientists at the Jawaharlal Nehru Tropical Botanic Garden and Research Institute
(JNTBGRI) here have confirmed the multiple therapeutic properties of Neurocalyx
calycinus used by the Cholanaickan tribe, one of the particularly vulnerable groups
in Kerala, to treat inflammations and wounds.
 The researchers have filed for a patent on a novel herbal drug formulation
possessing wound-healing, burn-healing, anti-cancer, analgesic, anti-inflammatory,
immuno- enhancing, platelet-augmentation and anti-oxidant effects.
 The presence of high Vitamin E content and potent cytoprotective activity in cell
lines in the plant species have also enhanced the prospects of developing an anti-
cancer drug.

15. Consider the following statements regarding the Earth hour:


1. Earth hour is a global environmental movement by WWF.
2. As a part of earth hour the people switch of the lights on certain day of March
3. The One Planet City Challenge is a year-long competition that rewards cities for their
long term efforts to combat climate change.
Which of the statements given above is/are correct?
(a) 1 only (b) 2 and 3 only (c) 1 and 3 only (d) 1, 2 and 3

Answer (d)
Explanation

 9446331522 9446331522 www.neoias.com Page 8


https://t.me/testseries4exams
Download All PDF form here :- https://t.me/pdf4exams
NEO IAS Most Important 150 Environmental Science Model Questions from TPS 2018

 Earth Hour is an annual environmental campaign that brings attention to the effects
of climate change by asking people to switch of lights at homes and businesses for
an hour on a certain day
 The mission of Earth Hour is uniting people to protect the planet.
 WWF is the parent organisation of Earth Hour and started Earth Hour with teams
and partners in Sydney, Australia back in 2007.
 Earth Hour was famously started as a lights-off event in Sydney, Australia in 2007.
 Earth hour is a global environmental movement by WWF.
 Earth Hour is a people's movement inspiring individuals from more than 7000
cities across the globe to take a stand against climate change, making it the
largest voluntary movement ever witnessed in history.
 Even though this is an open source campaign, the Earth Hour brand is legally
trademarked worldwide and licensed for use by WWF delivering the Earth Hour
movement.
 The WWFs annual Earth Hour City Challenge (EHCC) is a year-long competition
that highlights and rewards cities for their substantial long-term efforts to
combat climate change.
 The One Planet City Challenge, previously known as the Earth Hour City
Challenge, invities cities in participating countries to report ambitious and
innovative climate actions, and to demonstrate how they are delivering on the 2015
Paris Agreement.
 One Planet City Challenge is an initiative designed by WWF to mobilize action and
support from cities in the global transition towards a 100% renewable future. It
also aims to stimulate the development and dissemination of best practices for
climate mitigation and adaptation.

16. Increased biological oxygen demand is an indication of


1. Low microbial contamination
2. Absence of microbial pollution
3. High level of microbial contamination
Which of the statements given above is/ are correct?
(a) 1 only (b) 2 only (c) 3 only (d) None

Answer (c)
Explanation
 The amount of oxygen required by bacteria to break down the organic matter
present in a certain volume of a sample of water is called Biochemical Oxygen
Demand (BOD).
 The amount of BOD in the water is a measure of the amount of organic material in
the water, in terms of how much oxygen will be required to break it down
biologically.
 Clean water would have BOD value of less than 5 ppm whereas highly polluted
water could have a BOD value of 17 ppm or more.

17. If an animal is included in the Vermin category, what does it imply?


(a) The animal comes under the Schedule I of Wildlife (Protection) Act, 1972
(b) That particular animal can be culled as it cause’s harm to crops and humans
(c) That particular animal is a man-eater wild animal
(d) The animal is given a high degree of protection by Wildlife (Protection) Act, 1972
Answer (b)
Explanation
 When a species be declared Vermin under the Wildlife Act so that they could be
culled.

 9446331522 9446331522 www.neoias.com Page 9


https://t.me/testseries4exams
Download All PDF form here :- https://t.me/pdf4exams
NEO IAS Most Important 150 Environmental Science Model Questions from TPS 2018

 The reason for declaring them as vermin is that, the species may be causing harm
to the humans and damaging their agriculture.
 The main reason for this problem is their number is allegedly increasing or in
another word the carrying capacity is increased, so there is less resources for the
species, hence they are migrating to human fields for their survival.
 As per Section 62 of the Wildlife Protection Act, 1972, States can send a list of
wild animals to the Centre requesting it to declare them vermin for selective
slaughter.
 The Central Government may by notification, declare any wild animal other than
those specified in Schedule I and part 11 of Schedule II of the law to be vermin
for any area for a given period of time.
 As long as the notification is in force such wild animal shall be included in
Schedule V of the law, depriving them of any protection under that law.

18. Which one of the following is the online portal aiming to increase awareness to build
sustainable and energy efficient homes in the country
(a) SRISTI
(b) ECO-NIWAS
(c) BIOTA
(d) SAMEER

Answer (b)
Explanation
ECO-NIWAS:
 On the occasion of Energy Conservation Day, President unveiled the online portal,
ECO-NIWAS.
 ECO-NIWAS (Energy Conservation – New Indian Way for Affordable &
Sustainable homes) is an online portal to increase awareness to build sustainable
and energy efficient homes in the country.
SRISTI:
 The Ministry of New and Renewable Energy has proposed SRISTI (Sustainable
Rooftop Implementation for Solar Transfiguration of India), the scheme to incentivise
the installation of roof top solar projects in India.
BIOTA:
 Researchers of the CV Raman Laboratory of Ecological Informatics, which is part
of the Indian Institute of Information Technology and Management – Kerala
(IIITM-K), have developed BIOTA, a biodiversity app.
 App is aimed at gathering information on geographical distribution of species for
conservation and educational purposes.
 The app also aims at building up a database on different plant and animal
species.
SAMEER:
 Sameer, an APP provides hourly updates on the National Air Quality Index (AQI)
published by CPCB. Air Quality Index is a tool for effective communication of air
quality in single number, nomenclature and colour.

19. If you travel through the Western Ghats, you are likely to see which of the following
plants naturally growing there?
1. Rhododendron
2. Chenkurinji
3. Sandalwood
 9446331522 9446331522 www.neoias.com Page 10
https://t.me/testseries4exams
Download All PDF form here :- https://t.me/pdf4exams
NEO IAS Most Important 150 Environmental Science Model Questions from TPS 2018

Select the correct answer using the code given below.


(a) 1 and 2 only (b) 3 only (c) 1 and 3 only (d) 1, 2 and 3

Answer (d)
Explanation
 In the Nilgiris, Annamalai and Palani hills of the Western Ghats one can find the
bright red flowers of the Rhododendron arboreum plant, the only rhododendron in
the 1,600 km mountainous stretch. Right across the subcontinent, the Himalayas
are home to nearly 90 species of rhododendrons including a different sub-species of
the one found in the Western Ghats.
 The marvel is that two closely related species of plants are found in two different
ends of the Indian subcontinent. It could be, as some scientists suggest, the
rhododendron found in the Western Ghats is a Glacial Relict - plants and animals
that have survived from the ice age in a given territory because of particularly
favourable conditions found in a micro-habitat.
 There are many plants, animals, birds and even insects that are found in the
Western Ghats, with their closest relatives found thousands of km away in the
Himalayas or even further in Southeast Asia. One of the commonly held theories
about the occurrence of the species is that they are remnants from the last
glaciation event.
 The Nilgiri rhododendron is endemic to the Western Ghats.
 The peak and its surrounding places are abundant with Rhododendron; an
evergreen woody plant which yields scarlet flowers. Unlike Neelakurinji,
Rhododendron blooms every season in Meesapulimala and the Rhodo valley gets its
name from these plants.
 Sandalwood is grown in the Western Ghats. It is grown in Karnataka, Gujarat,
Madhya Pradesh, Orissa, and parts of Tamil Nadu, and Kerala.
 Agasthyamalai BR consist of mostly tropical forests. A main endemic tree is
Gluta travancornica (Chenkurinji) found in the shenderney valley. The other
Endemic Floras are Rudraksha tree, Black plums, Gaub tree, Wild dhaman.
Agastyamalai is also home to the Kanikaran, one of the oldest surviving ancient
tribes in the world. Their traditional knowledge on medicinal plants is acknowledge,
especially the usage of medicinal plant Arogyapacha (Trichopus zeylanicus) in
immune enhancer drug.

20. With reference to Microbeads, consider the following statements.


1. They are tiny bits of plastics
2. Even though it is an environmental pollutant, it doesn’t affect the food chain
3. They are found in exfoliating body washes, facial scrubs as well as in toothpastes.
Which of the above statements is/are correct?
(a) 1 only (b) 2 and 3 only (c) 1 and 3 only (d) 1, 2 and 3

Answer (c)
Explanation
 Microbeads are pieces of plastic, usually spherical in shape, that range in width
from a fraction of a millimeter to about a millimeter and a quarter.
 They're used in soaps because exfoliating products need small, hard particles to
rub debris from the skin.
 These particles can be natural materials, such as ground nut shells or crushed
apricot seeds--or they can be manufactured products like microbeads.

 9446331522 9446331522 www.neoias.com Page 11


https://t.me/testseries4exams
Download All PDF form here :- https://t.me/pdf4exams
NEO IAS Most Important 150 Environmental Science Model Questions from TPS 2018

 While microbeads are no better at scrubbing the skin than particles of shells or
seeds, they're much cheaper to mass-produce. Which is why, since the 1990s,
manufacturers have increasingly replaced natural materials with plastic shards.
 Microbeads have even made their way into certain toothpastes.
 Plastics used in microbeads readily absorb pollutants. And to a hungry aquatic
organism, little pieces of plastic look pretty tasty. The smallest microbeads can even
become snacks for plankton, and travel all the way up the food chain.
 These beads—while great at scraping dead dermis—are equally adept at killing
marine life and bringing harmful chemicals into the food chain.
 Microbeads don't just contain pollutants; the plastic can also release BPA and
other chemical additives.
 Usage: They can be found in chewing gum, certain toothpastes, industrial cleaning
products, synthetic clothing fibers and tires.
 Microbeads act as environmental pollutants, several leading companies in the
personal care industry have pledged to stop putting plastic microbeads in their
products

21. Which of the following is correct with reference to the natural vegetation of India.
(a) Jatropha curcas is a water intensive plant
(b) Balsam species are part of Himalayan vegetation
(c) Seemai karuvelam is an important plant native to wetlands of India
(d) Even though the Rhododendron is adapted to cold and mountain habitats it is not
found in Himalayan regions

Answer (b)
Explanation
 Biodiesel in India is mostly produced from the oils extracted from the seeds of
Jatropha plant. Jatropha has a high adaptability for thriving under a wide range
of physiographic and climatic conditions. It is found to grow in almost all parts
of the country up to an elevation 3000 feet. Jatropha is a perennial plant,
suitable for all soils including degraded and barren lands. It occupies limited
space hence is highly suitable for intercropping. Extensive research has shown
that jatropha requires low water and fertilizer for cultivation, is not grazed by
cattle or sheep, is pest resistant, is easily propagated, has a low gestation
period and has a high seed yield and oil content.
 Scientists have discovered four new balsam species from various locations in
Eastern Himalayas in northeastern state of Arunachal Pradesh. Balsams are
commonly known as jewel weeds because of diverse colour of their flowers. They are
distributed throughout the Eastern Himalayas and the Western Ghats, Sri Lanka,
South East Asia, Africa and Madagascar.
 Foundation stone of Rhodendron Park was laid in Tawang district of Arunachal
Pradesh by Chief Minister. Tawang district of Arunachal Pradesh was once home to
about 100 species of rhododendron but now they have been reduced to only 50 plus
species due to construction activities in border areas.

 9446331522 9446331522 www.neoias.com Page 12


https://t.me/testseries4exams
Download All PDF form here :- https://t.me/pdf4exams
NEO IAS Most Important 150 Environmental Science Model Questions from TPS 2018

22. With reference to National Ganga Council, consider the following statements:
1. NGRBA has been dissolved consequent to the constitution of National Ganga
Council
2. The Ministry of Water Resources, River Development and Ganga Rejuvenation
(MoWR, RD & GR) is the nodal Ministry for the National Ganga Council.
3. National Ganga Council is under the chairmanship of Prime Minister of India.
Which of the statements given above is/are correct?
(a) 1 only (b) 2 and 3 only (c) 1 and 3 only (d) 1, 2 and 3

Answer (d)
Explanation
 National Mission for Clean Ganga (NMCG) was registered as a society on 12th
August 2011 under the Societies Registration Act 1860.
 It acted as implementation arm of National Ganga River Basin Authority
(NGRBA) which was constituted under the provisions of the Environment
(Protection) Act (EPA), 1986.
 NGRBA has since been dissolved with effect from the 7th October 2016, consequent
to constitution of National Council for Rejuvenation, Protection and
Management of River Ganga (referred as National Ganga Council) vide
notification.
 National Ganga Council is under chairmanship of Hon’ble Prime Minister of
India.
 The Ministry of Water Resources, River Development and Ganga Rejuvenation
(MoWR, RD & GR) is the nodal Ministry for the National Ganga Council.

23. With reference to GRIHA and LEED, consider the following statements:
1. GRIHA and LEED are green building certification programmes
2. GRIHA attempts to minimize a building’s resource consumption, waste generation,
and overall ecological impact
3. GRIHA is a rating tool to assess the performance of building against certain
nationally acceptable benchmarks whereas LEED is rating system which gives
certification to corporate headquarters only
Which of the statements given above is/are correct?
(a) 1 and 2 only (b) 2 only (c) 3 only (d) 1, 2 and 3

Answer (a)
Explanation
 The three green building certification programmes in the country, the Indian
Council of Green Building (ICGB) rating, Green Rating for Integrated Habitat
Assessment (GRIHA) and Leadership in Energy and Environmental Design
(LEED), are not products of government initiative.
 LEED was developed by the non-profit US Green Building Council while the ICGB
and GRIHA are independent platforms, endorsed by the ministry of new and
renewable energy. The government has very little oversight over the three
programmes.
 GRIHA attempts to minimize a building’s resource consumption, waste
generation, and overall ecological impact to within certain nationally acceptable
limits / benchmarks.
 Going by the old adage ‘what gets measured, gets managed’, GRIHA attempts to
quantify aspects such as energy consumption, waste generation, renewable

 9446331522 9446331522 www.neoias.com Page 13


https://t.me/testseries4exams
Download All PDF form here :- https://t.me/pdf4exams
NEO IAS Most Important 150 Environmental Science Model Questions from TPS 2018

energy adoption, etc. so as to manage, control and reduce the same to the best
possible extent.
 GRIHA is a rating tool that helps people assesses the performance of their building
against certain nationally acceptable benchmarks. It evaluates the
environmental performance of a building holistically over its entire life cycle,
thereby providing a definitive standard for what constitutes a ‘green building’.
 The rating system, based on accepted energy and environmental principles, will seek
to strike a balance between the established practices and emerging concepts, both
national and international.
 Leaders around the world have made LEED the most widely used third-party
verification for green buildings.
 LEED works for all buildings—from homes to corporate headquarters—at all
phases of development. Projects pursuing LEED certification earn points across
several areas that address sustainability issues. Based on the number of points
achieved, a project then receives one of four LEED rating levels: Certified, Silver,
Gold and Platinum.
 LEED-certified buildings are resource efficient. They use less water and energy and
reduce greenhouse gas emissions. As an added bonus, they save money. LEED
continues to be the leading benchmark in green building.

24. With reference to ‘Jatropha Curcas’, consider the following statements:


1. Biodiesel in India is mostly produced from the oils extracted from its seeds.
2. It is found to grow at an elevation above 3000 feet as it is a coniferous tree.
3. It is an edible oilseed.
Which of the statements given above is/are correct?
(a) 1 only (b) 1 and 3 only (c) 2 and 3 only (d) 1, 2 and 3

Answer (a)
Explanation
 Biodiesel in India is mostly produced from the oils extracted from the seeds of
Jatropha, mainly because of the fact that edible oil is scarce and the country
already depends on huge quantity of imported oils for edible purposes.
 Apart from Jatropha, Pongamiapinnata, Mahua, Neem and Castor are also
considered as good source of non-edible oil-based biodiesel in India.
 In Western countries, biodiesel is typically made from vegetable oil (rapeseed oil,
sunflower oil and palm oil), animal tallow and used cooking oil. Rapeseed oil has 82
percent of the share of the world’s biodiesel feedstock followed by sunflower oil,
soybean and palm oil.
 Jatropha is a non-edible oil-bearing plant genus of nearly 175 species of shrubs,
low-growing plants, and trees.
 The Jatropha curcas plant is indigenous to parts of Central America, however it
has spread to other tropical and subtropical regions in Africa and Asia.
 Jatropha curcas is a perennial shrub that, on average, grows approximately three
to five meters in height.
 By virtue of being a member of the Euphorbiacea family, Jatropha has a high
adaptability for thriving under a wide range of physiographic and climatic
conditions.
 It is found to grow in almost all parts of the country up to an elevation 3000
feet.
 Jatropha is suitable for all soils including degraded and barren lands.
 It occupies limited space hence is highly suitable for intercropping.

 9446331522 9446331522 www.neoias.com Page 14


https://t.me/testseries4exams
Download All PDF form here :- https://t.me/pdf4exams
NEO IAS Most Important 150 Environmental Science Model Questions from TPS 2018

 Extensive research has shown that jatropha requires low water and fertilizer for
cultivation, is not grazed by cattle or sheep, is pest resistant, is easily
propagated, has a low gestation period and has a high seed yield and oil
content.
 It also produces high protein manure.

25. Which one of the following terms describes the process that use plants to clean up
the soil, air, and water which have been contaminated by chemicals?
(a) Bioremediation
(b) Phytoremediation
(c) Eutrophication
(d) Nitrification

Answer (b)
Explanation
 Phytoremediation refers to the technologies that use plants to clean up the soil,
air, and water which have been contaminated by chemicals.
 Phytoremediation is a cost-efficient plant based approach that takes advantage of
the ability of plants to concentrate elements and compounds from the
environment and metabolize various molecules in their tissues.
 It refers to the natural ability of certain plants to bioaccumulate, degrade, or
render harmless contaminants in soil, water, or air.
 Toxic heavy metals and organic pollutants are the targets for
phytoremediation.
 Recently, researchers from RIKEN Center for Sustainable Resource Science (CSRS)
in Japan found a moss (Funaria hygrometrica) that is known to grow well in sites
contaminated with metals like copper, zinc, and lead.
 Bioremediation: "Remediate" means to solve a problem, and "bio-remediate" means
to use biological organisms to solve an environmental problem such as
contaminated soil or groundwater. Bioremediation works by providing the pollution-
eating organisms with fertilizer, oxygen, and other conditions that encourage their
rapid growth. These organisms would then be able to break down the organic
pollutant at a correspondingly faster rate. In fact, bioremediation is often used to
help clean up oil spills. Bioremediation provides a good cleanup strategy for some
types of pollution, but as you might expect, it will not work for all. For example,
bioremediation may not provide a feasible strategy at sites with high concentrations
of chemicals that are toxic to most microorganisms. These chemicals include metals
such as cadmium or lead, and salts such as sodium chloride.

26. With reference to National Green Tribunal (NGT), consider the following statements:
1. It is a 'quasi-judicial' body exclusively deals with the environment related civil
litigations.
2. National Green Tribunal has been established under the Environmental (Protection)
Act, 1986.
3. Orders by NGT can only be challenged in the Supreme Court.
Which of the statements given above is/ are correct?
(a) 1 and 3 only (b) 2 only (c) 2 and 3 only (d) 1, 2 and 3

Answer (a)
Explanation
 9446331522 9446331522 www.neoias.com Page 15
https://t.me/testseries4exams
Download All PDF form here :- https://t.me/pdf4exams
NEO IAS Most Important 150 Environmental Science Model Questions from TPS 2018

 The National Green Tribunal has been established on 18.10.2010 under the
National Green Tribunal Act 2010 for effective and expeditious disposal of
cases relating to environmental protection and conservation of forests and
other natural resources including enforcement of any legal right relating to
environment and giving relief and compensation for damages to persons and
property and for matters connected therewith or incidental thereto.
 It is a specialized body equipped with the necessary expertise to handle
environmental disputes involving multi-disciplinary issues.
 The Tribunal shall not be bound by the procedure laid down under the Code of Civil
Procedure, 1908, but shall be guided by principles of natural justice.
 Tribunals such as NGT are given a ‘quasi-judicial’ status — giving them a power
to stop executive orders and order punitive action.
 Orders by NGT can only be challenged in the Supreme Court.
 The National Green Tribunal started with a principal bench in Delhi in 2010 and
has now expanded to four zonal benches in each region of the country and four
circuit benches -Shimla, Shillong, Jodhpur and Kochi.

27. LED machine for fish catch are banned by the center because of
(a) The LED stream passes up to two kilometres under water attracting fishes that
makes the fish catch very easy, leading to overexploitation of the marine resource.
(b) LED lights causes death for some marine life
(c) After one usage of the machine it is dumped to the marine leading to water pollution
(d) LED machines are causing skin disorders in trawlers

Answer (a)
Explanation
Why LED lights a concern for fisher folks?
 The LED stream passes up to two kilometres under water, attracting fishes.
 It makes the fish catch very easy, leading to overexploitation of the marine resource.
 Over 400 such trawlers are collecting fishes worth Rs. 4 crore every day.
 Trawlers with LED machine are banned by the center.

28. Other than indiscriminate cutting, what are the possible reasons for the very high
mortality rate in the population of Khejri tree, a Keystone species found in Rajasthan?
1. Decrease in groundwater level
2. Fungal attacks
3. Use of mechanical ploughs
4. Lopping off of branches for fodder
Select the correct answer using the code given below.
(a) 1 and 3 only (b) 2 only (c) 1, 2 and 4 only (d) 1, 2, 3 and 4

Answer (d)
Explanation
 Prosopis cineraria, locally called as Khejri or Jandi is an indigenous tree, which
effectively stabilizes sand dunes and can withstand periodic burial. Culturally, the
tree holds a very important place in lives Rajasthani people, especially the
Bishnois.
 There is a significant economic and environmental importance too. The Khejri
tree plays a vital role in maintaining the ecosystem of the dreary Thar region
because of its ability to survive in such tough conditions and the different ways

 9446331522 9446331522 www.neoias.com Page 16


https://t.me/testseries4exams
Download All PDF form here :- https://t.me/pdf4exams
NEO IAS Most Important 150 Environmental Science Model Questions from TPS 2018

in which it can be used by the farmers. Apart from being a source of firewood and
fodder, the Khejri also helps in sustaining the nutrient value of the soil and
ensuring a good yield. The khejri tree is adapted to withstand both frost and
drought, and survives both high temperature and low. It is known to improve soil
fertility; it aids nitrate retention in soil.
 It's in April every year that one expects to see the khejri tree, the lifeline of the
desert, bearing fruit. This year (2016), however, the trees, a hardy species of the pea
family, were afflicted by pests which have left the fruit, locally called sangria, unfit
not just for human consumption. Cattle too won't touch it. Dried sangria could
earlier be purchased for about Rs200 a kg. These days, it fetches about Rs 1,000 a
kg - production, it is estimated, has fallen by about 90%.The loss in this year is
mainly because of the mites and the resultant gall formation.
 The number of Khejri trees per hectare in the 12 dry districts of Rajasthan had
dropped to less than 35 per cent. This was happening because of a very high
mortality rate owing to factors like decrease in groundwater level, fungal attacks,
indiscriminate cutting, etc. The frequent and almost complete lopping off of
branches for fodder affects seed production and stresses the trees. Frequent
drought, the increased depth of the water table and use of mechanical ploughs -
which injure the roots - have also affected chances of regeneration. One cause of
the higher mortality of the khejri tree, scientists speculate, could also be the
changing topography, with the Indira Gandhi Canal making water flow through
once arid desert.

29. With reference to Mercury, consider the following statements:


1. Mercury is the only metal that is liquid at room temperature.
2. Once released into environment, mercury bio-accumulates and bio-magnifies up in
the food chain.
3. India is a signatory to Minamata Convention on Mercury.
Which of the above statements given above is/are correct?
(a) 1 only (b) 2 and 3 only (c) 2 only (d) 1, 2 and 3

Answer (d)
Explanation
Mercury (Hg)
 Mercury is considered by experts to be one of the most toxic metals known.
 Once released into environment, mercury bio-accumulates and bio-magnifies
up in the food chain, and easily enters the human body and impacts the nervous
system.
 Mercury is the only metallic element that is liquid at standard conditions for
temperature and pressure (at normal temperatures and pressure).

Minamata Convention on Mercury


 The Minamata Convention is a global treaty to protect human health and the
environment from adverse effects of mercury.
 In October 2013, at a conference in Kumamoto (Japan), the convention was formally
adopted. It was a global, legally binding treaty.

 9446331522 9446331522 www.neoias.com Page 17


https://t.me/testseries4exams
Download All PDF form here :- https://t.me/pdf4exams
NEO IAS Most Important 150 Environmental Science Model Questions from TPS 2018

 India is a signatory to the convention, which signed it on 30 September 2014.


India had actively participated in the negotiating process, making significant
contribution in finalizing the treaty text.
 The Union Cabinet chaired by the Prime Minister has approved the proposal for
ratification of Minamata Convention on Mercury and depositing the instrument
of ratification enabling India to become a Party of the Convention.
 The Minamata Convention on Mercury will be implemented in the context of
sustainable development.
 The Convention protects the most vulnerable from the harmful effects of mercury
and also protects the developmental space of developing countries. Therefore, the
interest of the poor and vulnerable groups will be protected.
 The Minamata Convention on Mercury will further urge enterprises to move to
mercury-free alternatives in products and non-mercury technologies in
manufacturing processes. This will drive research & development, and promote
innovation.
 The major highlights of the Convention include a ban on new mercury mines, the
phase-out of existing ones, the phase-out and phase-down of mercury use in a
number of products and processes. It also addresses interim storage of mercury and
its disposal once it becomes waste, and sites contaminated by mercury as well as
health issues.

30. Recently, a new species of parasitic flowering plant that has no chlorophyll, and is
only the fourth species from the genus Gleadovia to be found in the world is named
after a tribe in India. Which of the following is the tribe named after it?
(a) Cholanaikkan
(b) Mankidia tribe
(c) Kanikaran
(d) Konyak tribe of Nagas

Answer (d)
Explanation
 Scientists have discovered a new species of parasitic flowering plant that has no
chlorophyll, and survives by feeding on another species of plant that does.
 Parasitic plant found in Nagaland has no chlorophyll of its own, survives by stealing
food from its host
 The species, named Gleadovia konyakianorum, in honour of the Konyak tribe of
Nagas, was identified during a botanical exploration earlier this year near Tobu town
of Mon district in eastern Nagaland.
 Gleadovia konyakianorum is a root parasite that grows up to 10 cm in height and
bears white, tubular flowers. This is only the fourth species from the genus
Gleadovia to be found in the world.
 It is a holoparasite [complete parasite] that derives its entire nutritional requirement
from the host plant, which is a Strobilanthes species. The plant was found in the
semi-evergreen forest at an altitude of 1,500-1,600 metres.
 Though it is has no chlorophyll, the plant has a vascular system and extracts its
nutrition from the host plant with the help of a haustorium. A haustorium is a
specialised structure with which plant parasites attach themselves to the tissue of
host plants and derive nutrition.
 9446331522 9446331522 www.neoias.com Page 18
https://t.me/testseries4exams
Download All PDF form here :- https://t.me/pdf4exams
NEO IAS Most Important 150 Environmental Science Model Questions from TPS 2018

31. With reference to a mobile application ‘Roadkills’, Consider the following statements.
1. It helps citizens to report wildlife deaths from road or railway accidents by
uploading geo-tagged photographs to a public forum.
2. It allows citizens to report daily road accidents in a public forum thereby enabling
the Traffic Police to punish rash drivers.
Which of the statements given above is/are correct?
(a) 1 only (b) 2 only (c) Both 1 and 2 (d) Neither 1 nor 2

Answer (a)
Explanation
 Roadkills’ mobile application was launched by the Wildlife Conservation Trust
 It is a mobile-based application that will help citizens to report wildlife deaths from
road or railway line accidents by uploading geo-tagged photographs to a public
forum
 This can be used to identify crucial road or rail stretches that urgently
require mitigation measures.
 It will help to plan infrastructure needs better and devise win-win solutions for
wildlife to make infrastructure development smart and green.
Features
 Users can also include what taxon the animal belongs to (bird, mammal, reptile or
amphibian), the species’ name (if known) and the area where the roadkill was
seen.
 The information from all records reported across India with this citizen science
initiative will be compiled as a database, which can soon be viewed on a map on
the campaign’s website.
 The data will be shared with students, wildlife researchers or infrastructure
agencies who may need it to study patterns of wildlife deaths on roads and railway
lines.

32. Which of the following is/are feature/features of the ‘ASH TRACK’, sometimes seen
in news?
1. It is a mobile application launched by Ministry of Power
2. The App shows coal based power plants situated within radius of 100 km and 300
km from a given location.
3. It will allow user to select power station from where he wants to take fly ash.
4. The app will give plant wise, utility wise and state wise fly ash utilization status in
the country and also details of ash generation and utilization in real time.
Select the correct answer using the code given below.
(a) 1,3 and 4 only (b) 2 and 3 only (c) 1 and 4 only (d) 1, 2, 3 and 4

Answer (d)
Explanation
 Ministry of Power has launched Web based monitoring System and Fly Ash mobile
application named ASH TRACK.
 This platform will enable better management of fly ash produced by thermal power
plants by providing interface between ash producers (thermal power plants) and
potential ash users such as –cement plants, road contractors etc.
 ASH TRACK App shows coal based power plants situated within radius of 100 km
and 300 km from a given location.

 9446331522 9446331522 www.neoias.com Page 19


https://t.me/testseries4exams
Download All PDF form here :- https://t.me/pdf4exams
NEO IAS Most Important 150 Environmental Science Model Questions from TPS 2018

 It will allow user to select power station from where he wants to take fly ash.
 It will also show ash availability, distance from user’s location.
 It will help power plants to see location of prospective fly ash users surrounding it
like – cement plants, NHAI, contractors of Pradhan Mantri Gram Sadak Yojana
(PMGSY) projects, brick producers, etc.
 The app will give plant wise, utility wise and state wise fly ash utilization status
in the country and also details of ash generation and utilization in real time.
 It will allow effective monitoring and reviewing for increasing fly ash utilization.

33. With reference to Bombay Natural History Society (BNHS), which of the following
statements is/are correct?
1. It is a Governmental organisation in India for wildlife research.
2. They are trying to conserve the nature through scientific based research, education
and Public awareness.
3. It is the Partner of BirdLife International in India.
Which of the statements given above is/are correct?
(a) 1 only (b) 2 and 3 only (c) 2 only (d) 1, 2 and 3

Answer (b)
Explanation
 BNHS-India, is a pan-India wildlife research organization, has been promoting the
cause of nature conservation for the past 132 years, since 1883.
 It is an oldest and one of the largest environmental non-governmental
organisations in India.
 BNHS Mission is Conservation of Nature, primarily Biological Diversity through
action based on Research, Education and Public Awareness.
 This society is engaged in collecting information and specimens of fauna and
flora all through the country and played an important role in drawing public
attention to the need of wildlife conservation. During its journey from an
amateurs’ organization to a professional organization, the guiding principle of
BNHS has been that conservation should be based on scientific research.
 BNHS has been designated as a Scientific and Industrial Research Organization
(SIRO) by Department of Science & Technology, Government of India and is the
Partner of BirdLife International in India.
 In the true spirit of sustainable development, BNHS aims to continue its
conservation mission with a professional approach and strong research background,
while closely working with various stakeholders including government, research
institutes, academia, corporates, civil society groups and the general public.

34. With reference to ‘hydroponics technique’, sometimes seen in the news, consider the
following statements.
1. This technique is used in Floating Treatment Wetland to clean and purify the
polluted water body.
2. Hydroponics is the art of growing plants without soil.
3. Hydroponic growing uses mineral nutrient solutions to feed the plants in water.
Which of the statements given above is/are correct?
(a) 1 only (b) 1 and 3 only (c) 2 and 3 only (d) 1, 2 and 3

Answer (d)
Explanation

 9446331522 9446331522 www.neoias.com Page 20


https://t.me/testseries4exams
Download All PDF form here :- https://t.me/pdf4exams
NEO IAS Most Important 150 Environmental Science Model Questions from TPS 2018

 The Floating Treatment Wetland (FTW) was inaugurated on World Wetlands Day
(February 2) in Neknampur Lake in Hyderabad to clean and purify the polluted
water body. Plants planted on FTW can clean the lake by absorbing nitrates and
other pollutants in the water.
 FTW’s working is based on soil-less hydroponics technique.
 Hydroponics is a subset of hydroculture, which is the growing of plants in a soil less
medium, or an aquatic based environment.
 Hydroponics permits plants to grow on sunlight and water.
 Hydroponic growing uses mineral nutrient solutions to feed the plants in water,
without soil.
 There are small holes at bottom of base which facilitates flow of nutrients from water
to plants through biological uptake process.
 Micro-organisms growing on FTW and plant root systems of cleaning agents break
down and consume organic matter in water through microbial decomposition. The
root systems filter out sediments and pollutants, reduce content of these chemicals
from water body.

35. Consider the following statements.


1. Flu-Gas Desulfurization (FGD) is a set of technologies used to remove sulphur-
dioxide (SO2) from exhaust flue gases of fossil-fuel power plants
2. Flue Gases is mixture of gases produced by combustion of fuel and other materials
in power stations and various industrial plants.
Which of the statements given above is/are correct?
(a) 1 only (b) 2 only (c) Both 1 and 2 (d) Neither 1 nor 2

Answer (c)
Explanation
 Flu-Gas Desulfurization (FGD) is a set of technologies used to remove sulphur-
dioxide (SO2) from exhaust flue gases of fossil-fuel power plants, as well as from
the emissions of other SOx emitting processes.
 Common methods used in it are wet scrubbing method, Wet and Dry lime scrubbing
method, Spray-dry scrubbing method, SNOX method, Dry sorbent injection method,
etc.
 For a typical coal-fired power station, FGD system may remove 90% or more of the
SO2 in the flue gases. SO2 emissions are a primary contributor to acid rain and
have been regulated by every industrialized nation in the world.
 Flue Gases is mixture of gases produced by combustion of fuel and other
materials in power stations and various industrial plants and released via flu
(ducts) in atmosphere. It largely contains oxides of nitrogen derived from
combustion of air, sulphur oxides, carbon dioxide, carbon monoxide, water vapour,
excess oxygen, particulate matter like soot.

36. Which one of the following statements concerning the Rotterdam Convention is
correct?
(a) The Convention deals with banning of the human clone experiment
(b) The Convention deals with limiting the use of toxic chemicals
(c) The Convention deals with conservation of natural forests
(d) The Convention deals with reducing nuclear weapon stockpiles

 9446331522 9446331522 www.neoias.com Page 21


https://t.me/testseries4exams
Download All PDF form here :- https://t.me/pdf4exams
NEO IAS Most Important 150 Environmental Science Model Questions from TPS 2018

Answer (b)
Explanation
Rotterdam Convention on the Prior Informed Consent Procedure for Certain
Hazardous Chemicals and Pesticides in International Trade:
 The text of the Rotterdam Convention was adopted on 10 September 1998 by a
Conference of Plenipotentiaries in Rotterdam, the Netherlands. The Convention
entered into force on 24 February 2004.
 The objectives of the Convention are:
o to promote shared responsibility and cooperative efforts among Parties in the
international trade of certain hazardous chemicals in order to protect
human health and the environment from potential harm;
o to contribute to the environmentally sound use of those hazardous chemicals, by
facilitating information exchange about their characteristics, by providing for a
national decision-making process on their import and export and by
disseminating these decisions to Parties.
 The Convention creates legally binding obligations for the implementation of the
Prior Informed Consent (PIC) procedure. It built on the voluntary PIC procedure,
initiated by UNEP and FAO in 1989 and ceased on 24 February 2006.
 The Convention covers pesticides and industrial chemicals that have been banned
or severely restricted for health or environmental reasons by Parties and which
have been notified by Parties for inclusion in the PIC procedure.
 The convention promotes open exchange of information and calls on exporters of
hazardous chemicals to use proper labeling, include directions on safe handling, and
inform purchasers of any known restrictions or bans.

37. With reference to Green Energy, Consider the following statements.


1. Green energy refers to one which does not harm the ecosystem of planet Earth
2. Renewable energy sources are used to generate green energy
Which of the statements given above is/ are correct?
(a) 1 only (b) 2 only (c) Both 1 and 2 (d) Neither 1 nor 2

Answer (c)
Explanation
 'Green electricity' is power produced from sources that do not harm the
environment.
 'Green energy' or 'green electricity' is generated by transforming the natural
energy flows of the Earth such as the sun, water, wind and biomass.
 These are usually known as renewable energy sources, because they will never
run out.
 Green power production technologies are those that that clearly reduce the harmful
environmental impacts of energy generation.

38. In our country, which among the following chemicals are labelled toxic and
prohibited the use of it in the manufacture of firecrackers?
1. Antimony
2. Lithium
3. Mercury
4. Arsenic
5. Lead
Select the correct answer using the code given below

 9446331522 9446331522 www.neoias.com Page 22


https://t.me/testseries4exams
Download All PDF form here :- https://t.me/pdf4exams
NEO IAS Most Important 150 Environmental Science Model Questions from TPS 2018

(a) 1, 2 and 3 only (b) 2, 3 and 4 only (c) 1, 4 and 5 only (d) 1,2,3,4 and 5

Answer (d)
Explanation
 The Supreme Court on 2017 July prohibited the use of five chemicals, labelled as
toxic by the Central Pollution Control Board (CPCB), in the manufacture of
firecrackers.
 The Bench ordered that no firecrackers manufactured by the respondents shall
contain antimony, lithium, mercury, arsenic and lead in any form whatsoever.
 It is the responsibility of the Petroleum and Explosive Safety Organisation (PESO) to
ensure compliance.
 Strontium is another chemical branded toxic in firecrackers.

39. With reference to Convention on the Conservation of Migratory Species of Wild


Animals (CMS), sometimes seen in news, consider the following statements:
1. It is an environmental treaty under the aegis of the United Nations Environment
Programme (UNEP)
2. CMS provides a global platform for the conservation and sustainable use of
migratory animals and their habitats.
3. India and china are not a party of the treaty.
Which of the statements given above is/are correct?
(a) 1 and 2 only (b) 2 only (c) 3 only (d) 1, 2 and 3

Answer (a)
Explanation
 Several species of vultures, including four that have India on their migratory routes,
were awarded the highest protection by the Convention on the Conservation of
Migratory Species of Wild Animals.
 The Asian vultures that are set to get collaborative international protection are the
red-headed vulture, white-rumped vulture, Indian vulture and slender-billed
vulture. They are faced with threats such as poisoning, hunting, collision with
electricity cables and habitat degradation.
About CONVENTION ON THE CONSERVATION OF MIGRATORY SPECIES OF WILD
ANIMALS (CMS) OR BONN CONVENTION
 As an environmental treaty under the aegis of the United Nations Environment
Programme (UNEP), CMS provides a global platform for the conservation and
sustainable use of migratory animals and their habitats.
 As the only global convention specializing in the conservation of migratory
species, their habitats and migration routes, CMS complements and co-operates
with a number of other international organizations, NGOs and partners in the media
as well as in the corporate sector.
 CMS brings together the States through which migratory animals pass, the Range
States, and lays the legal foundation for internationally coordinated conservation
measures throughout a migratory range.
 Migratory species threatened with extinction are listed on Appendix I of the
Convention.
 CMS Parties/states strive towards strictly protecting these animals, conserving or
restoring the places where they live, mitigating obstacles to migration and
controlling other factors that might endanger them.

 9446331522 9446331522 www.neoias.com Page 23


https://t.me/testseries4exams
Download All PDF form here :- https://t.me/pdf4exams
NEO IAS Most Important 150 Environmental Science Model Questions from TPS 2018

 The Agreements may range from legally binding treaties to less formal
instruments, such as Memoranda of Understanding, and can be adapted to the
requirements of particular regions.
 The development of models tailored according to the conservation needs throughout
the migratory range is a unique capacity to CMS.
 India is a party of CMS since 1983.
 More than 120 states are party to the Convention, but this does not include China
and many other Asian countries.
 The Government of India signed the ‘Raptor MoU’, covering 76 species, out of which
46 including vultures, falcons, eagles, owls, hawks, kites, harriers and others are
also found in India. The agreement was signed on March 7, 2016 at the Convention
on Migratory Species (CMS) in Abu Dhabi. The agreement, under Article IV
paragraph 4 of the CMS, is not “legally binding”.

40. India’s National Forest Policy 1988 aims at maintaining


(a) 33 per cent of the geographical area under forest cover
(b) 53 per cent of the geographical area under forest cover
(c) 23 per cent of the geographical area under forest cover
(d) 63 per cent of the geographical area under forest cover

Answer (a)
Explanation
 National Forest Policy 1988 says that the national goal should be to have a
minimum of one-third of the total land area of the country under forest or tree
cover.
 In the hills and in mountainous regions, the aim should be to maintain two-third of
the area under such cover in order to prevent erosion and land degradation and to
ensure the stability of the fragile eco-system.
 It proposed that 60% of the land in the hills and 20% in the plains and in all 33% of
the total geographical area should be under forest/tree cover.

41. With reference to Particulate Matter (PM2.5) Consider the following statements:
1. PM is the mixture of solid particles and liquid droplets found in the air.
2. The PM2.5 causes chronic illnesses such as lung cancer, chronic obstructive
pulmonary disease (COPD) and cardiovascular diseases, that cause premature
death.
3. A lot of the fine particulate matter comes from fuel combustion, both from mobile
sources such as vehicles and from stationary sources such as power plants,
industry, households or biomass burning.
Which of the statements given above is/are correct?
(a) 1 only (b) 1 and 3 only (c) 2 and 3 only (d) 1, 2 and 3

Answer (d)
Explanation
 PM stands for particulate matter (also called particle pollution): the term for a
mixture of solid particles and liquid droplets found in the air.
 Some particles, such as dust, dirt, soot, or smoke, are large or dark enough to be
seen with the naked eye. Others are so small they can only be detected using an
electron microscope.
 The impact of fine particulate matter (PM2.5) highlighted by the World Health
Organisation (WHO) study is felt through a broad spectrum of acute and chronic
illnesses that cause premature death. These include lung cancer, chronic
obstructive pulmonary disease (COPD) and cardiovascular diseases.

 9446331522 9446331522 www.neoias.com Page 24


https://t.me/testseries4exams
Download All PDF form here :- https://t.me/pdf4exams
NEO IAS Most Important 150 Environmental Science Model Questions from TPS 2018

 Of all of pollutants, fine particulate matter has the greatest impact on health. A lot
of the fine particulate matter comes from fuel combustion, both from mobile sources
such as vehicles and from stationary sources such as power plants, industry,
households or biomass burning.
 The particles come in many sizes and shapes and can be made up of hundreds of
different chemicals.
 Some are emitted directly from a source, such as construction sites, unpaved
roads, fields, smokestacks or fires. Most particles form in the atmosphere as a
result of complex reactions of chemicals such as sulfur dioxide and nitrogen oxides,
which are pollutants emitted from power plants, industries and automobiles.
 PM10, or coarse particulate matter with diameter between 2.5 and 10 micrometers,
are primarily made up of dirt and dust from farming, factories and roads, and
caused due to the crushing of rocks and soil.
 A lot of the fine particulate matter comes from fuel combustion, both from
mobile sources such as vehicles and from stationary sources such as coal-fired
power plants, industry, household fuel or biomass burning.

42. Which one of the following terms best describe “the large-scale schemes to tackle
climate change by removing CO2 from the air or limiting the sunlight reaching the
planet”.
(a) Geoengineering
(b) Phytoremediation
(c) Bioremediation
(d) Carbon Sequestration

Answer (a)
Explanation
 Geoengineering (literally "Earth-engineering") is the currently fashionable term for
making large-scale interventions in how the planet works to slow down or reverse
the effects of climate change.
 Geoengineering aims to tackle climate change by removing CO2 from the air or
limiting the sunlight reaching the planet.
 In theory, the word "geoengineering" could be used to describe almost any large-
scale scheme for tackling climate change.
 For example, if millions of people in China all planted a tree on the same day to
capture carbon dioxide (CO2) from the atmosphere, that might alter the planet
enough to be considered geoengineering.

43. Consider the following substances:


1. Lead
2. Mercury
3. Metanil yellow
4. Pesticide residue
Which of the above substances are harmful for health if it is present in food items?
(a) 1 and 2 only (b) 2 and 3 only (c) 1, 3 and 4 only (d) 1, 2, 3 and 4

Answer (d)
Explanation

 9446331522 9446331522 www.neoias.com Page 25


https://t.me/testseries4exams
Download All PDF form here :- https://t.me/pdf4exams
NEO IAS Most Important 150 Environmental Science Model Questions from TPS 2018

 Food contamination can take place at various stages of the food chain, from farm to
table.
 Apart from chemical contamination of food from various sources such as
industries, vehicles, pesticides and fertilizers, pollution resulting from growing
of vegetables in degraded environmental conditions in peri-urban zones also
affects food safety.
 This is coupled with further pollution from vehicles and industries during
marketing.
 The common sources include presence of heavy metals, pesticides, preservatives,
colouring agents and other additives and adulterants in food.
 Lead is a highly toxic metal and a very strong poison. Lead poisoning is a serious
and sometimes fatal condition.
 Mercury is a naturally occurring element found in air, water and soil. A highly toxic
form (methyl mercury) builds up in fish, shellfish and animals that eat fish. Fish
and shellfish are the main sources of methyl mercury exposure to humans. Other
sources of mercury can be silver-colored dental fillings that contain up to 50%
mercury by weight and can release mercury vapor, fluorescent light bulbs that use
electricity to excite mercury vapor, and mercury fever thermometers made of glass.
 Metanil yellow is the principal non-permitted food colour used extensively in India.
The effects of long-term consumption of metanil yellow on the developing and adult
brain were studied using Wistar rats.
 Pesticide residue refers to the pesticides that may remain on or in food after they
are applied to food crops.

44. Consider the following statements:


1. State Government for the purpose of protecting, propagating or developing wild life,
by notification, declare its intention to constitute an area as a National Park.
2. National parks are also declared by Central Government.
3. No alteration of the boundaries of a National Park shall be made except on a
resolution passed by the Legislature of the state.
Which of the statements given above is/are correct?
(a) 1 only (b) 2 and 3 only (c) 1 and 3 only (d) 1, 2 and 3

Answer (d)
Explanation
 National park is an area which is strictly reserved for the betterment of the
wildlife & biodiversity, and where activities like developmental, forestry, poaching,
hunting and grazing on cultivation are not permitted. Their boundaries are well
marked and circumscribed. No alteration of the boundaries of a National Park shall
be made except on a resolution passed by the Legislature of the state.
 Whenever it appears to the State Government that an area, whether within a
Sanctuary or not, is by reason of its ecological, faunal, floral, geomorphological or
zoological association or importance, needed to be constituted as a National Park for
the purpose of protecting, propagating or developing wild life therein or its
environment, it may, by notification, declare its intention to constitute that area as a
National Park.
 No human activity is permitted inside the national park except for the ones
permitted by the Chief Wildlife Warden of the state under the conditions given in
CHAPTER IV, WPA 1972.

 9446331522 9446331522 www.neoias.com Page 26


https://t.me/testseries4exams
Download All PDF form here :- https://t.me/pdf4exams
NEO IAS Most Important 150 Environmental Science Model Questions from TPS 2018

 National Parks have been setup with the legal framework of Wild Life (protection)
Act, 1972.
 Power of Central Government to declare areas as Sanctuaries or National Park:
1. Where the State Government leases or otherwise transfers any area under
its control, not being an area within a Sanctuary, to the Central Government the
Central Government may, if it is satisfied that the conditions specified in sec.18
are fulfilled in relation to the area so transferred to it, declare such area, by
notification, to be a sanctuary and the provisions of [sec 18 to 35 (both
inclusive)], 54 and 55 shall apply in relation to such sanctuary as they
apply in relation to a sanctuary declared by the State Government.
2. The Central Government may, if it is satisfied that the conditions specified in
sec.35 are fulfilled in relation to any area referred to in sub-section (1),
whether or not such area has been declared, to be a sanctuary by the
Central Government, or the State Government, declare such area, by
notification, to be a National Park and the provisions of secs.35. 54 and
55 shall apply to such National Park as they apply in relation to a
National Park declared by the State Government.

45. The poisonous nature of Carbon monoxide (CO) is due to its


(a) insolubility in water
(b) ability to form a complex with haemoglobin
(c) ability to reduce some metal oxides
(d) property of having pungent smell

Answer (b)
Explanation
 Carbon monoxide (CO) is a poisonous, colorless, odorless, and tasteless gas. CO is a
common industrial hazard resulting from the incomplete burning of natural gas and
any other material containing carbon such as gasoline, kerosene, oil, propane, coal,
or wood.
 Carbon monoxide is harmful when breathed because it displaces oxygen in the blood
and deprives the heart, brain, and other vital organs of oxygen.
 Carbon monoxide primarily causes adverse effects by combining with hemoglobin to
form carboxyhemoglobin (HbCO) preventing the blood from carrying oxygen.

46. With reference to National Board for Wildlife (NBWL), consider the following
statements:
1. It is established under the Environment (Protection) Act, 1986.
2. It is a statutory body.
3. It is chaired by the Prime Minister.
Which of the statements given above is/are correct?
(a) 1 only (b) 2 and 3 only (c) 2 only (d) 1, 2 and 3

Answer (b)
Explanation
 National Board for Wildlife (NBWL) is a statutory Board constituted on 22nd
September 2003 under Section 5 of the Wild Life (Protection) Act, 1972.

 9446331522 9446331522 www.neoias.com Page 27


https://t.me/testseries4exams
Download All PDF form here :- https://t.me/pdf4exams
NEO IAS Most Important 150 Environmental Science Model Questions from TPS 2018

 As per the amendment of the Wildlife (Protection) Act in 2002, a provision was
incorporated for the constitution of the National Board for Wildlife, replacing the
Indian Board for Wildlife.
 The NBWL is chaired by the Hon’ble Prime Minister.

47. The largest component of the soil system is


(a) Living organism
(b) Minerals
(c) Soil organic matter
(d) Water

Answer (b)
Explanation
 A soil is simply a porous medium consisting of minerals, water, gases, organic
matter, and microorganisms. The traditional definition is: Soil is a dynamic natural
body having properties derived from the combined effects of climate and biotic
activities, as modified by topography, acting on parent materials over time.
 There are five basic components of soil that, when present in the proper amounts,
are the backbone of all terrestrial plant ecosystems. The components are rocks
(minerals), water, air organic material (leaves and decomposed animals, for example)
and living organisms present in soil.
1. Mineral
 The largest component of soil is the mineral
portion, which makes up
approximately 45% to 49% of the volume.
The rocks and minerals found in soil come from
nonliving, inorganic materials.
2. Water
 Water is the second basic component of soil.
Water is important for transporting nutrients to growing plants and soil organisms
and for facilitating both biological and chemical decomposition. Water in soil usually
contains dissolved salts and other chemicals. Soil water availability is the capacity of
a particular soil to hold water that is available for plant use.
3. Organic matter
 Organic matter is found in soils at levels of approximately 1% to 5%. Organic matter
is derived from dead plants and animals and as such has a high capacity to hold
onto and/or provide the essential elements and water for plant growth.
 Organic matter also influences the water-holding capacity of soils because of organic
matter's high affinity for water. The higher the percentage of organic material in soil,
the higher the soil's water-holding capacity.
 Through decomposition, organic materials are broken down and turned into
nutrients that plants can use.
4. Gases or air
 Gases or air can occupy the same spaces as water. Oxygen is essential for root and
microbe respiration, which helps support plant growth. Carbon dioxide and nitrogen
also are important for belowground plant functions such as for nitrogen-fixing
bacteria.
5. Microorganisms

 9446331522 9446331522 www.neoias.com Page 28


https://t.me/testseries4exams
Download All PDF form here :- https://t.me/pdf4exams
NEO IAS Most Important 150 Environmental Science Model Questions from TPS 2018

 They are found in the soil in very high numbers but make up much less than 1% of
the soil volume.
 A common estimate is that one thimble full of topsoil may hold more than 20,000
microbial organisms. The largest of these organisms are earthworms and nematodes
and the smallest are bacteria, actinomycetes, algae, and fungi.

48. In which one of the following groups of States in India is the Integrated Coastal Zone
Management (ICZM) Project being implemented as a pilot investment?
(a) Gujarat, Kerala and Goa
(b) Kerala, Karnataka and Andhra Pradesh
(c) Gujarat, Odisha and West Bengal
(d) Maharashtra, Andhra Pradesh and Tamil Nadu

Answer (c)
Explanation
Integrated Coastal Zone Management Project (ICSMP)
 The objective of the Integrated Coastal Zone Management (ICZM) Project is to assist
Government of India in building national capacity for implementation of
comprehensive coastal management approach in the country, and piloting the
integrated coastal zone management approach in states of Gujarat, Orissa and
West Bengal.
 The project area covers 13% of West Bengal’s coastline comprising of Sagar Island
and Digha area.
 The two main objectives of the project will be achieved through a combination of
targeted capacity building activities and demonstrative pilot investments.
 The project, under the Integrated Coastal Zone Management Project (ICZMP), was
developed in West Bengal, Odisha and Gujarat as pilot projects between 2011 and
2016, at a total cost of over Rs1,200 crore.
 The Integrated Coastal Zone Management Project consists of four components.
Under the first component, mapping, delineation and demarcation of hazard lines.
Besides this, mapping, delineation and demarcation of environmentally sensitive
areas, capacity building of the Ministry of Environment and Forests and the State
Coastal Zone Management Authorities, and a nation-wide training programme is
also done.
 The second component is the ICZM approach for the coastline of Gulf of Kuchchh.
 The third component includes Orissa
 Fourth component mainly address the coastal zone management issues in three
sectors in West Bengal; Sunderbans, Haldia and Digha-Shankarpur.
 The World Bank funded the ICZM project as The Government of India and the World
Bank signed a loan agreement for the implementation of Integrated Coastal Zone
Project.

49. With reference the global initiative ‘BreathLife’, consider the following statements:
1. BreathLife is a major campaign on air quality.
2. It is a Climate and Clean Air Coalition initiative led by the WHO and UN
Environment.
Which of the statements given above is/are correct?
(a) 1 only (b) 2 only (c) Both 1 and 2 (d) Neither 1 nor 2

 9446331522 9446331522 www.neoias.com Page 29


https://t.me/testseries4exams
Download All PDF form here :- https://t.me/pdf4exams
NEO IAS Most Important 150 Environmental Science Model Questions from TPS 2018

Answer (c)
Explanation
 ‘BreathLife’ is a major campaign on air quality.
 It’s a global campaign for clean air.
 BreatheLife mobilizes communities to reduce the impact of air pollution on our
health & climate.
 BreatheLife is a Climate and Clean Air Coalition initiative led by the WHO and UN
Environment.
 This global campaign aims to mobilize cities and individuals to protect our health
and our planet from the effects of air pollution.

50. With reference to biodiversity hotspot, consider the following statements


1. The concept of biodiversity hotspot was originated from the organisation IUCN
2. Exceptional levels of plant endemism is an important criteria for identifying
biodiversity hot spots
3. Serious levels of habitat loss is the only criteria for identifying biodiversity hotspots
Which of the statements given above is/are correct?
(a) 1 and 2 only (b) 2 only (c) 2 and 3 only (d) 3 only

Answer (b)
Explanation
 Certain areas have large numbers of endemic species — those found nowhere else.
Many of these are heavily threatened by habitat loss and other human activities.
These areas are the biodiversity hotspots.
 The British biologist Norman Myers coined the term biodiversity hotspot in 1988
as a biogeographic region characterised both by exceptional levels of plant
endemism and by serious levels of habitat loss.
 Conservation International (CI) adopted Myers’ hotspots as its institutional
blueprint in 1989, and in 1996, the organization made the decision to undertake a
reassessment of the hotspots concept.
 According to CI, to qualify as a biodiversity hotspot, a region must meet two strict
criteria:
1. It must have at least 1,500 vascular plants as endemics - which is to say, it
must have a high percentage of plant life found nowhere else on the planet. A
hotspot, in other words, is irreplaceable.
2. It must have 30% or less of its original natural vegetation. In other words, it
must be threatened. (in other words, it has to have lost at least 70% of its original
habitat)

51. With reference to Global Snow Leopard and Ecosystem Protection Program (GSLEP),
sometimes seen in the news, consider the following statements:
1. The GSLEP is the world first joint initiative that aims to conserve the snow leopard
2. GSLEP is a joint initiative of range country governments, international agencies, civil
society, and the private sector
3. Bishkek Declaration is related to the snow leopard protection
Which of the statements given above is/are correct?
(a) 1 only (b) 2 and 3 only (c) 1 and 3 only (d) 1, 2 and 3

Answer (d)
Explanation
The Global Snow Leopard and Ecosystem Protection Program (GSLEP).

 9446331522 9446331522 www.neoias.com Page 30


https://t.me/testseries4exams
Download All PDF form here :- https://t.me/pdf4exams
NEO IAS Most Important 150 Environmental Science Model Questions from TPS 2018

 The GSLEP is a world first joint initiative that aims to conserve the endangered
snow leopard within the broader context of also conserving valuable high mountain
ecosystems.
 Its goal is to secure the long-term survival of the snow leopard in its natural
ecosystem.
 GSLEP unites all 12 range country governments, nongovernmental and inter-
governmental organisations, local communities, and the private sector around
this aim.
 The Global Snow Leopard & Ecosystem Protection Plan (GSLEP) is a joint initiative of
range country governments, international agencies, civil society, and the private
sector.
 With the adoption of the Bishkek Declaration, the snow leopard range countries
have pledged to secure 20 snow leopard landscapes by 2020 – the first step to saving
this iconic, endangered cat.
 In 2013 the 12 snow leopard range countries and partners signed the Bishkek
Declaration (in the Kyrgyz Republic capital city) and agreed to the goal of the GSLEP
for the 7 years through 2020. The snow leopard range countries agree, with support
from stakeholder and interested organisations, to work together to identify and
secure at least 20 snow leopard landscapes across the cat’s range by 2020 or, in
short – “Secure 20 by 2020”.

52. A National Park, which lies within one of the global biodiversity hotspots in India,
has the world's third highest peak in it. This park has been recently inscribed as India’s
first Mixed World Heritage Site on UNESCO World Heritage List. Which of the following
could be that National Park?
(a) Sunderbans (b) Bhitarkanika
(c) Khangchendzonga (d) Nandadevi

Answer (c)
Explanation
 Khangchendzonga National Park (KNP), Sikkim has been inscribed as India’s first
“Mixed World Heritage Site” on UNESCO World Heritage List, by fulfilling the
nomination criteria under both natural and cultural heritage.
 The approval was granted at the 40th Session of the UNESCO World Heritage
Committee.
 The KNP exhibits one of the widest altitudinal ranges of any protected area
worldwide.
 The Park has an extraordinary vertical sweep of over 7 kilometres (1,220m to
8,586m) within an area of only 178,400 ha and comprises a unique diversity of
lowlands, steep-sided valleys and specular snow-clad mountains including the
world’s third highest peak, Mt. Khangchendzonga.
 Numerous lakes and glaciers, including the 26 km long Zemu Glacier, dot the
barren high altitudes. The KNP lies within the Himalaya global biodiversity
hotspot and displays an unsurpassed range of sub-tropical to alpine ecosystems.
 The cultural significance of KNP is portrayed by the multi-layered sacred landscape
of Khangchendzonga and the cultural and religious relevance of the hidden land
(beyul in Tibetan Buddhism and Mayel Lyang, in Lepcha tradition) is specific to
Sikkim and is a unique example of co-existence and exchange between different
religious traditional and people.

 9446331522 9446331522 www.neoias.com Page 31


https://t.me/testseries4exams
Download All PDF form here :- https://t.me/pdf4exams
NEO IAS Most Important 150 Environmental Science Model Questions from TPS 2018

53. Which one of the following statements about microbes is not correct?
(a) They are used in sewage treatment plants
(b) They are used in industrial fermenters for the production of beverages
(c) No antibiotics has been obtained from any microbe
(d) They are used to get many bioactive molecules for the treatment of diseases.

Answer (c)
Explanation
 The main component of sewage is organic matter (undigested food) but there are
other substances such as oil, heavy metals, nitrogen and phosphorous compounds
(from artificial fertilisers and detergents) which also have to be removed.
 Microbes has important role in the sewage treatment process. Sewage works rely on
the diverse capabilities of microbes for the complete process to be effective.
 The microbes use the organic material in the sewage as their source of carbohydrate
for respiration.
 Fermentation is the process of controlling microbes (bacteria, yeast, and moulds) to
modify food, producing a desired product. The production of alcohol beverages is a
process that involves the active participation of microorganisms, most often yeasts.
Yeasts are the main fermentor and alcohol producer in the production of wine, beer
and other alcohol drinks.
 Chemical, Enzymes and other Bioactive Molecules are commercially produced by
microbes. Bioactive molecules cyclosporine A, which is used as an
immunosuppressive agent in organ-transplant patients is produced by fungus
Trichoderma polysporium.
 Some microorganisms are harmful [pathogens] as they cause various diseases in
humans. Some other microorganisms are beneficial and contribute to human
welfare. Some microorganisms are sources of medicines like antibiotic tablets,
capsules or injections such as of penicillin. These medicines kill or stop the growth
of the disease-causing microorganisms. Such medicines are called antibiotics. These
days a number of antibiotics are being produced from bacteria and fungi.

54. Which one of the following is not a correct statement regarding Montreux Record
(a) Its a data book which have the names of wetlands that require international help for
conservation
(b) It’s a principal tool of the Ramsar Convention for highlighting those sites where an
adverse change in ecological character has occurred, is occurring, or is likely to
occur
(c) Whatever the wetlands added to the ramsar list, will be automatically added to the
Montreux Record
(d) Right now there are only two wetlands in India which is added in the Montreux
Record
Answer (c)
Explanation
Montreux Record
 As a part of the conservation strategy a data book called Montreaux Record is kept
of all those wetlands that require international help for conservation.
 The inclusion of a site in this list makes it eligible for a global package for
conservation related activities.

 9446331522 9446331522 www.neoias.com Page 32


https://t.me/testseries4exams
Download All PDF form here :- https://t.me/pdf4exams
NEO IAS Most Important 150 Environmental Science Model Questions from TPS 2018

 The Montreux Record (MR) is the principal tool of the Convention for highlighting
those sites where an adverse change in ecological character has occurred, is
occurring, or is likely to occur, as a result of technological developments or
other human interference, and which are therefore in need of priority
conservation attention.
 It shall be maintained as part of the Ramsar Database and shall be subject to
continuous review.
 It is maintained by the Secretariat in consultation with the Contracting Party
concerned (Recommendation 4.8).
 It was established by Recommendation 4.8 (1990) to identify priority sites for
positive national and international conservation attention.
 Ramsar sites in India that included in montrex record are;
 Keoladeo National Park in the Rajasthan.
 Loktak Lake in the Manipur.
 Ramsar site that removed from the Montreux Record in India is, Chilika Lake.

55. Which one of the following is the world’s first sanctuary for white tigers?
(a) Mukundpur
(b) Bandhavgarh
(c) Kanha
(d) Nilgiri

Answer (a)
Explanation
 Mukundpur sanctuary is the world's first white tiger sanctuary, it was opened in
Madhya Pradesh.
 The 25-hectare sanctuary at Mukundpur is 20km from Rewa

56. Consider the following statements with regard to the Forest Right Act, 2006:
1. It recognises the rights of tribes and other forest dwelling communities to manage
forest.
2. It empowers the forest department to manage the forest by giving additional rights.
3. It recognises the tribe’s right of ownership and the right to use and collect the minor
forest produce.
4. It recognises forest dwelling Scheduled tribe’s community right to Intellectual
property and traditional knowledge related to biodiversity and cultural diversity.
Which of the statements given above are correct?
(a) 1 and 2 only (b) 2 and 3 only (c) 1, 3 and 4 only (d) 1, 2, 3 and 4

Answer (c)
Explanation
 2nd statement is wrong. Forest Right Act, 2006 empowers the Gramasabha to
manage the forest
 The Scheduled Tribes and Other Traditional Forest Dwellers (Recognition of
Forest Rights) Act, 2006 is unique in being a legislation that guarantees traditional
local communities, the right over forest land on which they have been dependent
since ancient times. The act recognises the rights of forest dwellers, including
Scheduled Tribes and others, to use, protect and manage forest resources where
they live.

THE FOREST RIGHTS ACT RECOGNIZES AND SECURES-

 9446331522 9446331522 www.neoias.com Page 33


https://t.me/testseries4exams
Download All PDF form here :- https://t.me/pdf4exams
NEO IAS Most Important 150 Environmental Science Model Questions from TPS 2018

 Livelihood and occupancy for individuals and communities for ST’s and Other
Traditional Forest Dwellers. It recognizes the right of ownership and also to use
and collect minor forest produce.
 Also, the rights such as rights to water bodies and flora and fauna.
 Settlement and conversion of all forest villages into revenue villages.
 Right to protect, regenerate, or conserve or manage any community forest
resource, which they have been traditionally protecting and conserving for
sustainable use.
 Intellectual property rights and traditional knowledge related to biodiversity.
 Rehabilitation and settlements rights including the provision to get alternate land
in cases of displacements or evictions.
 Community Forest Resource (CFR) Rights gives the gram sabha the right to
protect and manage their forest. No project can come up in the forest nor can any
conservation plan for the forest be carried out without the approval of the gram
sabha.

57. The most important strategy for the conservation of wetlands together with the
sustainable use is the establishment of
(a) Biosphere reserves
(b) Biodiversity Hotspots
(c) National Parks
(d) Wetlands declared under Ramsar convention

Answer (d)
Explanation
 Wetlands declared under Ramsar convention (Ramsar site): A wetland designated
as international importance under the Ramsar Convention. The Convention
defines wise use of wetlands as “the maintenance of their ecological character,
achieved through the implementation of ecosystem approaches, within the context of
sustainable development”.
 Biosphere reserves: Conservation of biodiversity together with traditional human
life
 Biodiversity Hotspots: Biologically richest and most threatened regions
 National Parks: Protected area which prohibit all kinds of human activities
except tourism.

58. Which one of the following tiger reserve becomes first in India to get official mascot,
by declaring Bhoorsingh as the mascot?
(a) Satpura
(b) Kanha
(c) Pench
(d) Panna

Answer (b)
Explanation
 Kanha has become the first tiger reserve in India to officially introduce a mascot —
Bhoorsingh the Barasingha — to present the hard ground swamp deer as the spirit
of the reserve and spread awareness to save it from possible extinction.
 Barasingha, or swamp deer, is the state animal of Madhya Pradesh.
 The Kanha tiger reserve, spread over Mandla and Balaghat districts, is the place
where the species exists.

 9446331522 9446331522 www.neoias.com Page 34


https://t.me/testseries4exams
Download All PDF form here :- https://t.me/pdf4exams
NEO IAS Most Important 150 Environmental Science Model Questions from TPS 2018

59. With reference to the Wildlife (Protection) Act 1972, consider the following
statements:
1. It was enacted to fulfil India’s international obligation under the Convention on
Biological Diversity.
2. As per the law only ‘a part of the wild animal’ is considered as a wildlife trophy.
3. A person can keep the wildlife trophy with the written permission of a Chief Wildlife
Warden.
Which of the statements given above is/are correct?
(a) 1 only (b) 2 and 3 only (c) 3 only (d) 1, 2 and 3

Answer (c)
Explanation
 In 1972, the Wild Life (Protection) Act (WPA) was enacted for the purpose of
protection of wild animals, birds and plants.
 It was made to prevent hunting and also to control trade in wild life products.
 For the purpose of protecting, propagating or developing wildlife and its
environment, the power is conferred on the State Governments and Central
Government to proclaim wildlife sanctuaries and national parks.
 Section 9 of WPA prohibits hunting of any wild animal specified in Schedule 1, 2,
3 and 4.
 However the Chief Wild Life Warden may permit hunting of wild animals in
certain cases.
 The Wildlife (Protection) Act, 1972, has included antler in the definition of wildlife
trophy.
 A wildlife trophy is defined as the “whole or any part of any captive animal or
wild animal”.
 Section 39 of the Act also states that “no person shall, without the previous
permission in writing of the Chief Wildlife Warden or the authorised officer
acquire or keep in his possession, custody or control or transfer to any person,
whether by way of gift, sale or otherwise or destroy or damage such property.”
 Wildlife and wildlife trophies are considered as owned by the government.
 The Act also prescribes imprisonment up to three years and fine of RS. 25,000 for
offences involving wildlife trophies.

60. Recently there was a proposal to translocate some of the Sangai deers from their
natural habitat in Keibul Lamjao National Park to which one of the following sites?
(a) Pumlen Pat lake
(b) Loktak lake
(c) Wular lake
(d) Harikke Barrage

Answer (a)
Explanation
 The critically endangered species of brow-antlered deer or ‘Sangai’, which is found
only on a floating biomass (Phumdis) on Manipur’s Loktak lake, will soon be
provided a second home to save it from extinction.
 The Manipur Forest Department plans to translocate a section of the rare species of
the endemic deer to fresh water lake Pumlen Pat, which is close to its existing
habitat in Loktak.
 Keibul Lamjao National Park (KLNP), situated inside the loktak lake, is the only
remaining natural habitat of ‘Sangai’ which was once found across the Manipur
valley.

 9446331522 9446331522 www.neoias.com Page 35


https://t.me/testseries4exams
Download All PDF form here :- https://t.me/pdf4exams
NEO IAS Most Important 150 Environmental Science Model Questions from TPS 2018

61. Consider the following statements:


1. Aichi Targets is a short term plan, a ten-year framework for action by all countries to
save biodiversity
2. Cartagena Protocol on Biosafety is the international regulatory framework for safe
transfer, handling and use of Living Modified Organisms (LMOs)
3. The Nagoya Protocol on Access to Genetic Resources and the Fair and Equitable
Sharing of Benefits Arising from their Utilization (ABS) is a supplementary
agreement to the Convention on Biological Diversity.
Which of the statements given above is/ are correct?
(a) 1 only (b) 2 and 3 only (c) 1 and 3only (d) 1, 2 and 3

Answer (d)
Explanation
CONVENTION ON BIOLOGICAL DIVERSITY
 This agreement, adopted at the 1992 Earth Summit in Rio de Janeiro, Brazil, is
part of a comprehensive strategy for sustainable development that is, meeting
the needs of the current generation of human beings while ensuring a healthy and
viable world for future generations. The convention sets three main goals:
conservation of biological diversity, sustainable use of plant and animal species,
and equitable sharing of benefits from the use of genetic resources.
 COP 10 was held at Aichi precinct district of Nagoya, Japan. By this meeting Nagoya
protocol on Genetic Resources and the Aichi Targets for Genetic Resources were
originated. Aichi Targets is a short term plan, a ten-year framework for action by
all countries to save biodiversity, and is officially known as Strategic Plan for
Biodiversity 2011-2020. It provides 20 ambitious yet achievable targets collectively
known as Aichi Targets. These 20 targets are divided into 5 sections. COP 11 held on
Oct 2012 in Hyderabad, India declared the current decade as the United Nations
Decade on Biodiversity and United Nations Decade for Deserts and the Fight
against Desertification.

CARTAGENA PROTOCOL ON BIOSAFETY


 The Cartagena Protocol on Biosafety (CPB), the first international regulatory
framework for safe transfer, handling and use of Living Modified Organisms
(LMOs) was negotiated under the aegis of the Convention on Biological Diversity
(CBD). The objective of the Protocol is to contribute to ensuring an adequate level of
protection in the field of the safe transfer, handling and use of LMOs resulting from
modern biotechnology that may have adverse effects on the conservation and
sustainable use of biological diversity, taking also into account risks to human
health, and specifically focusing on transboundary movements. The Protocol
promotes biosafety by establishing rules and procedures for the safe transfer,
handling, and use of LMOs.

NAGOYA PROTOCOL ON ACCESS AND BENEFIT SHARING (ABS)


 The Nagoya Protocol on Access to Genetic Resources and the Fair and Equitable
Sharing of Benefits Arising from their Utilization (ABS) to the Convention on
Biological Diversity is a supplementary agreement to the Convention on Biological
Diversity. It provides a transparent legal framework for the effective
implementation of one of the three objectives of the CBD: the fair and equitable
sharing of benefits arising out of the utilization of genetic resources. The Nagoya
Protocol on ABS was adopted on 29 October 2010 in Nagoya, Japan and entered
into force on 12 October 2014, 90 days after the deposit of the fiftieth instrument
of ratification. Its objective is the fair and equitable sharing of benefits arising from

 9446331522 9446331522 www.neoias.com Page 36


https://t.me/testseries4exams
Download All PDF form here :- https://t.me/pdf4exams
NEO IAS Most Important 150 Environmental Science Model Questions from TPS 2018

the utilization of genetic resources, thereby contributing to the conservation and


sustainable use of biodiversity.

62. ‘BIOTA 1.0’, sometimes seen in the news, is related to


(a) a biofuel
(b) a satellite in PSLV C37
(c) a biodiversity app
(d) an anti-cancer drug produced form plants

Answer (c)
Explanation
 Researchers of the CV Raman Laboratory of Ecological Informatics, which is part
of the Indian Institute of Information Technology and Management – Kerala
(IIITM-K), have developed BIOTA, a biodiversity app.
 App is aimed at gathering information on geographical distribution of species for
conservation and educational purposes.
 The app also aims at building up a database on different plant and animal
species.
 The beta version of the app, BIOTA 1.0, was released as part of a research paper in
the recently-concluded National Biodiversity Conference in Thiruvananthapuram

63. In which one of the following Mangrove forests do Tigers inhibit?


(a) Bhitarkanika mangroves
(b) Sundarbans
(c) Pichavaram mangroves
(d) None of the above has tigers

Answer (b)
Explanation
 The Sundarbans is the largest area of mangrove forest in the world and the only
one that is inhabited by the tiger.
 The Pichavaram mangroves is in Tamil Nadu
 Bhitarkanika mangroves is in Odisha. It is famous for saltwater crocodile, white
crocodile and olive ridley turtles (Gahirmatha beach).

64. Which of the following areas are identified for the intensive conservation and
management of coral reefs in India?
1. Gulf of Mannar
2. Sundarbans
3. Lakshadweep
4. Gulf of Kutch
5. Andaman and Nicobar
Select the correct answer using the code given below.
(a) 1 and 2 only (b) 1, 3 and 4 only (c) 1, 3, 4 and 5 only (d) 1, 2, 3, 4 and 5

Answer (c)
Explanation
 Coral reefs are the skeletons of stony coral polyps cemented together. It is a diverse
underwater ecosystem held together by calcium carbonate structures secreted by
corals.

 9446331522 9446331522 www.neoias.com Page 37


https://t.me/testseries4exams
Download All PDF form here :- https://t.me/pdf4exams
NEO IAS Most Important 150 Environmental Science Model Questions from TPS 2018

 The four major coral reefs areas identified for intensive conservation and
management in India are: i) Gulf of Mannar, (ii) Gulf of Kutch; (iii) Lakshadweep;
and iv) Andaman and Nicobar Islands.
 The ministry provides financial assistance to the state forest departments for all
the four identified coral reef areas for conservation and management of coral and
associates.
 Besides, the ministry also supports R&D activities with emphasis on targeted
research on coral biodiversity, its management and various aspects of pollution in
these areas.

65. Which of the following Ramsar site has water with exceptional purity due to the
presence of Cavaborus larva?
(a) Kolleru Lake
(b) Tso moriri Lake
(c) Sasthamkotta Lake
(d) Wular Lake

Answer (c)
Explanation
 SASTHAMKOTTA LAKE
Sasthamkotta Lake is the largest freshwater lake in kerala, home to 27 freshwater
fish species. It's also home to a larva, Cavaborus, which contributes to what is
described as its "exceptional purity". It is surrounded by hills, the source of water
is from the underground sprouts and spring-fed.
 Tso moriri Lake : Jammu & Kashmir
 Kolleru Lake : Andhra Pradesh
 Wular Lake : Jammu & Kashmir

66. With reference to the Global Tiger Forum (GTF), which of the following statements
is/are correct?
1. It is an international non-governmental organization.
2. It provides a leadership throughout the world to safeguard the survival of the Tiger,
its prey and its habitat.
3. GTF membership is open to Tiger range countries only
Which of the statements given above is/are correct?
(a) 1 only (b) 1 and 2 only (c) 2 only (d) 1, 2 and 3

Answer (c)
Explanation
 The Global Tiger Forum was established in 1994, with a goal to
highlight the rationale for Tiger preservation and provide leadership and common
approach throughout the world in order to safeguard the survival of the Tiger, its
prey and the habitat.
 The Global Tiger Forum is an inter-governmental international organization for
the conservation of tigers in the wild.
 The GTF is committed to work through mandate ratified by the Tiger Range
Countries (TRCs) to ensure the implementation of the Global Tiger Recovery
Programme (GTRP).
 It is the only inter-governmental and international body functioning exclusively
for the conservation of tigers in the wild.
 GTF is involved in various activities and programs in the tiger range countries.
 The GTF has several members from the Tiger range countries, non-Tiger range
countries, international and national NGOs.

 9446331522 9446331522 www.neoias.com Page 38


https://t.me/testseries4exams
Download All PDF form here :- https://t.me/pdf4exams
NEO IAS Most Important 150 Environmental Science Model Questions from TPS 2018

 The tiger range countries are Bangladesh, Bhutan, Cambodia, China, India,
Indonesia, Lao PDR, Malaysia, Myanmar, Nepal, Russia, Thailand and Vietnam and
the Non Tiger Range countries willing to support and/or participate in Tiger
conservation programmes is United Kingdom. .

67. Recently, for the first time in our country, which of the following places got the
World Heritage City tag?
(a) Allahabad (b) Kolkata
(c) Hyderabad (d) Ahmadabad

Answer (d)
Explanation
 Ahmedabad in Gujarat was formally accorded the status of India's first World
Heritage City from the United Nations Educational, Scientific and Cultural
Organisation (UNESCO)
 UNESCO World Heritage city, a symbol of intercultural dialogue and unity in
diversity.
 There are 250 heritage cities in the world and Ahmedabad is one of them.
 Significance of the city lies beyond the physical beauty of its architectural
heritage. A landmark city where (Mahatama) Gandhiji began India's freedom
struggle
 Founded in the 15th century, the walled city of Ahmedabad, on the eastern bank of
the Sabarmati river, presents a rich architectural heritage.
 It has 28 Archaeological Survey of India's centrally protected monuments.
 India now has a total of 36 World Heritage Inscriptions -- 28 cultural, 7 natural and
1 mixed site.
 India is second after China in terms of number of world heritage properties in
ASPAC (Asia and Pacific) region, and overall seventh in the world.

68. Recently, for the first time in our country, which of the following cities has declared
the Ganges dolphin as City Animal?
(a) Allahabad (b) Kolkata
(c) Agra (d) Guwahati

Answer (d)
Explanation
 The Ministry of Environment and Forests notified the Ganges River Dolphin as the
National Aquatic Animal.
 Assam's Guwahati became the first city in the country to have its own city
animal with the district administration declaring the Gangetic river dolphin as the
mascot.
 Gangetic river dolphin is listed in Schedule I of the Wildlife (Protection) Act 1972.

69. What would happen if excess fertilisers are run off in to a lake ecosystem?
1. The excess in nutrients results in algal bloom
2. Biodiversity of the lake will increase
3. Decrease in the oxygen level of the lake and becomes dead zones
Which of the statements given above is/are correct?
(a) 1 only (b) 1 and 3 only (c) 2 and 3 only (d) 1, 2 and 3

Answer (b)
Explanation

 9446331522 9446331522 www.neoias.com Page 39


https://t.me/testseries4exams
Download All PDF form here :- https://t.me/pdf4exams
NEO IAS Most Important 150 Environmental Science Model Questions from TPS 2018

 The application of Nitrogen fertilizers to crops has caused increased rates of


denitrification and leaching of nitrate into groundwater.
 The additional Nitrogen entering the groundwater system eventually flows into
streams, rivers, lakes, and estuaries. In these systems, the added Nitrogen can
lead to eutrophication.
 Increased deposition of Nitrogen from atmospheric sources is also because of fossil
fuel combustion and forest burning. Both of these processes release variety forms
of Nitrogen through combustion.
 Livestock release a large amount of ammonia into the environment from their
wastes.
 These all Nitrogen enters the soil system and then the hydrologic system through
leaching, groundwater flow, and runoff.
 Eutrophication is the natural aging of a lake by nutrient enrichment of its water.
 Pollutants from man’s activities like effluents from the industries and homes can
radically accelerate the aging process. This phenomenon has been called Cultural or
Accelerated Eutrophication.
 During the past century, lakes in many parts of the earth have been severely
eutrophied by sewage and agricultural and industrial wastes. The prime
contaminants are nitrates and phosphates, which act as plant nutrients. They
overstimulate the growth of algae, causing unsightly scum and unpleasant odors,
and robbing the water of dissolved oxygen vital to other aquatic life. At the same
time, other pollutants flowing into a lake may poison whole populations of fish,
whose decomposing remains further deplete the water’s dissolved oxygen content.
In such fashion, a lake can literally choke to death.
 Excessive quantities of chemicals which get washed from the fields act as nutrients
for algae in the nearby water body to flourish (algal bloom - excessive growth of
planktonic (free-floating) algae).
 Algal blooms cause deterioration of the water quality and fish mortality. Once these
algae die, they serve as food for decomposers like bacteria. A lot of oxygen in the
water body gets used up. This results in a decrease in the oxygen level which may
kill aquatic organisms, and becomes dead zones. Some bloom-forming algae are
extremely toxic to human beings and animals.

70. One of the Ramsar site in India which is unique in being a swamp with floating
vegetation that supports a rich biodiversity, is deteriorating primarily because of the
change in water regime due to construction of a barrages and the commissioning of a
Hydro-Power Project in that region. Which of the following could be that Ramsar site?
(a) Bhitarkanika Mangroves
(b) Chilika Lake
(c) East Calcutta Wetlands
(d) Loktak Lake

Answer (d)
Explanation
About Loktak lake
 About two-third of the surface area of the lake is occupied by a floating Vegetation
which is locally called ‘phumdi’.

 9446331522 9446331522 www.neoias.com Page 40


https://t.me/testseries4exams
Download All PDF form here :- https://t.me/pdf4exams
NEO IAS Most Important 150 Environmental Science Model Questions from TPS 2018

 A mammal, sangai/dancing deers habitat is restricted to the marshy wetland of


Keibul Lamjao National Park in the Southeastern fringe of in Loktak Lake
(Ramsar Site), Manipur.
 The habitat in the Park is deteriorating primarily because of the change in water
regime due to construction of Itahi barrage.
 The phumdi, which used to settle during lean season and get replenished with soil
and nourishment, are now continuously floating resulting in their thinning.
Consequently, so they are unable to bear the weight of deer now.
 After the commissioning of the Loktak Hydro-Power Project in 1983, large
agricultural areas at the lake periphery have been submerged which have changed
the economic life of the people making them more dependent on the Park.

71. Which one of the following statements is/are not correct?


(a) India is not a signatory to the Convention on Biological Diversity
(b) The World Heritage Convention does not have a bearing on the biodiversity
(c) The Montreal Protocol was designed to reduce the production and consumption of
ozone depleting substances.
(d) Both (a) and (b)

Answer (d)
Explanation
 The Convention on Biological Diversity (CBD), to conserve biodiversity, is an
international treaty drawn at UN Conference on Environment and Development,
Rio de Janeiro, 1992. India is a signatory to the CBD.
 The World Heritage Convention have a bearing on the biodiversity. It have a category
called Natural World Heritage sites.
 Montreal Protocol
o The Montreal Protocol on Substances that Deplete the Ozone Layer was
designed to reduce the production and consumption of ozone depleting
substances in order to reduce their abundance in the atmosphere, and thereby
protect the earth’s fragile ozone Layer.

72. With reference to Environmental Impact Assessment (EIA), consider the following
statements:
1. EIA is a formal study process of identifying and evaluating the likely environmental
impacts of a proposed project or development.
2. EIA is given legal status under the Wildlife (Protection) Act, 1972
Which of the statements given above is/are correct?
(a) 1 only(b) 2 only (c) Both 1 and 2 (d) Neither 1 nor 2

Answer (a)
Explanation
 Environmental Impact Assessment (EIA) is a formal study process of identifying
and evaluating the likely environmental impacts (both beneficial and adverse) of a
proposed project or development, taking into account inter-related socio-
economic, cultural and human-health impacts.
 The Government of India enacted the Environment (Protection) Act on 1986. This
gave EIA a statutory status.

 9446331522 9446331522 www.neoias.com Page 41


https://t.me/testseries4exams
Download All PDF form here :- https://t.me/pdf4exams
NEO IAS Most Important 150 Environmental Science Model Questions from TPS 2018

73. Which one of the following Ramsar site has a natural waste water treatment system
developed by the local people?
(a) Harike Lake
(b) Chilika Lake
(c) Bhitarkanika
(d) East Calcutta Wetland

Answer (d)
Explanation
 Bengal's East Calcutta wetlands, through a system developed by local people over
the years, has saved Kolkata the costs of wastewater treatment plants.
 It forms an urban facility for treating the city's wastewater and using the treated
waste for pisciculture and agriculture, through the recovery of nutrients in an
efficient manner - the water flows through fish ponds covering about 4,000 ha, and
the ponds act as solar reactors and complete most of their bio-chemical reactions
with the help of solar energy.
 It is described as a rare example of environmental protection and development
management.
 These wetlands are well known over the world for their multiple uses.
 The wetland comprises of intertidal marshes including salt marshes, salt meadows
with significant waste water treatment areas like sewage farms, settling ponds and
oxidation basins.
 This wetland is under threat due to the developers, over fishing and industrial
waste.

74. With reference to ‘Eco- Sensitive Zones’ (ESZ), which of the following statements is
/are correct?
1. ESZs are the transition zones around the Protected areas that are declared under
the Environmental (Protection) Act, 1986.
2. ESZ are declared to prevent ecological damage caused due to developmental
activities around National Parks and Wildlife Sanctuaries
Select the correct answer using the code given below.
(a) 1 only (b) 2 only (c) Both 1 and 2 (d) Neither 1 nor 2

Answer (c)
Explanation
ECOSENSITIVE ZONE or ECO-FRAGILE ZONE
 Ecosensitive zones are the transition zones around the Protected areas and is
guidelined by the Ministry of Environment and Forests (MoEF) by the National
Wildlife Action Plan (2002-2016). It is declared as Eco sensitive zone under the
Environmental (Protection) Act, 1986.
 The land falling within 10km of the boundaries of Protected Areas are declared as
Eco sensitive zone/ Eco fragile zone, to prevent ecological damage caused due
to developmental activities around National Parks and Wildlife Sanctuaries by
acting as a shock absorber or transition zone.
 The guidelines for the Eco sensitive zone were issued in a notification letter dated
19 February, 2011.
 The methods to manage such zones are by classifying the activities in to
Prohibited, Regulated and Permitted.

 9446331522 9446331522 www.neoias.com Page 42


https://t.me/testseries4exams
Download All PDF form here :- https://t.me/pdf4exams
NEO IAS Most Important 150 Environmental Science Model Questions from TPS 2018

75. Which one of the following sites in India got the CA|TS approval?
(a) Periyar Tiger Reserve
(b) Ranthambhore Tiger Reserve
(c) Lansdowne Forest Division
(d) Sathyamangalam Forest Division

Answer (c)
Explanation
 CA|TS is a partnership between governments, NGOs and tiger conservation areas to
define and implement the conservation standards.
 CA|TS is a set of criteria which allows tiger sites to check if their management
will lead to successful tiger conservation.
 CA|TS was developed by tiger and protected area experts. Officially launched in
2013, CA|TS is an important part of Tx2, the global goal to double wild tiger
numbers by the year 2022.
 So far there are 14 registered sites in India, out of that only one site got approved by
CA|TS and it is Lansdowne Forest Division.

76. With reference to India Biodiversity Awards (IBA), which of the following statements
is/are correct?
1. It is a joint initiative of the Ministry of Environment, Forest and Climate Change
(MoEFCC), National Biodiversity Authority (NBA) and United Nations Development
programme (UNDP)
2. It recognise the contribution of stakeholders towards the conservation of biodiversity
and excellence in biodiversity governance
3. Awards to the steps taken for conservation of threatened species is the only award
under the initiative
Select the correct answer using the code given below.
(a) 1 only (b) 1 and 2 only (c) 2 and 3 only (d) 1, 2 and 3

Answer (b)
Explanation
 India Biodiversity Awards is a joint initiative of the Ministry of Environment,
Forest and Climate Change (MoEFCC), National Biodiversity Authority (NBA) and
United Nations Development programme (UNDP).
 It Recognise the contribution of stakeholders towards the conservation of
biodiversity and excellence in biodiversity governance.
 The first India Biodiversity Awards were jointly announced by the Ministry and
UNDP India in 2012 at the high level segment of the Eleventh meeting of Conference
of Parties (COPs) to the Convention on Biological Diversity, during India’s
Presidency of the COPs to the Convention.
 In 2016, the categories of the India Biodiversity Awards have been modified to link
them with the provisions of the Biological Diversity Act, 2002, in order to
incentivize stakeholders for the conservation of biodiversity and the effective
implementation of the Act.
The four categories of awards:
1. Conservation of threatened species
2. Sustainable use of biological resources
3. Successful mechanisms/models for access and benefit sharing
4. Biodiversity Management Committee (BMC)

 9446331522 9446331522 www.neoias.com Page 43


https://t.me/testseries4exams
Download All PDF form here :- https://t.me/pdf4exams
NEO IAS Most Important 150 Environmental Science Model Questions from TPS 2018

77. Apart from being known as the ‘orange city of India’, which among the following is
also known as the 'Tiger Capital or Tiger Gateway of India'?
(a) Jaipur
(b) Nagpur
(c) Jodhpur
(d) Ahmedabad

Answer (b)
Explanation
 Nagpur is known by many names – ‘The Orange City’, ‘The Winter Capital of
Maharashtra’ and the ‘Tiger Capital of India’.
 Jaipur is called the Pink City and Jodhpur Blue City due to the presence of
prominent pink and blue architechtures in these cities, respectively.

78. Which one of the following terms describes process by which organic material is
microbiologically converted under anaerobic conditions to biogas?
(a) Ammonification (b) Bioremediation
(c) Phytoremediation (d) Biomethanation

Answer (d)
Explanation
 Biomethanation is a process by which organic material is microbiologically
converted under anaerobic conditions to biogas.
 Phytoremediation refers to the technologies that use plants to clean up the soil,
air, and water which have been contaminated by chemicals.
 Bioremediation: "bio-remediate" means to use biological organisms to solve an
environmental problem such as contaminated soil or groundwater. Describes the
process of using microbes to treat areas of land or sea that have been contaminated
by pesticides, oil or solvents. Bioremediation works by providing the pollution-eating
organisms with fertilizer, oxygen, and other conditions that encourage their rapid
growth. In fact, bioremediation is often used to help clean up oil spills.
 Decomposition of organic nitrogen of dead plants and animals into ammonia is
called ‘Ammonification’.

79. Consider the following statements in respect of Foundation for Environmental


Education (FEE):
1. FEE is a Governmental organisation under UNESCO.
2. The certification of Blue Flag for beaches and marinas is issued by FEE
3. The mission of the Foundation for Environmental Education is to engage and
empower people through education in collaboration with their members and
partners worldwide
Which of the above statements is/are correct?
(a) 1 only (b) 2 and 3 only (c) 1 and 3 only (d) 1, 2 and 3

Answer (b)
Explanation
 FEE is a not-for-profit, non-governmental organisation.
 The Blue Flag is a certification by the Foundation for Environmental Education
(FEE) that a beach, marina or sustainable boating tourism operator meets its 30
stringent standards for cleanliness, upkeep and basic amenities (30 criteria
 9446331522 9446331522 www.neoias.com Page 44
https://t.me/testseries4exams
Download All PDF form here :- https://t.me/pdf4exams
NEO IAS Most Important 150 Environmental Science Model Questions from TPS 2018

related to water quality, environmental cleanliness and waste management, safety


and other services)
 Certificates, which FEE refers to as awards, are issued on an annual basis to
beaches and marinas of FEE member countries.
 The mission of the Foundation for Environmental Education is to engage and
empower people through education in collaboration with their members and
partners worldwide
 The vision of the Foundation for Environmental Education is a sustainable world in
which education creates positive change for all.
 FEE is recognised by UNESCO as a world-leader within the fields of Environmental
Education and Education for Sustainable Development.
 The Foundation for Environmental Education does not just focus on long-term
goals: their programmes help to make a difference every day. FEE has five programs:
Eco-Schools, Young Reporters for the Environment and Learning About Forests
educate, investigate and stimulate in equal measure, while Blue Flag champions the
coasts and Green Key works to improve the environmental performance of the
hospitality industry.
 So far, there is no beach in India that has been awarded Blue Flag certification.
 Selected beaches will be funded through the ongoing Integrated Coastal
Management Programme
 The Foundation for Environmental Education is partnered by some of the world's
foremost organisations in the fields of education and the environment. These
illustrious partners include UNESCO, the UNEP, UNWTO, ISESCO, YMP and Earth
Charter.

80. Consider the following pairs:


Protected Area River flowing through the Park
1. Umred Karhandla : Mahanadi
2. Satkosia : Ken
3. Panna : Wainganga
Which of the above pairs is/are correctly matched?
(a) 1 and 2 (b) 3 only (c) 1 and 3 (d) None

Answer (d)
Explanation
1. Umred Karhandla : Wainganga
2. Satkosia : Mahanadi
3. Panna : Ken

81. With reference to ‘Project Tiger’, consider the following statements:


1. It was launched in 1973 as a Centrally Sponsored Scheme of Government of India
2. First Tiger Reserve established in India is Jim Corbett National Park
3. The objective of the project is to preserve the habitats and tigers therein
4. Project Tiger is administrated by the National Tiger Conservation Authority
Which of the statements given above is/are correct?
(a) 1 and 2 only (b) 1, 3 and 4 only (c) 2, 3 and 4 only (d) 1, 2, 3 and 4

Answer (d)

 9446331522 9446331522 www.neoias.com Page 45


https://t.me/testseries4exams
Download All PDF form here :- https://t.me/pdf4exams
NEO IAS Most Important 150 Environmental Science Model Questions from TPS 2018

Explanation
 Project Tiger was launched by the Government of India in the year 1973 to save the
threatened species of tiger in the country.
 Project Tiger is a Centrally Sponsored Scheme of Government of India which was
launched on the 1st of April, 1973 for in-situ conservation of wild tigers in
designated tiger reserves. First Tiger Reserve established in India is Jim Corbett
National Park.
 There are 50 tiger reserves in India, which are governed by Project Tiger which is
administrated by the National Tiger Conservation Authority.
 The National Tiger Conservation Authority (NTCA) is a statutory body of the
Ministry, with an overarching supervisory / coordination role, performing functions
as provided in the Wildlife (Protection) Act, 1972.
 The role of Project Tiger / NTCA is to provide technical guidance and funding
support.
 The State Government on recommendation of the National Tiger Conservation
Authority notify an area as a Tiger reserve.
 The strategy adopted for tiger conservation in Project Tiger involves exclusive tiger
agenda in the core/critical tiger habitat, inclusive people-wildlife agenda in the outer
buffer, besides fostering the latter agenda in the corridors.
 This strategy / roadmap is reflected in a tiger reserve specific Tiger Conservation
Plan for each reserve prepared under section 38V of the Wildlife (Protection) Act,
1972.

82. Consider the following pairs:


Protected areas River flowing
1. Rajaji : Gomati
2. Dandeli Anshi : Kali Ganga
3. Pilihibit : Ganga
Which of the above pairs is/are correctly matched?
(a) 1 and 2 (b) 3 only (c) 1 and 3 (d) None

Answer (d)
Explanation
 Rajaji : Ganga
 Dandeli Anshi : Kali river
 Pilihibit : Gomati
 Kali Ganga (Sarda river – head stream of Ghaghara river) is a river in Uttarakhand.
 Through Dandeli Anshi (Kali tiger reserve) the Kali river (Kalinadi) is flowing.
 The river Gomati is originating from the Pilihibit.

83. With reference to ‘Global Environment Facility (GEF)’, which of the following
statements is/are correct?
1. GEF is an international partnership of countries, international institutions, civil
society organizations, and private sector to address global environmental issues.
2. The GEF was established as a pilot program in the World Bank to assist in the
protection of the global environment and to promote environmental sustainable
development.
3. GEF entrusted to be the financial mechanism for both the UN Convention on
Biological Diversity and the UN Framework Convention on Climate Change.
Select the correct answer using the code given below.
(a) 1 only (b) 2 and 3 only (c) 1 and 3 only (d) 1, 2 and 3

Answer (d)

 9446331522 9446331522 www.neoias.com Page 46


https://t.me/testseries4exams
Download All PDF form here :- https://t.me/pdf4exams
NEO IAS Most Important 150 Environmental Science Model Questions from TPS 2018

Explanation
 The Global Environment Facility was established in October 1991 as a $1 billion
pilot program in the World Bank to assist in the protection of the global
environment and to promote environmental sustainable development.
 The GEF has become an international partnership of 183 countries,
international institutions, civil society organizations, and private sector to
address global environmental issues.
 In 1994, at the Rio Earth Summit, the GEF was restructured and moved out of
the World Bank system to become a permanent, separate institution.
 As part of the restructuring, the GEF was entrusted to become the financial
mechanism for both the UN Convention on Biological Diversity and the UN
Framework Convention on Climate Change.
 The GEF subsequently was also selected to serve as financial mechanism for
three more international conventions: The Stockholm Convention on
Persistent Organic Pollutants (2001), the United Nations Convention to
Combat Desertification (2003) and the Minamata Convention on Mercury
(2013).
 The United Nations Development Programme, the United Nations
Environment Program, and the World Bank were the three initial partners
implementing GEF projects.
 The GEF would provide new and additional grants and concessional funding to
cover the incremental or additional costs associated with transforming a project
with national benefits into one with global environmental benefits.

84. Out of all the eighteen Biosphere Reserves in India, ten have been included in the
World Network of Biosphere Reserves by UNESCO. Which of the following is not one of
them?
(a) Panna (b) Gulf of Mannar
(c) Simlipal (d) Panchmarhi

Answer (a)
Explanation
 Biosphere Reserve (BR) is representative parts of natural and cultural landscapes
extending over large area of terrestrial or coastal/marine ecosystems or a
combination thereof, where, through appropriate zoning patterns and
management mechanisms, the conservation of ecosystems and their
biodiversity is ensured.
 The Indian government has selected 18 Biosphere Reserves across the country; ten
of the eighteen are a part of the World Network of Biosphere Reserves. Nilgiri,
Nanda Devi, Nokrek, Manas, Sunderbans, Gulf of Mannar, Great Nicobar, Similipal,
Dibru-Saikhova, Dehang-Dibang, Pachmarhi, Khangchendzonga, Agasthyamalai,
Achanakmar- Amarkantak, Rann of Kachchh, Cold Desert, Seshachalam and
Panna.
 10 BR’s those included in the World Network are; Nilgiri, Agasthyamalai, Gulf of
Mannar, Great Nicobar, Nanda Devi, Nokrek, Pachmarhi, Achanakmar-
Amarkantak, Similipal and Sunderbans.

85. Which one of the following elements is essential for the green plants for chlorophyll
synthesis?
(a) Calcium
(b) Iron
(c) Magnesium
(d) Potassium
 9446331522 9446331522 www.neoias.com Page 47
https://t.me/testseries4exams
Download All PDF form here :- https://t.me/pdf4exams
NEO IAS Most Important 150 Environmental Science Model Questions from TPS 2018

Answer (c)
Explanation
 Chlorophyll is a green photosynthetic pigment that absorbs sunlight and uses its
energy to synthesise carbohydrates from CO2 and water. In the structure of
chlorophyll, the central atom is magnesium.
 Chlorophyll consists of a porphyrin head and a phytol tail.
 Porphyrin head is made up of 4 pyrrole rings.
 A plant requires magnesium for Chlorophyll synthesis.
 It is essential for the formation of chlorophyll in green plants.
 Mg is situated at the centre of porphyrin head.
 All four rings bind with metal atom magnesium (Mg++), which remain present at the
center of chlorophyll molecule.

Chlorophyll formula = C55H72O5N4Mg

86. With reference to ‘Paryatan Parv’, sometimes seen in news, consider the following
statements
1. It is an initiative by Ministry of Tourism
2. It is a programme organised with the aim of drawing attention on the benefits of
tourism, showcasing the cultural diversity of the country and reinforcing the
principle of “Tourism for All”.
Which of the statements given above is/are correct?
(a) 1 only (b) 2 only
(c) Both 1 and 2 (d) Neither 1 nor 2

Answer (c)
Explanation
 The Ministry of Tourism kick-started ‘Paryatan Parv’ a 20-day programme
organised with the aim of drawing attention on the benefits of tourism,
showcasing the cultural diversity of the country and reinforcing the principle of
“Tourism for All”.
 ‘Paryatan Parv’ was inaugurated by Minister of State for culture at the Humayun’s
Tomb in New Delhi.
 The programme is being organised by the Ministry of Tourism, in collaboration
with other Central Ministries, State Governments and stakeholders.
 The programme focus on encouraging Indians to explore their own country.

 9446331522 9446331522 www.neoias.com Page 48


https://t.me/testseries4exams
Download All PDF form here :- https://t.me/pdf4exams
NEO IAS Most Important 150 Environmental Science Model Questions from TPS 2018

 Tourism events such as interactive sessions and workshops on skill


development and innovation in the sector is also conducted across all states

87. Consider the following protected areas:


1. Palamau 2. Amrabad
3. Orang 4. Mukandra Hills
Which of the above are declared Tiger Reserves?
(a) 1 and 2 only (b) 1, 3 and 4 only (c) 2, 3 and 4 only (d) 1, 2, 3 and 4

Answer (d)
Explanation
The following table shows the tiger reserves in India.

Sl no Name Sl no Name
Andhra Pradesh Maharashtra
1 Nagarjunsagar 27 Sahyadri
Srisailam (part)*
Arunachal 28 Melghat
Pradesh
2 Namdapha 29 Pench
3 Pakke 30 Nawegaon-
Nagzira
4 Kamlang Tiger 31 Bor
Reserve
Assam 32 Tadoba-Andhari
5 Manas Mizoram
6 Nameri 33 Dampa
7 Kaziranga Odisha
8 Orang Tiger 34 Similipal
Reserve
Bihar 35 Satkosia
9 Valmiki Rajasthan
Chattisgarh 36 Ranthambore
10 Indravati 37 Sariska
11 Achanakmar 38 Mukandra Hills
12 Udanti-Sitanadi Tamil Nadu
Jharkhand 39 Kalakad-
Mundanthurai
13 Palamau 40 Anamalai
Karnataka 41 Mudumalai
14 Bandipur 42 Sathyamangalam
15 Nagarahole Telangana
16 Dandeli-Anshi 43 Kawal
17 Biligiri 44 Amrabad
Ranganatha
Temple
18 Bhadra Uttar Pradesh
Kerala 45 Dudhwa
19 Parambikulam 46 Pilibhit

 9446331522 9446331522 www.neoias.com Page 49


https://t.me/testseries4exams
Download All PDF form here :- https://t.me/pdf4exams
NEO IAS Most Important 150 Environmental Science Model Questions from TPS 2018

20 Periyar Uttarakhand
Madhya Pradesh 47 Jim Corbett
21 Kanha 48 Rajaji
22 Pench West Bengal
23 Bandhavgarh 49 Sunderbans
24 Panna 50 Buxa
25 Satpura
26 Sanjay-Dubri

88. With reference to the term Allelopathy, consider the following statements.
1. Allelopathy refers to the chemical inhibition of one species by another
2. Allelopathic plants prevent other plants from using the available resources
3. Allelopathy is a biological phenomenon seen in plants and not in any other
organisms
Which of the statements given above is/ are correct?
(a) 1 and 2 only (b) 2 only (c) 3 only (d) 1, 2 and 3

Answer (a)
Explanation
 Allelopathy is a common biological phenomenon by which one organism produces
biochemicals that influence the growth, survival, development, and
reproduction of other organisms.
 These biochemicals are known as allelochemicals and have beneficial or
detrimental effects on target organisms.
 Allelopathy refers to the chemical inhibition of one species by another.
 Allelopathy is a form of chemical competition.
 Plant allelopathy is one of the modes of interaction between receptor and donor
plants and may exert either positive effects (e.g., for agricultural management, such
as weed control, crop protection, or crop re-establishment) or negative effects (e.g.,
autotoxicity, soil sickness, or biological invasion).
 To ensure sustainable agricultural development, it is important to exploit cultivation
systems that take advantage of the stimulatory/inhibitory influence of allelopathic
plants to regulate plant growth.
 Allelochemicals can potentially be used as growth regulators, herbicides,
insecticides, and antimicrobial crop protection products.
 Plant allelopathy is the ‘chemical warfare’ among the plants imposed by one plant
on another to suppress the latter and take advantage from that suppression.
 In the phenomenon of plant allelopathy, allelopathic plants create adverse
conditions to other neighboring plants by reducing their seed germination and
seedling growth. The "inhibitory" chemical is released into the environment where it
affects the development and growth of neighboring plants. The allelopathic plants
are very effective in weed killing and known as Nature’s Weed Killers.
 Allelopathic plants prevent other plants from using the available resources and
thus influence the evolution and distribution of other species.
 One of the most famous allelopathic plants is Black Walnut (Juglans nigra).

 9446331522 9446331522 www.neoias.com Page 50


https://t.me/testseries4exams
Download All PDF form here :- https://t.me/pdf4exams
NEO IAS Most Important 150 Environmental Science Model Questions from TPS 2018

 The definition of allelopathy was first used by Molish in 1937 to indicate all of the
effects that directly and indirectly result from biochemical substances transferred
from one plant to another.
 The term was refined by Rice (1984) to define “any direct or indirect harmful or
beneficial effect by one plant (including microorganisms) on another through
production of chemical compounds that escape into the environment” (Rice, 1984).
 In 1996, the International Allelopathy Society broadened its definition of allelopathy
to refer to any process involving secondary metabolites produced by plants,
microorganisms, viruses and fungi that influence the growth and development of
agricultural and biological systems. In addition, the allelopathic donor and receiver
should include animals.

89. Which one of the following is not a World Heritage Site?


(a) Western Ghats (b) Keoladeo National Park
(c) Sariska National Park (d) Nanda Devi and Valley of Flowers National Parks

Answer (c)
Explanation
 A UNESCO World Heritage Site is a place of importance that is listed by the
United Nations Educational, Scientific and Cultural Organization as of special
cultural or physical significance.
 UNESCO to encourage the identification, protection and preservation of cultural,
and natural heritage around the world considered to be of outstanding value to
humanity.
 This is embodied in an international treaty called the World Heritage Convention
concerning the Protection of the World Cultural and Natural Heritage, adopted by
UNESCO in 1972.
 There are natural and cultural sites. For Natural WHS, the site must be of
outstanding universal value, the site should be of outstanding example ongoing
ecological process, it should contain outstanding unique natural phenomena
and beauty, contain exceptional biodiversity.
 For the Cultural WHS it should represent a masterpiece of human creative,
monumental arts, outstanding architecture, traditional settlement their
cultural tradition or civilisation, etc

NATURAL WORLD HERITAGE SITES IN INDIA


Sl. Name of WH Site State Year of Area
No. Location Notification (sq.km)
1 Great Himalayan National Park Himachal Pradesh 2014 905.4
Conservation Area
2 Western Ghats Maharashtra, 2012 7,953.15
Goa,
Karnataka,
Tamil Nadu and
Kerala
3 Nanda Devi and Uttarakhand 1988 630.00
Valley of Flowers National Parks 87.50
4 Sundarbans National Park West Bengal 1987 1,330.10
5 Kaziranga National Park Assam 1985 429.96
6 Keoladeo National Park Rajasthan 1985 28.73
7 Manas Wildlife Sanctuary Assam 1985 391.00

 9446331522 9446331522 www.neoias.com Page 51


https://t.me/testseries4exams
Download All PDF form here :- https://t.me/pdf4exams
NEO IAS Most Important 150 Environmental Science Model Questions from TPS 2018

90. With reference to ‘Irrawady Dolphin’, which of the following statements is/are
correct?
1. It is an aquatic mammal found in estuaries and mangrove areas, with freshwater
populations occurring in river systems.
2. It is not found naturally in India
3. It is given legal protection under Schedule I of the Wildlife (Protection) Act, 1972.
Select the correct answer using the code given below.
(a) 1 and 2 (b) 2 only (c) 1 and 3 (d) 3 only

Answer (c)
Explanation
 Irrawaddy dolphins are aquatic mammals
 The species eats fish and crustaceans.
 Irrawaddy dolphins are distributed in shallow, near-shore tropical and subtropical
marine waters.
 They are primarily found in estuaries and semi-enclosed water bodies such as bays
and sounds, usually close to mangrove forests. Freshwater populations occur in
river systems.
 Freshwater subpopulations occur in the river Mahakam of Kalimantan (Indonesian
Borneo), the Ayeyarwady (formerly Irrawaddy) of Myanmar (formerly Burma), and
the Mekong Delta of Laos, Cambodia, and Vietnam. They are also reported in
isolated brackish (saltwater and freshwater) water bodies, such as Chilka Lake in
India and Songkhla Lake in Thailand.
 The species is protected under schedule 1 of Indian Wildlife (protection) act
1972.

91. With reference to “star rating for cities”, sometimes seen in news, consider the
following statements.
1. It is a star-rating initiative to rate cities on a 7-star rating system based on multiple
cleanliness indicators for solid waste management.
2. Cities will need to get recertified themselves every year to retain their star-status.
Which of the statements given above is/are correct?
(a) 1 only (b) 2 only
(c) Both 1 and 2 (d) Neither 1 nor 2

Answer (c)
Explanation
STAR RATING FOR CITIES
 It is a star-rating initiative to rate cities on a 7-star rating system based on multiple
cleanliness indicators for solid waste management.
 It is developed by the Swachh Bharat Mission –Urban
 Solid Waste management of each cities will be evaluated in terms of Door to Door
Collection, bulk generator compliance, source segregation, sweeping, scientific
processing of waste, scientific land filling, plastic waste management,
construction and demolition management, dump remediation & citizen grievance
redressal system etc.

How cities are rated?


 Cities can be rated as 1, 2, 3, 4, 5 and 7 star based on their compliance
with the protocol conditions specified for each of the rating.
 9446331522 9446331522 www.neoias.com Page 52
https://t.me/testseries4exams
Download All PDF form here :- https://t.me/pdf4exams
NEO IAS Most Important 150 Environmental Science Model Questions from TPS 2018

 Further, city should be ODF (Open Defecation Free) before it could be given rating of
3 star or above.
 While cities may self-declare themselves as 1- star, 2- star or 4 - star, MoHUA will
carry out an additional verification through an independent third party to certify
cities as 3-star, 5-star or 7-star.
 Cities will need to get recertified themselves every year to retain their star-status.

92. The ‘Operation Save Kurma’, sometimes seen in the news, in the context of
(a) Military attack
(b) Illegal trade of live turtles
(c) Money Laundering
(d) Abating pollution in River Ganga

Answer (b)
Explanation
 India has been awarded a certificate of commendation for its effort to combat
illegal wildlife trade by the Convention on International Trade in Endangered
Species of Wild Fauna and Flora (CITES).
 The award was given to the Wildlife Crime Control Bureau (WCCB) under the
Ministry for its efforts in conducting and coordinating a species-specific wildlife
enforcement operation, codenamed ‘Operation Save Kurma’.
 The operation Save Kurma was conducted by the WCCB to combat the
proliferating illegal trade of live turtles and its parts from the country to
destinations abroad.

93. With reference to Phytoplankton consider the following statements:


1. It has a great role in making ocean the largest carbon sink.
2. They are the first trophic level component of aquatic food chain.
3. Some of them uses bioluminescence phenomenon for communication when they
have stresses in the water body.
Which of the statements given above is/are correct?
(a) 1 only (b) 1 and 2 only (c) 2 and 3 only (d) 1, 2 and 3

Answer (d)
Explanation
 Phytoplankton is the plant like organisms of the water that carry out photosynthesis
and float in the upper areas of the world’s ocean.
 They are the producers present in the aquatic ecosystem.
 Phytoplankton, also known as microalgae, are similar to terrestrial plants in that
they contain chlorophyll to capture sunlight, and they use photosynthesis to turn it
into chemical energy in order to live and grow.
 Phytoplankton is responsible for most of the transfer of carbon in the Carbon dioxide
from the atmosphere to the ocean. Worldwide, this “biological carbon pump”
transfers about 10 gigatonnes of carbon from the atmosphere to the deep ocean each
year.
 Certain creatures both on land and sea can produce light through chemical
reactions taking place within their bodies known as Bioluminescence.
Bioluminescent phytoplankton occur in all the world’s oceans. The most common of
these are Dinoflagellates which are tiny unicellular marine plankton also known as
fire plants. Bioluminescence is used to evade predators and acts as a defense

 9446331522 9446331522 www.neoias.com Page 53


https://t.me/testseries4exams
Download All PDF form here :- https://t.me/pdf4exams
NEO IAS Most Important 150 Environmental Science Model Questions from TPS 2018

mechanism in dinoflagellates. Dinoflagelletes produce light when disturbed and will


give a light flash lasting a fraction of a second. The flash is meant to attract a
predator to the creature disturbing or trying to consume the dinoflagellate. The light
flash also surprises the predator causing it to worry about other predators attacking
it, making the predator less likely to prey on the dinoflagellate.

94. Consider the following pairs:


Wetlands State
1. Chilika Lake : Orissa
2. Keoladeo Ghana : Manipur
National Park
3. Kolleru Lake : Andhra
Pradesh
4. Wular Lake : Punjab
Which of the above pairs is/are correctly matched?
(a) 1, 2 and 3 only (b) 1 and 3 only (c) 2 and 4 only (d) 1, 2, 3 and 4

Answer (b)
Explanation
 Chilika Lake : Orissa
 Keoladeo Ghana National Park: Rajasthan
 Kolleru Lake : Andhra Pradesh
 Wular Lake : Jammu & Kashmir

95. Consider the following pairs.


Threatened animals hunted for
1. Chiru : shahtoosh
2. Musk deer : musk glands
3. Pangolins : scales
4. Helmeted hornbill : casque
Which of the above pairs are correctly matched?
(a) 1, 2 and 3 only (b) 1, 3 and 4 only (c) 2 and 4 only (d) 1, 2, 3 and 4

Answer (d)
Explanation
 Chiru / Tibetan Antelope inhabit high altitude plains, undulating hills plateaux
and montane valleys at elevations of 3,700-5,500 m. Chiru is adapted to live in
steppes and semi-desert areas of cold high mountains. They are found in Tibet cold
desert. Chiru have long been hunted for their underfur (shahtoosh), which is
renowned for its quality and which has traditionally been transported to Srinagar in
Kashmir, where it is woven into an extremely fine fabric used to make shawls. It is
also hunted for meat, magnificent horns.
 Musk deer lack antlers, but they possess a pair of enlarged canines that grow
continuously. It is found in Kashmir, Kumaon and Sikkim. This species inhabits
high alpine environments. Besides hunting for meat, it is hunted primarily for
trade of musk glands. The musk produced by the male deer is highly valued for its
cosmetic and alleged pharmaceutical properties.
 Pangolins are threatened by hunting (for their meat and scales) and heavy
deforestation of their natural habitats, and are the most trafficked mammals in the
world.

 9446331522 9446331522 www.neoias.com Page 54


https://t.me/testseries4exams
Download All PDF form here :- https://t.me/pdf4exams
NEO IAS Most Important 150 Environmental Science Model Questions from TPS 2018

 Helmeted hornbill, the striking bird with monochrome plumage and a formidable
“beak”, the helmeted hornbill is being hunted to extinction, one of the latest victims
of a thriving global trade in exotic wildlife. For decades poachers in Borneo’s western
forests focused on capturing orangutans and sun bears, but in the past few years a
surge in demand for hornbill “ivory” has pushed the avian species to the brink.
Poachers aren’t interested in their brilliant plumage or large bills, but a helmet-like
block of reddish-gold keratin at the front of the skulls known as a casque. It’s this
soft, ivory-like substance that’s carved by craftsmen in China into luxury
ornaments, statues and jewellery - trendy top-shelf trinkets that have soared in
value as so-called “red ivory” has grown more prestigious.

96. With reference to ‘The World Organisation for Animal Health (OIE)’, which of the
following statements is/are correct?
1. The OIE is the intergovernmental organisation responsible for improving animal
health worldwide.
2. It safeguards world trade by publishing health standards for international trade in
animals and animal products
3. Since India has a separate legislation for animal welfare, so far India does not took
the membership of OIE.
Select the correct answer using the code given below.
(a) 1 and 2 (b) 2 only (c) 1 and 3 (d) 3 only

Answer (a)
Explanation
The World Organisation for Animal Health (OIE)
 The need to fight animal diseases at global level led to the creation of the Office
International des Epizooties through the international Agreement signed on January
25th 1924. In May 2003 the Office became the World Organisation for Animal Health
but kept its historical acronym OIE.
 The OIE is the intergovernmental organisation responsible for improving animal
health worldwide.
 It is recognised as a reference organisation by the World Trade Organization (WTO)
and in 2017 has a total of 181 Member Countries. The OIE maintains permanent
relations with 71 other international and regional organisations.
 The organisation is placed under the authority and control of a World Assembly of
Delegates consisting of Delegates designated by the Governments of all Member
Countries.
 The Headquarters is situated in Paris.
 The OIE's financial resources are derived principally from compulsory annual
contributions backed up by voluntary contributions from Member Countries.
 India is a member

Activities of (OIE)
 Disseminates the information on animal disease outbreaks to other countries,
which can take the necessary preventive action. It uses Dissemination is via e-
mail, Disease Information and the World Animal Health Information System
Interface (WAHIS Interface).

 9446331522 9446331522 www.neoias.com Page 55


https://t.me/testseries4exams
Download All PDF form here :- https://t.me/pdf4exams
NEO IAS Most Important 150 Environmental Science Model Questions from TPS 2018

 Provides technical support to Member Countries requesting assistance with


animal disease control and eradication operations.
 Safeguards world trade by publishing health standards for international trade in
animals and animal products
 Help countries improve the legal framework and resources of national
Veterinary Services enabling them to benefit more fully from the WTO Sanitary
and Phytosanitary Agreement (SPS Agreement)

97. What is/are unique about ‘Khur’, a breed found in India?


1. It is a mammal which can race at nearly 60 kmph.
2. It is native to the Little Rann of Kutch.
3. The species has come out from the brink of extinction, mainly due to the nomadic
tribes and villagers of the Rann of Kutch area.
4. The increase in the Khur population in recent times can be attributed to improved
agricultural practices as well as water availability due to the Sardar Sarovar Canal
system.
Which of the above statements given above is/are correct?
(a) 1 only (b) 2 and 3 only (c) 1, 2 and 4 only (d) 1, 2, 3 and 4

Answer (d)
Explanation
 The Indian wild ass is a subspecies of the onager, native to Southern Asia. As of
2016, it is listed as Near Threatened by IUCN.
 It can race at nearly 60 kmph.
 Native to the Little Rann of Kutch, the ass is locally known as ‘khur’. During
the early ’60s and ’70s it was on the brink of extinction, but has since
rebounded, and the head count currently stands at 4,000.
 They are usually seen in small herds, especially during the breeding season.
 The success story of the Wild Ass is attributed not only to the concerted efforts of
the Central and State governments but also mainly to the nomadic tribes and
gutsy villagers of the area.
 Small and medium farmers growing groundnut, bajra, cotton, wheat, and rice do
not have any skirmishes with the wandering Wild Ass even if it feeds on their
farm produce.
 Rann is the only place on earth where the endangered Indian Wild Ass continues
to endure despite all odds.
 The increase in the khur population in recent times can be attributed to
improved agricultural practices as well as water availability due to the
Sardar Sarovar Canal system. The change in cropping patterns, with more cash
crops, means that the species receives better forage and water. This reduces its
search for food and thereby helps it conserve its energy in an arid landscape

98. With reference to coral reefs, consider the following statements:


1. Fringing reefs are reefs that grow directly from a shore.
2. Barrier reefs are extensive linear reef complexes that parallel a shore, and are
separated from it by lagoon.
3. Atolls are usually circular or oval in shape, with an open lagoon in the center
Which of the statements given above is/are correct?
(a) 1 and 2 only (b) 3 only (c) 1, 2 and 3 (d) None

 9446331522 9446331522 www.neoias.com Page 56


https://t.me/testseries4exams
Download All PDF form here :- https://t.me/pdf4exams
NEO IAS Most Important 150 Environmental Science Model Questions from TPS 2018

Answer (c)
Explanation
 Coral reefs are important because they bring in billions of dollars to our economy
through tourism, protect coastal homes from storms, support promising medical
treatments, and provide a home for millions of aquatic species.
 Scientists generally divide coral reefs into four classes: fringing reefs, barrier reefs,
atolls, and patch reefs.
 Fringing reefs are reefs that grow directly from a shore. The most common type of
reef is the fringing reef. This type of reef grows seaward directly from the shore. They
form borders along the shoreline and surrounding islands.
 Barrier reefs are extensive linear reef complexes that parallel a shore, and are
separated from it by lagoon. Barrier reefs are similar to fringing reefs in that they
also border a shoreline; however, instead of growing directly out from the shore, they
are separated from land by an expanse of water. This creates a lagoon of open, often
deep water between the reef and the shore.
 An atoll is a roughly circular (annular) oceanic reef system surrounding a large (and
often deep) central lagoon. When a fringing reef continues to grow upward from a
volcanic island that has sunk entirely below sea level, an atoll is formed. Atolls are
usually circular or oval in shape, with an open lagoon in the center.

99. Consider the following statements:


(a) Loktak Lake is a fresh water lake which has Phumdis floating over it.
(b) Keibul Lamjao National Park located on the Loktak Lake is a natural refuge of the
Sangai (dancing deer, Eld’s deer).
Which of the statements given above is/are correct?
(a) 1 only (b) 2 only (c) Both 1 and 2 (d) Neither 1 nor 2

Answer (c)
Explanation
 Loktak Lake is the largest fresh water lake in Northeast India.
 This Lake is known as the only floating lake in the world due to the floating
phumdis (heterogeneous mass of vegetation, soil, and organic matters at various
stages of decomposition) on it.
 Keibul Lamjao NP in Manipur is the only floating park in the world, and is situated
on the south bank of loktak lake.
 The Brow-antlered deer, which was first discovered in Manipur, in the Keibul
Lamjao Park area, which necessitated declaring this reserve park area as a national
park to protect and conserve the deer.
 The species is now confined to a single small population at the southern end of
Loktak Lake in Manipur, India.

100. With reference to ‘Olive Ridley Turtle, a reptile found in India, which of the following
statements is/are correct?
1. They are the smallest and most abundant of all sea turtles found in the world.
2. They live in warm waters of Pacific, Atlantic and Indian Oceans.
3. The Coromandel Coast in India is the largest mass nesting site for the olive ridley
turtles
Select the correct answer using the code given below.
(a) 1 and 2 only (b) 2 and 3 only (c) 1 and 3 only (d) 1, 2 and 3

 9446331522 9446331522 www.neoias.com Page 57


https://t.me/testseries4exams
Download All PDF form here :- https://t.me/pdf4exams
NEO IAS Most Important 150 Environmental Science Model Questions from TPS 2018

Answer (a)
Explanation
 The Olive ridley turtles are the smallest and most abundant of all sea turtles
found in the world, inhabiting warm waters of the Pacific, Atlantic and Indian
oceans.
 These turtles, along with their cousin the Kemps ridley turtle, are best known for
their unique mass nesting called Arribada, where thousands of females come
together on the same beach to lay eggs.
 They are carnivores, and feed mainly on jellyfish, shrimp, snails, crabs, molluscs
and a variety of fish and their eggs.
 These turtles spend their entire lives in the ocean, and migrate thousands of
kilometers between feeding and mating grounds in the course of a year.
 The coast of Orissa in India is the largest mass nesting site for the Olive-ridley,
followed by the coasts of Mexico and Costa Rica.

101. With reference to Environmental Performance Index (EPI), consider the following
statements.
1. It is a method of quantifying and numerically marking the environmental
performance of a state's policies.
2. It is developed by the organisation Global Footprint Network.
Which of the statements given above is/are correct?
(a) 1 only (b) 2 only (c) Both 1 and 2 (d) Neither 1 nor 2

Answer (a)
Explanation
Environmental Performance Index (EPI)
 The Environmental Performance Index (EPI) is a method of quantifying and
numerically marking the environmental performance of a state's policies.
 This index was developed from the Pilot Environmental Performance Index, first
published in 2002, and designed to supplement the environmental targets
set forth in the United Nations Millennium Development Goals.
 EPI is developed by Yale University (Yale Center for Environmental Law and Policy)
and Columbia University (Center for International Earth Science Information
Network) in collaboration with the World Economic Forum and the Joint
Research Centre of the European Commission.
 High rank in the EPI shows long-standing commitments to protecting public health,
preserving natural resources and decoupling greenhouse gas (GHG) emissions from
economic activity.
 India has been ranked 177 among 180 countries in the Environmental Performance
Index (EPI) –2018

Global Footprint Network


 Global Footprint Network is a research organization that is changing how the
world manages its natural resources and responds to climate change. Global
Footprint Network calculate the Ecological Footprint and Earth overshoot day.

 9446331522 9446331522 www.neoias.com Page 58


https://t.me/testseries4exams
Download All PDF form here :- https://t.me/pdf4exams
NEO IAS Most Important 150 Environmental Science Model Questions from TPS 2018

102. With reference to insects which comes under the arthropoda, consider the following
statements.
1. The fall in population of insects has a role in the agricultural production
2. Water striders are a group of insects adapted to life on the surface of water, using
surface tension to their advantage.
Which of the statements given above is/are correct?
(a) 1 only (b) 2 only (c) Both 1 and 2 (d) Neither 1 nor 2

Answer (c)
Explanation
 Scientists from the Zoological Survey of India (ZSI) have discovered a new species of
water strider from Nagaland. The species, named Ptilomera nagalanda Jehamalar
and Chandra, was found in the river Intanki, Peren district.
 Water striders are a group of insects adapted to life on the surface of water, using
surface tension to their advantage.
 Scientists working on the water striders say that their presence serves as an
indicator of water quality and they are found on water surface.
 So far, only five species of water striders under the subgenus Ptilomera were known
in India.
 What is unique about Ptilomera is that they are only found in rocky, fast flowing
streams and rivers that are not exposed to a lot of sunlight.
 Ptilomera has hair on the middle legs that help the insects resist the strong current
of streams.
 Water striders have three pairs of legs.

103. With reference to ‘ecological niche’, which of the following statements is /are
correct?
1. Ecological niche is the physical position and functional role of a species within the
ecosystem.
2. An ecological niche describes how an organism or population responds to the
distribution of resources and competitors.
Select the correct answer using the code given below.
(a) 1 only (b) 2 only (c) Both 1 and 2 (d) Neither 1 nor 2

Answer (c)
Explanation
 An ecological niche is the role and position a species has in its environment
(ecosystem); how it meets its needs for food and shelter, how it survives, and how it
reproduces.
 Ecological niche is the physical space or position and functional role of a
species within the community or ecosystem.
 A species niche includes all of its interactions with the biotic and abiotic factors
of its environment.
 A niche is unique for a species while many species share the habitat.
 No two species in a habitat can have the same niche. This is because of the
competition with one another until one is displaced.
 The description of a niche may include descriptions of the organism's life history,
habitat, and place in the food chain.
 More formally, the niche includes how a population responds to the abundance of
its resources and enemies.
 9446331522 9446331522 www.neoias.com Page 59
https://t.me/testseries4exams
Download All PDF form here :- https://t.me/pdf4exams
NEO IAS Most Important 150 Environmental Science Model Questions from TPS 2018

 The abiotic or physical environment is also part of the niche because it influences
how populations affect, and are affected by, resources and enemies.

104. Consider the following pairs:


1. Dandeli Anshi : Karnataka
(Kali) Tiger
reserve
2. Nagarjunsagar : Tamil Nadu
Srisailam Tiger
Reserve
3. Satyamangalam : Andhra
Tiger Reserve Pradesh
Which of the above pairs is/are correctly matched?
(a) 1 only (b) 2 and 3 only (c) 1 and 3 only (d) 1, 2 and 3

Answer (a)
Explanation
 Dandeli Anshi (Kali) is a Tiger reserve in Karnataka
 Nagarjunsagar Srisailam is a Tiger Reserve in Andhra Pradesh
 Satyamangalam is a Tiger Reserve in Tamil Nadu

105. Recently, for the first time in our country, in which of the following states is an
insect museum established?
(a) Tamil Nadu (b) Kerala
(c) Karnataka (d) Telengana

Answer (a)
Explanation
 India’s first insect museum with state-of-the-art amenities was unveiled at Tamil
Nadu Agricultural University campus in Coimbatore, Tamil Nadu.
 It was inaugurated by Tamil Nadu Chief Minister.
 Under the theme 'Bugs Are Kings', the museum displays insects as preserved
specimens and models on their behaviour, habits and habitat.
 The museum will help farmers to gain more knowledge about insects.
 The museum displays insects as preserved specimens, live specimens depicting their
life cycle, images, videos and models on their behaviour, habits and habitat under
the theme 'Bugs Are Kings'.

106. Which one of the following is the animal that has been named Person of the Year by
the world's largest animal rights groups, PETA.
(a) A Lion tailed macaque
(b) A Nilgiri langur
(c) A Golden langur
(d) An Indonesian monkey

Answer (d)
Explanation
 PETA awards its annual “Person of the Year” distinction to an individual whose
significant contributions have helped propel the animal rights movement forward.

 9446331522 9446331522 www.neoias.com Page 60


https://t.me/testseries4exams
Download All PDF form here :- https://t.me/pdf4exams
NEO IAS Most Important 150 Environmental Science Model Questions from TPS 2018

 An Indonesian monkey (crested black macaque) that triggered a landmark


copyright case when it took a photograph of itself has been named Person of the
Year by one of the world's largest animal rights groups.
 The Indonesian monkey who shot to fame after it snapped a grinning selfie — and
sparked a landmark U.S. copyright case — was named “Person of the Year” by the
animal rights group, PETA. The People for the Ethical Treatment of Animals (PETA)
said it was honouring Naruto, a crested black macaque with a goofy-looking grin, to
recognise that “he is someone, not something”.
 In 2011, the monkey pressed the shutter button while staring down the lens of a
camera set up British nature photographer David Slater on the island of Sulawesi.
 The photos quickly went viral and PETA launched a lawsuit that claimed the then
six-year-old Naruto should be “declared the author and owner of his photograph”.
 Naruto’s historic selfie challenged the idea of who is a person and who is not and
resulted in the first-ever lawsuit seeking to declare a nonhuman animal the
owner of property, rather than being declared property himself.

107. Consider the following with reference to Gangetic Dolphin, the National Aquatic
animal of India:
1. The developmental project in Ganga River for shipping, especially due to the National
Waterway I Project.
2. Depletion of prey base.
3. Accidental mortality in fishing nets.
4. Accidents with vessel propellers.
Which of the above are likely causes for reduction in numbers of the animal?
(a) 1 and 2 only (b) 2 only (c) 1 and 3 only (d) 1, 2, 3 and 4

Answer (d)
Explanation
 The Ganges River dolphin or Susu, lives in one of the most densely populated
regions, Ganges river.
 One of the main threats to the species is loss of habitat due in large part to the
creation of dams and irrigation projects.
 The species is facing threat due to the pollution of river, fragmentation of habitat,
hunted for their meat.
 National waterways 1 project also threatens Gangetic dolphins.
 The development of the Ganga for shipping is seen by wildlife conservationists as the
single-largest threat to the survival of Gangetic Dolphins, whose numbers are
declining in most parts of their natural habitat.
 This is mainly due to construction of dams and barrages on the river.
 Aside from losing habitat to increased developmental work on the river, the dolphins
also suffered due to depletion of prey base, accidental mortality in fishing nets
and accidents with vessel propellers.
 The large-scale modification of the river, the proposed movement of numerous ships,
may well sound the death knell of the species. The species are practically blind, and
rely on bio-sonar method to move around. The ships’ noise-levels would disrupt
the ability to navigate, and find prey.
 Assam's Guwahati became the first city in the country to have its own city
animal with the district administration declaring the Gangetic river dolphin as the
mascot.
 The animal is protected under Schedule I of the Indian Wildlife Protection Act,
1972.

 9446331522 9446331522 www.neoias.com Page 61


https://t.me/testseries4exams
Download All PDF form here :- https://t.me/pdf4exams
NEO IAS Most Important 150 Environmental Science Model Questions from TPS 2018

108. Which one of the following is not a correct statement?


(a) Carbon footprint is the amount of carbon dioxide released into the atmosphere as a
result of the activities of a particular individual, organization, or community.
(b) Carbon sequestration enables the long-term storage of carbon.
(c) Photosynthesis is not a means for carbon sequestration
(d) There are artificial processes for carbon sequestration

Answer (c)
Explanation
 Carbon footprint is the amount of carbon dioxide released into the atmosphere
as a result of the activities of a particular individual, organization, or community.
 Carbon sequestration is both a natural and artificial process by which carbon
dioxide is removed from the Earth’s atmosphere and then stored in liquid or solid
form.
 Carbon sequestration, the long-term storage of carbon in plants, soils, geologic
formations, and the ocean.
 It is a process of capture and deliberate, whether natural or artificial, storage of CO2
over a long period of time. The initial purpose of doing this is to delay global
warming and avoid extreme climate change.
 A more scientific explanation (and example) is; the removal and storage of carbon
from the atmosphere to sinks – oceans, soil, forests – through physical means and
the natural process best known as photosynthesis.

109. Consider the following pairs:


Protected areas well known for
1. Manas : Pygmy Hog
2. Ranthambhore : Tiger
3. Eravikulam : Nilgiri Tahr
Which of the statements given above is/are correct?
(a) 1 only (b) 2 and 3 only (c) 1 and 3 only (d) 1, 2 and 3

Answer (d)
Explanation
Protected areas Well known for
1. Manas (Assam) : Pygmy Hog
2. Ranthambhore (Rajasthan) : Tiger
3. Eravikulam (Kerala) : Nilgiri Tahr
 The Pygmy Hog is the smallest and the rarest wild suid (wild pig) in the world. This
species is dependent on riverine communities, typically comprising dense tall
grasslands, commonly referred to as 'thatchland'. Previously they spread across
India, Nepal, and Bhutan, but it is now confined to a very few locations in and
around Manas National Park in north-western Assam.

110. With reference to Pika, a species which was discovered recently from India, consider
the following statements:
1. It is a mammal belonging to the rabbit and hare family.
2. Pikas are Pioneer species and ecosystem engineers in an ecosystem
3. It was discovered from the Himalayas in Sikkim.
4. They hibernate during winter to escape from food shortage
Which of the statements given above is/are correct?
(a) 1, 2 and 3 only (b) 2, only (c) 1 and 3 only (d) 1, 2, 3 and 4

 9446331522 9446331522 www.neoias.com Page 62


https://t.me/testseries4exams
Download All PDF form here :- https://t.me/pdf4exams
NEO IAS Most Important 150 Environmental Science Model Questions from TPS 2018

Answer (c)
Explanation
 Scientists claim to have discovered a new species of Pika, a mammal belonging to
the rabbit and hare family (Lagomorpha), in the Himalayas in Sikkim.
 After six years of research, a team of international collaborators, led by scientists
from Bengaluru’s National Centre for Biological Sciences (NCBS) announced the
discovery.
 Pikas live in the mountains or in temperate regions.
 Pikas are a keystone species and ecosystem engineers.
 Pikas do not hibernate unlike other mammalian species inhabiting such cold
climates.

111. With reference to coral bleaching, consider the following statements:


1. Corals will expel the algae (zooxanthellae) living in their tissues causing the coral to
turn white.
2. Coral beaching occurs only due to the rise in temperature of water
Which of the statements given above is/are correct?
(a) 1 only (b) 2 only
(c) Both 1 and 2 (d) Neither 1 nor 2

Answer (a)
Explanation
 When corals are stressed by changes in conditions such as temperature, light, or
nutrients, they expel the symbiotic algae living in their tissues, causing them to turn
completely white.
 Coral bleaching takes place when the symbiotic relationship between algae
(zooxanthellae) and their host corals breaks down under certain environmental
stresses.
 This results in the host expelling their zooxanthellae. In the absence of symbiotic
algae, the corals expose their white underlying calcium carbonate coral skeleton and
the affected coral colony becomes pale in colour.
 Coral bleaching can be activated and persist during varied environmental stresses
 Coral reefs are very sensitive ecosystems – even slight changes in water temperature
and salinity can lead to coral “bleaching” and even death.

112. Consider the following:


1. Clouded leopard
2. Cheetah
3. Black-necked crane
4. Pangolin
Which of the above are naturally found in India?
(a) 1, 2 and 3 only (b) 1, 3 and 4 only (c) 2 and 4 only (d) 1, 2, 3 and 4

Answer (b)
Explanation
 Except Cheetah Rest of the all are naturally found in India.
 In India Cheetah are extinct in the wild or regionally extinct.
 Cheetahs are particularly vulnerable to habitat loss and fragmentation of their
habitat.
 One reason for their extirpation across most of their Asian range is thought to have
been the live capture of cheetahs, which were then trained to hunt deer and
gazelle as sport for the aristocracy.

 9446331522 9446331522 www.neoias.com Page 63


https://t.me/testseries4exams
Download All PDF form here :- https://t.me/pdf4exams
NEO IAS Most Important 150 Environmental Science Model Questions from TPS 2018

 Other key causes of the disappearance of Cheetah from the region are likely to have
been depletion of wild prey, especially gazelles, the direct killing of Cheetahs.
 Black necked crane winters mainly in remote parts in Bhutan, and Arunachal
Pradesh, India. The ramsar site Tsomoriri lake in India is said to represent the
only breeding ground outside of China for the Black-necked crane.
 The Clouded Leopard is found in the Himalayan foothills.
 In India, Pangolin is widely distributed from the plains and lower hills south of the
Himalayas to extreme southern India.

113. With reference to the term ‘Green Good Deeds’, sometimes seen in the news,
consider the following statements.
1. It is a people-oriented campaign launched by the Union Ministry of Environment,
Forests and Climate Change (MoEFCC)
2. The purpose of campaign is to sensitise people and students, in particular about
climate change and global warming.
Which of the statements given above is/are correct?
(a) 1 only (b) 2 only (c) Both 1 and 2 (d) Neither 1 nor 2

Answer (c)
Explanation
 The Union Ministry of Environment, Forests and Climate Change (MoEFCC) has
launched Green Good Deeds campaign.
 The purpose of the people-oriented campaign is to sensitise people and students, in
particular about climate change and global warming.
 The campaign aims to broaden its base with the involvement of teachers, students
and other voluntary organisations.

114. Vulture population in India perished since 90s possibly due to poisoning after eating
carcass of cattle treated with the anti-inflammatory drug Diclofenac. The drug was
banned by the Centre. Another drug which came as an alternative also caused the
same effect on the vulture population, it was banned by the Tamil Nadu Government in
2015. Which one of the following could be that non-steroidal anti-inflammatory drug
(NSAID)?
(a) Aspirin
(b) Ketoprofen
(c) Streptomycin
(d) Meloxicam

Answer (b)
Explanation
 Around 97% of the vulture population in India perished since 90s possibly due to
poisoning after eating carcass of cattle treated with the anti-inflammatory drug
Diclofenac.
 The Centre had banned veterinary use of the drug in 2006.
 Vultures act as scavengers, preying on dead animals and Diclofenac in carcasses led
to slow death of vultures.
 Wildlife activists said that Ketoprofen, which came as an alternative, caused the
same effect on the vulture population.

 9446331522 9446331522 www.neoias.com Page 64


https://t.me/testseries4exams
Download All PDF form here :- https://t.me/pdf4exams
NEO IAS Most Important 150 Environmental Science Model Questions from TPS 2018

 Ketoprofen - a non-steroidal anti-inflammatory drug (NSAID) prescribed by


veterinarians as a pain killer for cattle - can kill vultures when they eat the
carcasses of animals treated with it.
 They cause acute kidney failure in vultures.

115. Consider the following pairs


1. Lion-tailed Macaque : Western Ghats
2. Red Sanders : Eastern Ghats
3. Holloock Gibbon : North West India
Which of the above pairs is / are correctly matched?
(a) 1 and 2 only (b) 3 only (c) 1, 2 and 3 (d) None

Answer (a)
Explanation
1. Lion-tailed Macaque : Western Ghats
2. Red Sanders : Eastern Ghats
3. Holloock Gibbon : North East India

116. With reference to Methanol, sometimes seen in news, consider the following
statements:
1. Methanol can be manufactured from natural gas and coal.
2. Methanol can be an alternative to conventional transportation fuels.
Which of the statements given above is/are correct?
(a) 1 only (b) 2 only (c) Both 1 and 2 (d) Neither 1 nor 2

Answer (c)
Explanation
 Methanol is a liquid chemical with the formula CH3OH (often abbreviated MeOH).
 It is colorless, volatile, flammable, and poisonous.
 Methanol is also used to produce biodiesel by transesterification of vegetable
oil.
 Methanol can be derived from coal at cost comparable to that of synthetic gasoline.
 Methanol can be an alternative to conventional transportation fuels.
 Its principal uses are in organic synthesis, as a fuel, solvent, and antifreeze.
 It may potentially be used to create hydrogen for hydrogen fuel cell vehicles in the
future.
 Union Roads Minister said that the government would soon announce a policy for
15% blending of methanol in petrol to make it cheaper and to reduce pollution.

117. Consider the following statements.


1. Polar bears hunt on penguins
2. Polar bears and penguins are living in symbiotic relation helping each other to
survive.
3. Polar bear and penguins never coexist under natural conditions
Which of the statements given above is/are correct?
(a) 1 and 2 (b) 2 only (c) 2 and 3 (d) 3 only

Answer (d)
Explanation
 Polar bears live in the Arctic, near the North Pole.

 9446331522 9446331522 www.neoias.com Page 65


https://t.me/testseries4exams
Download All PDF form here :- https://t.me/pdf4exams
NEO IAS Most Important 150 Environmental Science Model Questions from TPS 2018

 Penguins live on Antarctica and the neighbouring continents, near the South Pole.
They are literally poles apart.

118. With reference to ‘World Environment Day (WED)’, consider the following
statements.
1. Observed annually on 5th June for encouraging worldwide awareness and action for
the protection of our environment.
2. It was established by United Nations General Assembly to mark opening of United
Nations Conference on the Human Environment in 1972.
Which of the statements given above is/are correct?
(a) 1 only (b) 2 only (c) Both 1 and 2 (d) Neither 1 nor 2

Answer (c)
Explanation
 World Environment Day (WED) observed annually on 5th June for encouraging
worldwide awareness and action for the protection of our environment.
 It was established by United Nations General Assembly to mark opening of United
Nations Conference on the Human Environment in 1972 (Stockholm Conference).
 The first World Environment Day was observed in 1973 and since then it is being
held every year with different themes.
 WED serves as a flagship campaign for raising awareness on emerging
environmental issues from marine pollution, human overpopulation, and global
warming, to sustainable consumption and wildlife crime.

119. With reference to ‘Atal Bhujal Yojana (ABY)’, consider the following statements.
1. ABY is a water conservation scheme to tackle ever-deepening crisis of depleting
groundwater level.
2. The objective of scheme is to recharge ground water and create sufficient water
storage for agricultural purposes.
3. Its focus is primarily on involvement of communities and convergence with different
water schemes.
4. It also focuses on revival of surface water bodies.
Which of the statements given above is/are correct?
(a) 1only (b) 1 and 2 only (c) 3 and 4 only (d) 1, 2, 3 and 4

Answer (d)
Explanation
 The Union Government has formulated ambitious water conservation scheme Atal
Bhujal Yojana (ABY) to tackle ever-deepening crisis of depleting groundwater
level.
 The objective of scheme is to recharge ground water and create sufficient water
storage for agricultural purposes.
 It also focuses on revival of surface water bodies so that ground water level can
be increased, especially in the rural areas.
 It will give emphasis to recharging ground water sources and ensure efficient use
of water by involving people at local level.
 Centre will support half of the total project cost and rest of the budgetary cost
will be shared by the World Bank.

 9446331522 9446331522 www.neoias.com Page 66


https://t.me/testseries4exams
Download All PDF form here :- https://t.me/pdf4exams
NEO IAS Most Important 150 Environmental Science Model Questions from TPS 2018

 This scheme will help those who are in need for constant ground water supply
especially farmers who have been hard impacted by acute shortage of ground
water for past several years.
 Its focus is primarily on involvement of communities and convergence with
different water schemes.
 Its major component is making society responsible and bringing about behaviour
change to manage groundwater resource.

120. With reference to National Wildlife Action Plan (NWAP) of India for 2017-2031,
consider the following statements.
1. This is the Third National Wildlife Action Plan.
2. This NWAP is unique as this is the first time India has recognized the concerns
relating to climate change impact on wildlife.
Which of the statements given above is/are correct?
(a) 1 only (b) 2 only (c) Both 1 and 2 (d) Neither 1 nor 2

Answer (c)
Explanation
 The Union Ministry of Environment, Forests and Climate Change (MoEFCC) has
unveiled third National Wildlife Action Plan for 2017-2031 to chalk out future
road map for wildlife conservation.
 The plan was unveiled by Environment Minister on the inaugural day of Global
Wildlife Programme (GWP) conference. The GWP, initiated in 2015, is a World-Bank
led partnership of 19 countries to promote the conservation and sustainable
development by combating trafficking in wildlife.
 The third action plan comes after the first plan in 1983 and second from 2002 till
2016.
 The third National Wildlife Action Plan is unique as this is the first time India has
recognised the concerns relating to climate change impact on wildlife and
stressed on integrating actions that need to be taken for its mitigation and
adaptation into wildlife management planning processes.
 The key focus areas of this plan includes integration of climate change into wildlife
planning, conservation of coastal and marine ecosystem, mitigation of human-
wildlife conflict, focus on wildlife health among others.
 The NWAP has five components, 17 themes, 103 conservation actions and 250
projects. The five components are
1. strengthening and promoting the integrated management of wildlife and their
habitats;
2. adaptation to climate change and promoting integrated sustainable
management of aquatic biodiversity in India;
3. promoting eco-tourism, nature education and participatory management;
4. strengthening wildlife research and monitoring of development of human
resources in wildlife conservation and
5. enabling policies and resources for conservation of wildlife in India.
 It recommended assisted migration of wildlife and anticipatory planting along
ecological gradients, as climate change may result in die-offs of certain tree species
that are unable to adapt to newer environmental conditions.

 9446331522 9446331522 www.neoias.com Page 67


https://t.me/testseries4exams
Download All PDF form here :- https://t.me/pdf4exams
NEO IAS Most Important 150 Environmental Science Model Questions from TPS 2018

 The plan adopts landscape approach in conservation of all wildlife – uncultivated


flora and fauna that have an ecological value to the ecosystem and to mankind
irrespective of where they occur.
 It accords special emphasis to rehabilitation of threatened species of wildlife while
conserving their habitats which include inland aquatic, coastal and marine
ecosystems.
 It addresses rising human-animal conflict owing to shrinkage, fragmentation and
deterioration of habitats generating animosity against wild animals and protected
areas.
 The plan underscores the increasing need for people’s support for conservation of
wildlife and to this effect recommends strengthening the ‘core buffer multiple use
surround’ structure with higher inputs for eco-development, education,
innovation, training, extension, conservation awareness and outreach
programs.
 The government has also underlined an increased role of private sector in wildlife
conservation. The plan lays down that the Centre would ensure that adequate and
sustained funding including Corporate Social Responsibility funds are made
available for the National Wildlife Action Plan implementation.

121. Consider the following statements:


1. Many primates are arboreal animals
2. Insects are found in all kinds of habitats
3. Amphibians undergo only terrestrial life
4. Amphibian are most sensitive to environmental change
Which of the statements given above is/are correct?
(a) 1 and 3 only (b) 2 only (c) 1, 2 and 4 only (d) 1, 2, 3 and 4

Answer (c)
Explanation
 Any of mammals that are characterized especially by advanced development of
binocular vision resulting in stereoscopic depth perception, specialization of the
hands and feet for grasping, and enlargement of the cerebral hemispheres and that
include humans, apes, monkeys, and related forms (such as lemurs and tarsiers).
Many primates are arboreal animals. Arboreal animals are that living in trees.
 Insects are found in all kinds of habitats. Insects live in just about every habitat
on Earth, from the sands of hot deserts to cold snowy mountain streams. Most
insects live on land, in fact about 97% do. Many insects spend all of their life on
land, such as bees and caterpillars (which of course become moths or butterflies).
However, some insects live in freshwater. Dragonflies begin life in water such as
rivers, lakes, and ponds, but then take to the air as adults. And other insects, such
as the diving beetle, spend most of their time in the water, but they must return
repeatedly to the surface of the water to breathe air.
 Amphibians are cold-blooded vertebrates (vertebrates have backbones) that don’t
have scales. They live part of their lives in water and part on land. Examples of
amphibian are frogs, toads, salamanders, newts and caecilians. The world's
amphibians are in crisis; populations of most of the world's 6200 species of frogs,
toads, salamanders, newts and caecilians are in decline and one in three amphibian
species are threatened with extinction. Several species have become extinct in the
 9446331522 9446331522 www.neoias.com Page 68
https://t.me/testseries4exams
Download All PDF form here :- https://t.me/pdf4exams
NEO IAS Most Important 150 Environmental Science Model Questions from TPS 2018

last 20 years. The cause of this disastrous situation is human population growth
and the destructive ecological 'footprint' that inevitably results from it. Natural
habitats are destroyed and degraded as they are cleared for housing, agriculture and
industry; surviving habitats become polluted; climate change alters natural
environments, often making them hostile to their natural inhabitants. In the wake
of an environment that human disturbances rapidly altered, ecologists have adopted
amphibians as models for studying applied ecological issues such as habitat
loss, pollution, disease, and global climate change. Some of the characteristics of
amphibians that make them useful models for studying these environmental
problems are highlighted, including their trophic importance, environmental
sensitivity, research tractability, and impending extinction.

122. With reference to The Genetic Engineering Appraisal Committee (GEAC), consider
the following statements.
1. GEAC is the country’s biotech regulator, functioning in the Ministry of Environment.
2. GEAC is the apex body for approval of activities involving large scale use of
hazardous microorganisms and recombinants in research and industrial production
from the environmental angle.
3. It is constituted under ‘Rules 1989’, under the Environment Protection Act, 1986.
Which of the statements given above is/are correct?
(a) 1 only(b) 2 and 3 only (c) 1 and 3only (d) 1, 2 and 3

Answer (d)
Explanation
 The Genetic Engineering Appraisal Committee (GEAC) is the apex body constituted
in the Ministry of Environment and Forests under 'Rules for Manufacture, Use,
Import, Export and Storage of Hazardous Microorganisms/Genetically
Engineered Organisms or Cells 1989', under the Environment Protection Act,
1986. These rules commonly referred as ‘Rules 1989’.
 The Genetic Engineering Appraisal Committee (GEAC) functions in the Ministry of
Environment, Forest and Climate Change (MoEF&CC).
 GEAC is chaired by the Special Secretary/Additional Secretary of MoEF&CC and co-
chaired by a representative from the Department of Biotechnology (DBT).

Functions of GEAC
 To appraise activities involving large scale use of hazardous microorganisms and
recombinants in research and industrial production from the environmental angle.
 To appraise proposals relating to release of genetically engineered organisms and
products into the environment including experimental field trials.
 The committee or any persons authorized by it has powers to take punitive action
under the Environment Protection Act.

123. The main constituents of biogas are


(a) Methane and carbon dioxide
(b) Methane and nitric oxide
(c) Methane and sulphur dioxide
(d) Methane, hydrogen and nitric oxide

 9446331522 9446331522 www.neoias.com Page 69


https://t.me/testseries4exams
Download All PDF form here :- https://t.me/pdf4exams
NEO IAS Most Important 150 Environmental Science Model Questions from TPS 2018

Answer (a)
Explanation
 Biogas is a clean renewable energy source. Biogas is a type of biofuel that is
naturally produced from the decomposition of organic waste.
 When organic matter, such as food scraps and animal waste, break down in an
anaerobic environment (an environment absent of oxygen) they release a blend of
gases, primarily methane and carbon dioxide.
 Because this decomposition happens in an anaerobic environment, the process of
producing biogas is also known as anaerobic digestion.
 Anaerobic digestion is a natural form of waste-to-energy that uses the process of
fermentation to breakdown organic matter.

124. Which of the following statements is/are correct with reference to the Animal
Welfare Board of India (AWBI)?
1. AWBI is a statutory advisory body on Animal Welfare Laws under the Ministry of
Environment & Forests
2. AWBI was established under the Prevention of Cruelty to Animals Act, 1960
3. Recently Government has shifted headquarters of AWBI to Ballabhgarh in Faridabad
District of Haryana from Chennai, Tamil Nadu.
Select the correct answer using the codes given below:
(a) 1 only(b) 2 and 3 only (c) 1 and 3only (d) 1, 2 and 3

Answer (d)
Explanation
 AWBI is a statutory advisory body on Animal Welfare Laws under the Ministry of
Environment & Forests is the country’s apex institution for promoting welfare of
animals
 The Government of India established Animal Welfare Board of India mainly for two
purposes: for the promotion of animal welfare generally and for protecting
animals from being subjected to unnecessary pain or suffering.
 Animal Welfare Board of India (AWBI) was established in 1962 under Section 4 of
the Prevention of Cruelty to Animals Act, 1960
 AWBI provides financial assistance in the form of grants-in-aid to Animal Welfare
Organisations
 AWBI is Government of India’s largest funding agency for promotion of animal
welfare initiatives.
 The headquarters of the Animal Welfare Board of India (AWBI) has been shifted from
Chennai to Haryana's Ballabhgarh (Faridabad) for "better coordination" between
the environment ministry and the the board,
 Rule 3 of the Animal Welfare Board (Administrative) Rules, 1962 reads as - 'The
Headquarter of the Board hall be at New Delhi or at such other place as the central
government, may, after consultation with the Board direct'

125. Recently, for the first time in our country, which of the following states has launched
commercial e-bus service as a part of green transport?
(a) Arunachal Pradesh
(b) Himachal Pradesh
(c) Karnataka
(d) Maharashtra

 9446331522 9446331522 www.neoias.com Page 70


https://t.me/testseries4exams
Download All PDF form here :- https://t.me/pdf4exams
NEO IAS Most Important 150 Environmental Science Model Questions from TPS 2018

Answer (b)
Explanation
 Himachal Pradesh is the first state to launch India's first commercial e-bus
service.
 In a boost for green transport in Himachal Pradesh, Transport Minister flagged off
the country’s first electric bus service for Rohtang Pass, a tourist destination
near Manali.
 It is a pioneering initiative to check vehicular pollution at Rohtang Pass in the wake
of the National Green Tribunal (NGT) orders to restrict the number of diesel and
petrol operated tourist vehicles in the ecologically fragile area.

126. With reference to ‘the Convention on International Trade in Endangered Species of


Wild Fauna and Flora (CITES)’, consider the following statements.
1. CITES is an international agreement between governments.
2. It aims to stop overexploitation of wild fauna and flora through international trade.
3. The agreement also regulates trade in items made from plants and animals
4. CITES encourage legal, sustainable, and traceable wildlife trade.
5. CITES Secretariat is administered by United Nations Environment Programme
(UNEP)
Which of the statements given above is/are correct?
(a) 1, 2 and 3 only (b) 2, 3 and 4 only (c) 1, 4 and 5 only (d) 1,2,3,4 and 5

Answer (d)
Explanation
 CITES is an international agreement between governments which aims to ensure
that international trade in specimens of wild animals and plants does not
threaten their survival. In another words it aims to stop overexploitation of wild
fauna and flora through international trade.
 The CITES Secretariat is administered by UNEP and is located at Geneva,
Switzerland (Headquarters).
 The purpose of CITES is to ensure that wild fauna and flora in international trade
are not exploited unsustainably.
 CITES regulates international trade in over 36,000 species of plants and
animals, including their products and derivatives, ensuring their survival in the
wild with benefits for the livelihoods of local people and the global environment.
 CITES is an international convention that combines wildlife and trade themes with a
legally binding instrument for achieving conservation and sustainable use
objectives.
 The species covered by CITES are listed in three Appendices, according to the
degree of protection they need.
 CITES regulates international trade in wild fauna and flora listed in its Appendices
on the basis of a system of permits and certificates which are issued when certain
conditions are met, and which must be presented when leaving and entering a
country.
 Wildlife trade should be legal, sustainable and traceable: CITES seeks to ensure
that international trade in protected species (listed species) is legal, sustainable and
traceable.
 CITES is not having any enforcement authority that means they do not have its
own police force, they depend on laws and enforcement procedures of each
nation.

 9446331522 9446331522 www.neoias.com Page 71


https://t.me/testseries4exams
Download All PDF form here :- https://t.me/pdf4exams
NEO IAS Most Important 150 Environmental Science Model Questions from TPS 2018

 CITES was drafted as a result of a resolution adopted in 1963 at a meeting of


members of IUCN (The World Conservation Union).
 The text of the Convention was finally agreed on 3 March 1973, and on 1 July 1975
CITES entered in force.
 Although CITES is legally binding on the Parties – in other words they have to
implement the Convention – it does not take the place of national laws.
 Rather it provides a framework to be respected by each Party, which has to adopt
its own domestic legislation to ensure that CITES is implemented at the national
level.
 CITES works by subjecting international trade in specimens of selected species
to certain controls.
 The Government of India signed the Convention in July 1976, which was ratified
in October 1976.

127. In India, in which one of the following types of forests is Sandalwood a dominant
tree species?
(a) Tropical moist deciduous forest
(b) Tropical rain forest
(c) Tropical thorn scrub forest
(d) Temperate forest with grasslands

Answer (a)
Explanation
TROPICAL MOIST DECIDUOUS FOREST:
 Trees are normally deciduous, because of the dry period summer, during which they
shed their leaves to withstand the drought.
 The trees are tall, with broad and branched trunks and roots to hold them firmly to
the ground. Species found in these forests include Sandalwood, Teak, Sal, bamboo,
shisham, hurra, mahua, amla, semul, kusum, arjun, mulberry, khair, rosewood,
deodar, mango, etc.

128. Consider the following statements:


1. Carrying capacity is the largest number of any given species that a habitat can
support indefinitely
2. Carrying capacity is determined by the level of primary productivity
Which of the statements given above is/are correct?
(a) 1 only(b) 2 only (c) Both 1 and 2 (d) Neither 1 nor 2

Answer (c)
Explanation
 For a given region, carrying capacity is the maximum number of individuals of a
given species that an area's resources can sustain indefinitely without
significantly depleting or degrading those resources. Determining the carrying
capacities for most organisms is fairly straightforward. For humans carrying
capacity is much more complicated. The definition is expanded to include not
degrading our cultural and social environments and not harming the physical
environment in ways that would adversely affect future generations. Carrying
capacity is a measure of sustainability with the changing conditions environment.
 Carrying capacity is determined by the level of primary productivity and other
factors. Or the carrying capacity is mainly determined by available food and space.
Primary production is defined as the amount of biomass or organic matter
produced per unit area over a time period by plants during photosynthesis. It is
expressed in terms of weight or energy. Or Primary productivity means the rate of
 9446331522 9446331522 www.neoias.com Page 72
https://t.me/testseries4exams
Download All PDF form here :- https://t.me/pdf4exams
NEO IAS Most Important 150 Environmental Science Model Questions from TPS 2018

food produced by producers (autotrophs) or the rate of solar energy trapped at


first trophic level.

129. With reference to “CA|TS Partnership”, sometimes seen in the news, consider the
following statements.
1. CA|TS is a partnership between governments, NGOs and tiger conservation areas to
define and implement the conservation standards.
2. CA|TS is a set of criteria which allows tiger sites to check if their management will
lead to successful tiger conservation.
3. Even though India has the largest number of tiger population, so far India doesn’t
have any CA|TS approved site.
Which of the statements given above is/are correct?
(a) 1 and 2 only (b) 2 only (c) 1 and 3 only (d) 1, 2 and 3

Answer (a)
Explanation
 According to recent survey, only 13% of tiger conservation areas that are part of
Conservation Assured | Tiger Standards (CA|TS) Partnership meet global standards.
The survey was conducted over hundred tiger conservation areas by 11 leading
conservation organisations and countries with tiger ranges that are part of CA|TS
Partnership.
 CA|TS is a set of criteria which allows tiger sites to check if their
management will lead to successful tiger conservation. CA|TS is organised
under seven pillars and 17 elements of critical management activity.
 CA|TS is a management tool which sets basic criteria (e.g. minimum standards) for
effective management of tiger conservation reserves or other conservation reserves
and protected areas which have tiger populations.
 CA|TS was developed by tiger and protected area experts. Officially launched in
2013, CA|TS is an important part of Tx2, the global goal to double wild tiger
numbers by the year 2022.
 Conversation Assured Tiger Standards (CA|TS) is a partnership between
governments, NGOs and tiger conservation areas to define and implement the
conservation standards.
 Of the 13 tiger range countries Nepal, India, Bangladesh and Russia have registered
and rolled out CA|TS, whilst discussion is ongoing with Bhutan, Indonesia,
Thailand, Malaysia and China.
 The CA|TS team has been working closely with IUCN’s GLPA (Green List for
Protected Areas) Management Team.

How does CA|TS work?


 Sites taking part will initially be ‘registered’ (standards not yet attained) then, when
all required standards are met, ‘approved’ (standards achieved). An approved site
has achieved excellence in tiger site management. Sites are evaluated through an
assessment and independent review process.
 So far there are 14 registered sites in India, out of that only one site got approved by
CA|TS and it is Lansdowne Forest Division.
 The Worldwide Fund for Nature (WWF) and Global Tiger Forum (GTF) have
included the Lansdowne forest division (FD) in Garhwal in the list of areas
 9446331522 9446331522 www.neoias.com Page 73
https://t.me/testseries4exams
Download All PDF form here :- https://t.me/pdf4exams
NEO IAS Most Important 150 Environmental Science Model Questions from TPS 2018

accredited under the Conservation Assured Tiger Standards (CATS) scheme as the
forest area has the required parameters to be rated in the list of Conservation
Assured Tiger Standards (CATS). Lansdowne is only the third region in the world
after Chitwan National Park in Nepal and a forest in Russia to be included in the
list. The accreditation will remain valid for a period of three years after which the
state forest department will have to apply for its renewal.
 The CATS list includes stringent standards for effective management of target
species and encourages assessment of these standards in relevant conservation and
protected areas.
Some CA|TS support groups are:
 GTF, IUCN, WWF, UNDP, PANTHERA, etc

130. With reference to allopatric species, consider the following statements:


1. They are species having exclusive area of geographical distribution.
2. They are species facing extreme threat due to habitat loss.
Which of the statements given above is/are correct?
(a) 1 only (b) 2 only (c) Both 1 and 2 (d) Neither 1 nor 2

Answer (a)
Explanation
 The speciation occurring when the population becomes separated by geographical
barriers is called allopatric speciation.
 When a population is geographically continuous, the allele frequencies among its
members are similar; however, when a population becomes separated, the allele
frequencies between the two groups can begin to vary. Thus the chance of
interbreeding between these populations is reduced.
 If the separation between groups continues for a long period of time, the differences
between their alleles can become more and more pronounced due to differences in
climate, predation, food sources, and other factors, eventually leading to the
formation of a new species.
 Each separated population acquires mutations by natural selection to adapt to the
new environment.
 A long time, reproductive isolation sets in separating two populations into two
species.
 Allopatric speciation events can occur either by dispersal, when a few members of a
species move to a new geographical area, when a natural situation, such as the
formation of a river or valley, physically divide organisms.

131. With reference to ‘Neelakurinji (Strobilanthes kunthiana)’, sometimes seen in the


news, consider the following statements:
1. It is a plant species found in a certain part of South India.
2. The species blossoms only once in 12 years.
Which of the statements given above is/are correct?
(a) 1 only (b) 2 only (c) Both 1 and 2 (d) Neither 1 nor 2

Answer (c)
Explanation
 Kurinji or Neelakurinji is a shrub that is found in the shola forests of the Western
Ghats in South India.
 The species blossoms only once in 12 years.
 Neelakurinji or Strobilanthes kunthiana blooms in so varieties, a majority of them
being blue in color.
 9446331522 9446331522 www.neoias.com Page 74
https://t.me/testseries4exams
Download All PDF form here :- https://t.me/pdf4exams
NEO IAS Most Important 150 Environmental Science Model Questions from TPS 2018

 'Neela' literally translates to 'blue' and kurinji is the name given to it by the tribals of
the area.
 Plants that bloom at long intervals like kurinji are called plietesials.
 The plant is usually 30 to 60 cm high on the hills.
 They once used to cover the Nilgiri Hills and Palani Hills like a carpet during its
flowering season. Now plantations and dwellings occupy much of their habitat.

132. Which of the following is the historic agreement signed to protect the world’s largest
tropical peatland?
(a) Brazzaville Declaration
(b) Basel convention
(c) Minamata convention
(d) Paris agreement

Answer (a)
Explanation
 The Brazzaville Declaration aims to implement coordination and cooperation
between different government sectors to protect the benefits provided by peatland
ecosystems.
 The agreement is the beginning of a deep collaboration between Indonesia – covered
by vast expanses of peatlands – and the Congo Basin.
 In an unprecedented move to protect the Cuvette Centrale region in the Congo
Basin, the world’s largest tropical peatlands (which are about the size of
England), from unregulated land use and prevent its drainage and degradation, the
Democratic Republic of Congo (DRC), the Republic of Congo and Indonesia jointly
signed the Brazzaville declaration that promotes better management and
conservation of this globally important carbon store.
 Researchers mapped the Cuvette Centrale peatlands in the central Congo basin and
found they cover 145,500 sq km – an area larger than England. The swamps could
lock in 30bn tonnes of carbon that was previously not known to exist, making the
region one of the most carbon-rich ecosystems on Earth.
 The equivalent of three years of global greenhouse gas emissions are stored in the
Congo Basin, emissions that could be released if the peatlands are degraded or the
natural wetlands drained.
 Peatlands are wetlands that contain a mixture of decomposed organic material,
partially submerged in a layer of water, lacking oxygen. The complex biodiversity of
the peatlands means they are home to a variety of species, but their high carbon
content makes them uniquely vulnerable to incineration if they are drained.
 Peatlands only cover 3% of the land surface; they contain twice as much carbon as
the entire biomass of the world’s forests.
 The main peatland management principle is to keep the peatlands wet.
 Destroying the peatlands would be a grave assault on the Paris Agreement and the
climate.

 9446331522 9446331522 www.neoias.com Page 75


https://t.me/testseries4exams
Download All PDF form here :- https://t.me/pdf4exams
NEO IAS Most Important 150 Environmental Science Model Questions from TPS 2018

133. With reference to the transpiration, which of the following statements is/are correct?
1. Transpiration is the evaporation of water into the atmosphere from the leaves.
2. The rate of transpiration increases with decreasing humidity.
3. The rate of transpiration decreases with decreasing light intensity.
Select the correct answer using the code given below.
(a) 1 only (b) 2 and 3 only (c) 1 and 3 only (d) 1, 2 and 3

Answer (d)
Explanation
 Transpiration is the evaporation of water into the atmosphere from the leaves.
 The rate of transpiration increases with decreasing humidity.
 The rate of transpiration decreases with decreasing light intensity.

134. With reference to an initiative called ‘Global Peatlands Initiative’, consider the
following statements.
1. UN Environment (UNEP) initiated the Global Peatlands Initiative (GPI)
2. It aims to conserve and restore peatlands globally, and sustainable management of
peatlands
3. Even though Ramsar convention is the worldwide instrument for the sustainable use
of wetlands, it doesn’t have any relation with GPI
Which of the statements given above is/are correct?
(a) 1 and 2 only (b) 2 only (c) 1 and 3 only (d) 3 only

Answer (a)
Explanation
 To conserve and restore peatlands globally, UN Environment (UNEP) initiated the
Global Peatlands Initiative (GPI) whose membership includes forest and tropical peat
countries, multilateral organizations, and non-governmental organizations that have
committed to the protection, restoration and sustainable management of peatland.
 The Global Peatlands Initiative is an effort by leading experts and institutions to
save peatlands as the world’s largest terrestrial organic carbon stock and to prevent
it being emitted into the atmosphere.
 The current greenhouse gas emissions from drained or burned peatlands are
estimated to amount up to five percent of the global carbon budget — in the range of
two billion tonnes CO2 per year.
 Partners to the Initiative will work together within their respective areas of expertise
to improve the conservation, restoration and sustainable management of peatlands.
In this way the Initiative will contribute to several Sustainable Development Goals,
including by reducing greenhouse gas emissions, maintaining ecosystem services
and securing lives and livelihoods through improved adaptive capacity.
 By 2030 the Initiative aims to scale up the conservation, restoration and sustainable
management of peatlands in up to 25 key countries, to reduce their greenhouse gas
emissions and maintain the benefits which their ecosystems provide, and thereby
contribute to several Sustainable Development Goals.
 The Ramsar Convention on Wetlands, which represents the commitment of 169
Contracting Parties is very pleased to announce the launch of this initiative as a
founding partner along with the United Nations Environment Programme (UNEP),

 9446331522 9446331522 www.neoias.com Page 76


https://t.me/testseries4exams
Download All PDF form here :- https://t.me/pdf4exams
NEO IAS Most Important 150 Environmental Science Model Questions from TPS 2018

United Nations Food and Agriculture Organization (FAO), Wetlands International


and other institutions.
 The founding members of the Global Peatlands Initiative are the governments of
Indonesia, Peru, and the Republic of Congo, the UN Environment Programme
(UNEP), the Ramsar Convention on Wetlands, FAO, the Joint Research Center of the
European Commission, Wetlands International, UNEP-WCMC, GRID-Arendal, the
European Space Agency, World Resources Institute, Greifswald Mire Centre and
Satelligence.
 As a worldwide instrument for the sustainable use of wetlands, the Ramsar
Convention plays an important role in highlighting the climate regulation function of
peatlands and in stimulating their conservation and restoration.

135. With reference to the salt production in India, consider the following statements:
1. India is the third largest salt producing Country in the World after China and USA.
2. Gujarat is the leading producer of salt in India.
3. Sambhar Lake is a source of salt in Rajasthan.
4. Salt mining is carried out in Himachal Pradesh.
Which of the statements given above is/are correct?
(a) 1, 2 and 3 only (b) 2 and 3 only (c) 1 and 4 only (d) 1, 2, 3 and 4

Answer (d)
Explanation
 India is the third largest Salt producing Country in the World after China and
USA.
 Today India has not only achieved self-sufficiency in production of salt to meet
its domestic requirement but also in a position of exporting surplus salt to
foreign countries.
 The main sources of salt in India are:
o Sea brine
o Lake brine
o Sub-soil brine and
o Rock salt deposits
MAJOR SALT PRODUCING CENTERS:
 Sea water is an inexhaustible source of salt.
 Salt production along the coast is limited by weather and soil conditions.
 The major salt producing centres are:
o Marine Salt works along the coast of Gujarat (Jamnagar, Mithapur, Jhakhar,
Chira, Bhavnagar, Rajula, Dahej, Gandhidham, Kandla, Maliya, Lavanpur),
Tamil Nadu (Tuticorin, Vedaranyam, Covelong), Andhra Pradesh
(Chinnaganjam, Iskapalli, Krishnapatnam, Kakinada & Naupada), Maharashtra
(Bhandup, Bhayandar, Palghar), Orissa (Ganjam, Sumadi) and West Bengal
(Contai).
o Inland Salt Works in Rajasthan using lake brine and sub-soil brine viz.
Sambhar Lake, Nawa, Rajas, Kuchhaman, Sujangarh and Phalodi
o Salt works in Rann of Kutch using sub-soil brine viz: Kharaghoda,
Dhrangadhra; Santalpur
o Rock Salt Deposits at Mandi in the State of Himachal Pradesh
PROFILE & STATUS OF SALT INDUSTRY
o Gujarat, Tamil Nadu and Rajasthan are surplus Salt producing States
accounting for about 96 per cent of the Country’s production.

 9446331522 9446331522 www.neoias.com Page 77


https://t.me/testseries4exams
Download All PDF form here :- https://t.me/pdf4exams
NEO IAS Most Important 150 Environmental Science Model Questions from TPS 2018

o Gujarat contributes 76.7 per cent to the total production, followed by Tamil
Nadu (11.16 %) and Rajasthan (9.86%).
o The rest 2.28% production comes from Andhra Pradesh, Maharashtra, Orissa,
Karnataka, West Bengal, Goa, Himachal Pradesh, Diu & Daman.
o Government of India has adopted the strategy of Universal Salt Iodisation and
Consumption for elimination of Iodine Deficiency Disorders (IDD) in the
country under the National Iodine Deficiency Disorders Control Programme
(NIDDCP). Iodine is supplemented in the diet through Iodised Salt for combating
IDD. The Programme was started in 1962 initially confining to Goitre endemic
areas but after 1984 it was implemented throughout the country.

136. With reference to Wildlife Conservation Trust (WCT), consider the following
statements.
1. WCT is a not-for-profit non-governmental organisation working on wildlife protection
2. WCT lays equal emphasis on wildlife conservation and community development.
3. WCT conducts large-scale tiger population estimation programmes in corridors and
habitats outside protected areas.
4. WCT conducts diagnostic health check-up camps and provides clinical consultation
to the frontline forest staff
Which of the statements given above is/ are correct?
(a) 1, 2 and 3 only (b) 2 and 4 only (c) 1 and 3 only (d) 1, 2, 3 and 4

Answer (d)
Explanation
 Wildlife Conservation Trust (WCT) a not-for-profit non-governmental organisation.
 With several million people living in and around forests, we cannot separate
communities from conservation. Recognising this, WCT has adopted a 360°
approach to conservation by focusing equally on wildlife protection and
community development.
 WCT play a catalytic role in the landscapes they operate in, by collaborating to
strengthen sustainable ecosystems that will continue to serve wildlife and its
surrounding communities in the long term. To achieve this, WCT partner with
government bodies and local NGOs to provide funding, technical support and
consultancy.
 WCT help impart vocational training to young people and co-ordinate with over
100 job providers to find them gainful employment, thereby reducing their
dependency and negative impact on forests.
 WCT understand the value of education to a child’s future and work extensively
with government schools in forests by building the capacity of teachers,
providing infrastructural support and creating alternative avenues for learning.
 Realising that the remotely-located communities lack access to quality healthcare,
WCT conduct health camps, providing relief to both villagers and forest
department staff. WCT conducts diagnostic health check-up camps and provides
clinical consultation to the frontline forest staff, including permanent (forest
guards) and temporary (watchers) staff.
 They conduct scientific research to push for more robust wildlife management
policies.

 9446331522 9446331522 www.neoias.com Page 78


https://t.me/testseries4exams
Download All PDF form here :- https://t.me/pdf4exams
NEO IAS Most Important 150 Environmental Science Model Questions from TPS 2018

 WCT support the State and Central government in the field of tiger conservation.
WCT is an NGO that conducts large-scale tiger population estimation
programmes in corridors and habitats outside protected areas.
 WCT is a member of the Advisory Board of the Global Tiger Forum (GTF).
 WCT is a member of the Advisory Board of National Tiger Conservation Authority
(NTCA).

137. With reference to community in ecosystems, consider the following statements:


1. The term community refers to the population of different kinds of organisms living
together and sharing the same habitat.
2. It consists of all of the species living in an area.
3. It consists of interacting populations.
Which of the statements given above is/are correct?
(a) 1 only (b) 1 and 2 only (c) 1, 2 and 3 (d) None

Answer (c)
Explanation
 A population of a single species cannot survive by itself because there is inter
dependence of one form of life on another.
 An ecological community is a naturally occurring group of native plants, animals
and other organisms that are interacting in a unique habitat.
 Its structure, composition and distribution are determined by environmental factors
such as soil type, position in the landscape, altitude, climate and water availability.
 A community is bound together by the network of influences that species have on
one another.
 The term community refers to the population of different kinds of organisms living
together and sharing the same habitat.
 It consists of all of the species living in an area.
 It consists of interacting populations.

138. ‘PETA’, an NGO often seen in the news, focuses mainly on


(a) Health and sanitation
(b) Child rights
(c) Animal rights
(d) Eradication of corruption

Answer (c)
Explanation
 PETA is the largest animal rights organization in the world, with more than 5 million
members and supporters.
 PETA India operates under the simple principle that animals are not ours to eat,
wear, experiment on or use for entertainment, while educating policymakers and the
public about animal abuse and promoting an understanding of the right of all
animals to be treated with respect.

139. With reference to ‘Batagur baska’, a reptile, which of the following statements is/are
correct?
1. They are freshwater turtles
2. It is not found naturally in India
3. It is given legal protection under Schedule I of the Wildlife (Protection) Act, 1972.
Select the correct answer using the codes given below.

 9446331522 9446331522 www.neoias.com Page 79


https://t.me/testseries4exams
Download All PDF form here :- https://t.me/pdf4exams
NEO IAS Most Important 150 Environmental Science Model Questions from TPS 2018

(a) 1 and 2 (b) 2 only (c) 1 and 3 (d) 3 only

Answer (c)
Explanation
 They are freshwater turtles.
 The terrapin, has a river estuarine habitat.
 India ranks third in the list of countries with the largest number of threatened turtle
and tortoise species in the world after China and Vietnam.
 The country has two of the world’s 25 most threatened freshwater turtle species –
Northern River Terrapin (Batagur baska) found in the Sudarbans, West Bengal, and
the Red-Crowned Roof Turtle (Batagur kachuga), found only within the riverine
National Chambal Gharial Wildlife Sanctuary (NCGWS) in Madhya Pradesh.
 Three fresh water ponds in the Sunderbans Tiger Reserve house the rare Northern
river terrapin (Batagur baska), whose presence in the wild in West Bengal and
Odisha had declined to undetectable levels a decade ago.
 Sajnekhali wildlife sanctuary host over 200 of these turtles.
 Batagur baska, the 60-cm-long turtle that is presumed extinct in several Southeast
Asian countries, is classified as critically endangered by the International Union for
Conservation of Nature (IUCN) in its Red List of threatened species.
 For the past ten years, officials of the Sunderban Tiger Reserve with support from
experts at Turtle Survival Alliance (TSA), have coordinated a recovery program for
what is described as the world’s second most endangered turtle, through captive
conservation breeding.
 In India, the major threats include illegal trade of these species, drowning in fishing
nets (which is a major threat to Red-Crowned Roof Turtle), habitat loss, and
degradation due to sand mining, dam construction, and pollution.

140. With reference to productivity, consider the following statements:


1. Primary productivity is the rate of production of biomass by the producers in an
ecosystem.
2. Annual productivity of an ecosystem is greater than the annual increase in biomass
of the herbivores in the ecosystem
Which of the statements given above is/are correct?
(a) 1 only (b) 2 only (c) Both 1 and 2 (d) Neither 1 nor 2

Answer (c)
Explanation
 Primary productivity is the rate of production of biomass by the producers in an
ecosystem. OR Primary productivity is the amount of biomass or organic matter
produced per unit area over a period of time in the first trophic level.
 During each energy transformations some of the energy is lost because of the
respiration, so the annual productivity of the ecosystem will be greater than the
annual increase in biomass of the herbivores in the ecosystem.

141. Consider the following statements:


1. River Ganges is perennial because it is fed by the melting glacier
2. The perennial behaviour of Cauvery, Thamirabarani and Vaigai rivers are due to the
presence of shola forests.
Which of the statements given above is/are correct?
(a) 1 only (b) 2 only (c) Both 1 and 2 (d) Neither 1 nor 2
 9446331522 9446331522 www.neoias.com Page 80
https://t.me/testseries4exams
Download All PDF form here :- https://t.me/pdf4exams
NEO IAS Most Important 150 Environmental Science Model Questions from TPS 2018

Answer (c)
Explanation
 Shola forest are the source of water in rivers live Cauvery, Thamirabarani, Vaigai.
Unlike other rivers in the peninsular region these rivers are perennial and they never
go dry like the Ganges in the North India.
 River Ganges is perennial because it is fed by the melting glacier all the year
round. There is no ice in the Western Ghats and yet these rivers manage to supply
water all-round the year. The reason behind this is the presence of Shola forests.
 For Tamil Nadu, protecting the health of the shola-grassland ecosystem and the
other forests in the catchment would mean protecting the source of water flowing
through the perennial rivers.

142. Consider the following statements.


1. Bioaccumulation is a process of progressive accumulation of heavy metals and
pesticides in an organism.
2. Large fishes of the pond are found to have higher concentration of pesticides than
planktons of the same pond.
3. Bioconcentration and bioaccumulation happen within an organism, but
biomagnification occurs across levels of the food chain.
Which of the statements given above is/ are correct?
(a) 1 and 2 only (b) 2 and 3 only (c) 1 and 3 only (d) 1, 2 and 3

Answer (d)
Explanation
 Bioaccumulation refers to the accumulation of a toxic chemical in the tissue of a
particular organism.
 Biomagnification refers to an increase in the concentration of a toxic chemical as
you move up the food chain.
 Bioaccumulation occurs within an organism, where a concentration of a substance
builds up in the tissues and is absorbed faster than it is removed.
 Bioaccumulation often occurs in two ways, simultaneously: by eating contaminated
food, and by absorption directly from water. This second case is specifically referred
to as bioconcentration.
 Bioconcentration and bioaccumulation happen within an organism, but
biomagnification occurs across levels of the food chain.

143. With reference to the International Union for Conservation of Nature and Natural
Resources (IUCN), which of the following statements is/are correct?
1. IUCN is a membership union composed of both government and civil society
organisations (NGOs) for the conservation of biodiversity
2. IUCN has an official Observer Status at the United Nations General Assembly
3. IUCN runs field projects around the world to better manage natural environments.
Select the correct answer using the code given below.
(a) 1 only (b) 2 and 3 only (c) 1 and 3 only (d) 1, 2 and 3

Answer (d)
Explanation

 9446331522 9446331522 www.neoias.com Page 81


https://t.me/testseries4exams
Download All PDF form here :- https://t.me/pdf4exams
NEO IAS Most Important 150 Environmental Science Model Questions from TPS 2018

 International Union for Conservation of Nature (IUCN) is also known as the World
Conservation Union
 Former name - International Union for the Protection of Nature (IUPN)
 Membership union composed of both government and civil society organisations
(NGOs) for the conservation of biodiversity
 Established in 1948, headquarters in Gland, Switzerland
 Has official Observer Status at the United Nations General Assembly
 Prepare a list called IUCN Red List (Red Data Book), which classify the plants and
animals on the basis of their threat
 It provides public, private and non-governmental organisations with the knowledge
and tools that enable human progress, economic development and nature
conservation to take place together.
 It is a leading provider of conservation data, assessments and analysis.
 They run hundreds of field projects around the world to better manage natural
environments.

144. Consider the following statements:


1. Eastern Ghats are spread over five States
2. Gadgil Committee Report’ and ‘Kasturirangan Committee Report’, sometimes seen in
the news, are related to the Protection of Eastern Ghats
3. Easter Ghats is one of the four biodiversity hotspots India.
Which of the statements given above is/are correct?
(a) 1 only (b) 2 and 3 only (c) 1 and 3 only (d) None

Answer (a)
Explanation
 The 1,700-km-long Eastern Ghat spread across Odisha, Andhra Pradesh,
Telangana, Tamil Nadu and Karnataka on the east coast of India is an important
physiographic unit with immense bio-geographic, environmental, socio-economic,
cultural and spiritual significance.
 According to NGO Greens’ Alliance for Conservation of Eastern Ghats (GrACE),
deforestation, haphazard mining, construction of dams, rapid land use changes,
forest fires, temple tourism, agricultural and industrial encroachments, and burn
cultivations have caused severe damage to the eco-system.
 Gadgil Committee Report’ and ‘Kasturirangan Committee Report’, are related to
the Protection of Western Ghats
 India has four biodiversity hotspots
o Himalaya: Includes the entire Indian Himalayan region (and that falling in
Pakistan, Tibet, Nepal, Bhutan, China and Myanmar)
o Indo-Burma: Includes entire North-eastern India, except Assam and Andaman
group of Islands (and Myanmar, Thailand, Vietnam, Laos, Cambodia and
southern China)
o Sundalands: Includes Nicobar group of Islands (and Indonesia, Malaysia,
Singapore, Brunei, Philippines)
o Western Ghats and Sri Lanka: Includes entire Western Ghats (and Sri Lanka)

145. With reference to the natural vegetation of India, consider the following statements:
1. Teak and Sal are found in the moist deciduous forest.
2. Shola forests are found mainly on the upper reaches of Himalaya.
3. Khejri tree is a dominant tree in the north eastern rain forest.
Which of the statements given above is/are correct?
(a) 1 only (b) 2 and 3 only (c) 1 and 3 only (d) 1, 2 and 3

 9446331522 9446331522 www.neoias.com Page 82


https://t.me/testseries4exams
Download All PDF form here :- https://t.me/pdf4exams
NEO IAS Most Important 150 Environmental Science Model Questions from TPS 2018

Answer (a)
Explanation
 Sal tree and teak are deciduous trees and is a part of deciduous forests.
 Shola forests are tropical Montane forests found in the valleys separated by
rolling grasslands only in the higher elevations. They are found only in South
India in the Southern Western Ghats. The shola forests are patches of forests that
occur only in the valleys where there is least reach of the fog and mist. Other parts
of the mountains are covered in grasslands. The trees never grow on the mountain
tops. This is such a unique landscape formation that is native only to the southern
Western Ghats.
 Khejri tree or Jandi tree is a species in the arid regions. It plays a vital role in
preserving the ecosystem of arid and semi-arid areas. Basically jandi tree is a
symbol of socio-economic development of the arid regions. Since all the parts of the
tree are useful, it is called kalp taru. It is also known as the ‘king of desert’, and
the ‘wonder tree’.

146. Which one of the following gases is mainly released from the landfills in urban
areas?
(a) Oxygen (b) Hydrogen
(c) Nitrogen (d) Methane

Answer (d)
Explanation
 Landfill gas contains many different gases. Landfill gas (LFG) is a natural byproduct
of the decomposition of organic material in landfills. LFG is composed of roughly 50
percent methane (the primary component of natural gas), 50 percent carbon dioxide
(CO2) and a small amount of non-methane organic compounds. Methane is a potent
greenhouse gas 28 to 36 times more effective than CO2 at trapping heat in the
atmosphere over a 100-year period, per the latest Intergovernmental Panel on
Climate Change (IPCC) assessment report (AR5).
 Landfill gases are produced when bacteria break down organic waste. The amount of
these gases depends on the type of waste present in the landfill, the age of the
landfill, oxygen content, the amount of moisture, and temperature. For example, gas
production will increase if the temperature or moisture content increases. Though
production of these gases generally reaches a peak in five to seven years, a landfill
can continue to produce gases for more than 50 years.

147. Which one of the following terms describes the amount of the environment
necessary to produce the goods and services necessary to support a particular lifestyle?
(a) Carrying Capacity (b) Home Range
(c) Ecological Footprint (d) Edge Effect

Answer (c)
Explanation
 Ecological footprint:
The impact of human activities measured in terms of the area of biologically
productive land and water required to produce the goods consumed and to
assimilate the wastes generated.
More simply, it is the amount of the environment necessary to produce the goods
and services necessary to support a particular lifestyle.
The Ecological Footprint is defined as the area of productive land and water
ecosystems required for the production of the resources that the population

 9446331522 9446331522 www.neoias.com Page 83


https://t.me/testseries4exams
Download All PDF form here :- https://t.me/pdf4exams
NEO IAS Most Important 150 Environmental Science Model Questions from TPS 2018

consumes and assimilate the wastes that the population produces, wherever on
Earth the land and water is located. It is the environmental input of a person or
population.
 Carrying Capacity: The maximum number of individuals of a given species that
an area's resources can sustain indefinitely without significantly depleting or
degrading those resources.
 Home Range: The amount of space an animal uses on a regular basis. It is the
region that encompasses all the resources the animal requires to survive and
reproduce.
 Edge Effect: A transitional area of vegetation between two different ecosystems or
plant communities, such as forest and grassland is called as Ecotone. The tendency
for increased biodiversity within the ecotone is referred to as the "edge effect."

148. With reference to ‘National Air Quality Index’, sometimes seen in the news, consider
the following statements:
1. It is a tool for effective dissemination of air quality information to people by
describing as a ‘One Number- One Colour-One Description’
2. Carbon dioxide is a major parameter in the Air quality index.
Which of the statements given above is/are correct?
(a) 1 only (b) 2 only (c) Both 1 and 2 (d) Neither 1 nor 2

Answer (a)
Explanation
 The new National Air Quality Index (AQI) launched in 2014 can be described as a
‘One Number- One Colour-One Description’ for the common man to judge the air
quality within his vicinity.
 Air Quality Index (AQI) is a tool for effective dissemination of air quality
information to people.
 The current measurement index for air quality will consider eight pollutants or
parameters (PM10, PM2.5, Nitrogen dioxide (NO2), Sulfur dioxide (SO2), Carbon
monoxide (CO), ozone (O3), Ammonia (NH3), and Lead (Pb)).
 There are six AQI categories, namely Good, Satisfactory, Moderately polluted,
Poor, Very Poor, and Severe.

149. Green Economy Initiative is launched by


(a) World Bank
(b) Asian Infrastructure Investment Bank (AIIAB)
(c) United Nations Environment Programme (UNEP)
(d) Intergovernmental Panel on Climate Change (IPCC)

Answer (c)
Explanation
 In 2008, UN Environment launched the Green Economy Initiative (GEI), a
programme of global research and country-level assistance designed to motivate
policymakers to support environmental investments. At the UN General Assembly
2015, UN Environment published “Uncovering pathways towards an inclusive green
economy”.
 United Nations Environment launched the Green Economy Initiative (GEI) in 2008,
which consisted of global research and country-level assistance encouraging
policymakers to support environmental investments within the context of
sustainable development. “Green economy in the context of sustainable development

 9446331522 9446331522 www.neoias.com Page 84


https://t.me/testseries4exams
Download All PDF form here :- https://t.me/pdf4exams
NEO IAS Most Important 150 Environmental Science Model Questions from TPS 2018

and poverty eradication” was placed on the 2012 Rio+20 agenda and was
acknowledged as a tool for achieving sustainable development.
 The United Nations Environment Programme (UN Environment) is the leading global
environmental authority that sets the global environmental agenda, promotes the
coherent implementation of the environmental dimension of sustainable
development within the United Nations system, and serves as an authoritative
advocate for the global environment.

150. With reference to the cactus, a plant lives in hot and dry habitats, which of the
following statements is/are correct?
1. They uses Crassulacean acid metabolism in photosynthesis process.
2. Their leaves are reduced to spines to reduce water loss.
3. The stem of cactus does the photosynthesis.
Select the correct answer using the code given below.
(a) 1 only (b) 2 and 3 only (c) 1 and 3 only (d) 1, 2 and 3

Answer (d)
Explanation
 Cactus is a type of plant that can store large amounts of water and survive in
extremely hot and dry habitats.
 Almost all cacti are native to deserts and dry regions of South and North America.
Due to their attractive morphology, cacti can be found throughout the world today.
 They have waxy substance on the surface which prevents loss of water via
transpiration.
 Cacti have spines instead of leaves.
 Spines have two major roles: they prevent loss of water via transpiration and keep
the plant safe from animals.
 Since cacti live in dry areas, they need to absorb large amount of water and store it
in the stem and roots for the periods of drought. Besides storing of water, stem
plays role in the process of photosynthesis (production of food by using the
sunlight and carbon dioxide). In general, plants photosynthesize through their leaves
and, to a much lesser extent, through their stems.
 While different cactuses may have specialized photosynthetic techniques, in general,
photosynthesis occurs in a cactus' stem or trunk. The concept of photosynthesis in
a cactus or succulent plant may seem strange considering that most of them do not
have obvious leaves like many other plants, but their stems or trunks serve the
same photosynthetic function as some other plants' leaves.
 Water from cactus has higher density compared with tap water, but it is safe for
drinking. Cacti can survive from 15 to 300 years, depending on the species.
 Photosynthetic stems must also have stomate, pores, through which gas exchange
takes place.
 There is a functional trade-off problem because while gas exchange is necessary for
photosynthesis, water is lost through the pores. Cacti are among a group of plants
that solve this problem by capturing solar energy by day, and finishing the
photosynthesis by night when the stomata can be open with minimal water
loss. They use Crassulacean acid metabolism (CAM).

 9446331522 9446331522 www.neoias.com Page 85


https://t.me/testseries4exams
Download All PDF form here :- https://t.me/pdf4exams
NEO IAS Most Important 150 Environmental Science Model Questions from TPS 2018

 9446331522 9446331522 www.neoias.com Page 86


https://t.me/testseries4exams

S-ar putea să vă placă și